CME4PAC

Online CME for Physician Assistants

Menu

Skip to content
  • Home
  • Membership Benefits
  • My Account
  • About Us
  • Log In
  • Contact Us
  • Ortho on the Go

Free CME

Take your free CME quiz below

Time limit: 0

Quiz-summary

0 of 50 questions completed

Questions:

  1. 1
  2. 2
  3. 3
  4. 4
  5. 5
  6. 6
  7. 7
  8. 8
  9. 9
  10. 10
  11. 11
  12. 12
  13. 13
  14. 14
  15. 15
  16. 16
  17. 17
  18. 18
  19. 19
  20. 20
  21. 21
  22. 22
  23. 23
  24. 24
  25. 25
  26. 26
  27. 27
  28. 28
  29. 29
  30. 30
  31. 31
  32. 32
  33. 33
  34. 34
  35. 35
  36. 36
  37. 37
  38. 38
  39. 39
  40. 40
  41. 41
  42. 42
  43. 43
  44. 44
  45. 45
  46. 46
  47. 47
  48. 48
  49. 49
  50. 50

Information

Approved for 5 Category 1 SA Credits 
Learning Objectives: At the conclusion of the sports self-assessment exam physician assistants should be familiar with the diagnosis and treatment of femoroacetabular impingement, be able to recognize peripheral nerve entrapement of the upper extremities, be able to recognize and know the indications for different types of physical exam tests of the shoulder and hip, know which type of meniscus tears are amenable to being repaired, know the treatment of a bony mallet finger, be able to describe the pathoanatomy of a sports hernia, be able to diagnose common causes of leg pain, understand the treatment of avulsion fractures of the hip, be aware of the different types of AC joint separation, know when athletes should return to play after a concussion, know the potential causes of rotator cuff repair failure, know medications that could potentially cause tendinitis, know causes of elbow pain in the throwing athlete, know treatment of labral tears, know treatment of partial cuff tears, know graft choices available for ACL reconstruction, and know treatment of patella dislocations

You have already completed the quiz before. Hence you can not start it again.

Quiz is loading...

You must sign in or sign up to start the quiz.

You have to finish following quiz, to start this quiz:

Results

0 of 50 questions answered correctly

Your time:

Time has elapsed

You have reached 0 of 0 points, (0)

Average score
 
 
Your score
 
 

Categories

  1. Not categorized 0%
  • You are required to complete the post-exam evaluation to receive CME credits. Please complete the post-exam evaluation here. Your CME certificate will be emailed to you shortly. If you don’t receive your certificate today, and the certificate email from CME4PAC is not located in your spam folder, please email, Dagan Cloutier, at dcloutier@cme4pac.com.

  1. 1
  2. 2
  3. 3
  4. 4
  5. 5
  6. 6
  7. 7
  8. 8
  9. 9
  10. 10
  11. 11
  12. 12
  13. 13
  14. 14
  15. 15
  16. 16
  17. 17
  18. 18
  19. 19
  20. 20
  21. 21
  22. 22
  23. 23
  24. 24
  25. 25
  26. 26
  27. 27
  28. 28
  29. 29
  30. 30
  31. 31
  32. 32
  33. 33
  34. 34
  35. 35
  36. 36
  37. 37
  38. 38
  39. 39
  40. 40
  41. 41
  42. 42
  43. 43
  44. 44
  45. 45
  46. 46
  47. 47
  48. 48
  49. 49
  50. 50
  1. Answered
  2. Review
  1. Question 1 of 50
    1. Question

    A 17-year old male cross country runner presents to the office with a history of activity related bilateral leg pain for 8 months.  The pain becomes increasingly painful after a few minutes of running which forces him to stop. The pain slowly subsides after a several minutes of rest. On exam the patient has no pain to palpation along the posterior-medial tibial crest. He ambulates with a normal gait and has no pain with single leg hops bilaterally.  X-rays and MRI’s of bilateral lower extremities are normal. What is the most likely diagnosis?

    Correct

    Chronic compartment syndrome, or exercised induced compartment syndrome, is an uncommon cause of leg pain in athletes who participate in repetitive impact exercises.   The muscles, nerves, and vessels of the lower extremities are contained within four compartments; these include the anterior, lateral, superficial posterior, and deep posterior compartments. During exercise muscle weight can increase by up to 20% which forces the surrounding fascia to expand to accommodate the increased volume. When the fascia fails to expand to accommodate the increase in muscle volume compartmental pressures can increase above diastolic blood pressure.  As a result, arteriole blood is decreased and can become insufficient to meet the needs of the muscle. Inadequate tissue perfusion causes ischemia and increasing pain with activities. Athletes describe the pain as predictable, deep, and achy pain that is relieved with rest. Physical exam in the office setting is almost always normal. An MRI or bone scan is often ordered to rule out a stress fracture and medial tibial stress syndrome. The diagnosis of chronic compartment syndrome is made by measuring the resting and post-exercise compartment pressures. Normal compartment pressures are between 0 and 10 mm Hg. Chronic compartment syndrome can be diagnosed with a resting pressure > 15 mm Hg,  1-minute post exertion pressure > 30, or a 5-minute postexertional pressure > 20.  Surgical fasciotomy is the definitive treatment of choice with a return to sports rate of over 90%.

    The term medial tibial stress syndrome (MTTS) describes a condition of periostitis at the muscular attachment site along the posterior medial tibia. MTTS causes exertion pain in athletes that is relieved with rest. Risk factors may include excessive foot pronation, hindfoot varus, and changes in exercise routine including intensity and duration.  On exam, pain to palpation may be noted over the posterior medial tibia. Pain may increase with resisted plantar flexion and with toe raises.  MRI or bone scan will likely show increased activity along the posteriormedial tibia. Treatment is usually successful with a period of rest, immobilization, and NSAIDs.

    Stress fractures of the tibia occur when repetitive stress is placed on bone that fails to respond to mechanical loads placed upon it. This failure results in the accumulation of microfractures that can result in a stress fracture.  Women are 12 times more likely to have a stress fracture than males and 72% of stress fractures occur in runners. Athletes with stress fractures complain of increasing pain over the middle to distal posteriormedial tibia. Physical exam reveals localized boney tenderness and patients may be unable to perform a single leg hop. Radiographs are often negative, particularly early in presentation. MRI will show bone edema, and in some cases, a fracture line. Treatment is usually successful with a period of rest, immobilization, and NSAIDs. Athletes should not return to play until daily activities are pain free and no tenderness to palpation is present over the previous fracture site.

    Popliteal artery entrapement syndrome (PAES) is a rare condition caused by a partial or completely obstructed popliteal artery. This may be caused by a congenitally abnormal medial head of the gastrocnemius muscle compressing on the popliteal artery. Functional popliteal artery compression has also been found to be caused by compression of the popliteus muscle during repetitive use. Symptoms include intermittent claudication that worsens with activity and is relieved with rest.  PAES occurs bilaterally in up to 25% of patients which unlike chronic compartment syndrome which occurs in up to 95% bilaterally. Decreased pulses and vascular skin changes are rarely found with PAES. Computed tomographic angiogram findings help to confirm the diagnosis. Vascular surgery consultation is the most appropriate initial step when PAES is suspected.

    References
    Meininger AK, Turnipseed WD, Hutchinson MR. Recognition and treatment of leg pain in athletes. Orthopedic Online Journal 2009. 7(2).

    Fraipoint MJ, Adamson GJ. Chronic Exertional Compartment Syndrome. J Am Acad Orthop Surg 2003; 11; 268-276.

     

    Incorrect

    Chronic compartment syndrome, or exercised induced compartment syndrome, is an uncommon cause of leg pain in athletes who participate in repetitive impact exercises.   The muscles, nerves, and vessels of the lower extremities are contained within four compartments; these include the anterior, lateral, superficial posterior, and deep posterior compartments. During exercise muscle weight can increase by up to 20% which forces the surrounding fascia to expand to accommodate the increased volume. When the fascia fails to expand to accommodate the increase in muscle volume compartmental pressures can increase above diastolic blood pressure.  As a result, arteriole blood is decreased and can become insufficient to meet the needs of the muscle. Inadequate tissue perfusion causes ischemia and increasing pain with activities. Athletes describe the pain as predictable, deep, and achy pain that is relieved with rest. Physical exam in the office setting is almost always normal. An MRI or bone scan is often ordered to rule out a stress fracture and medial tibial stress syndrome. The diagnosis of chronic compartment syndrome is made by measuring the resting and post-exercise compartment pressures. Normal compartment pressures are between 0 and 10 mm Hg. Chronic compartment syndrome can be diagnosed with a resting pressure > 15 mm Hg,  1-minute post exertion pressure > 30, or a 5-minute postexertional pressure > 20.  Surgical fasciotomy is the definitive treatment of choice with a return to sports rate of over 90%.

    The term medial tibial stress syndrome (MTTS) describes a condition of periostitis at the muscular attachment site along the posterior medial tibia. MTTS causes exertion pain in athletes that is relieved with rest. Risk factors may include excessive foot pronation, hindfoot varus, and changes in exercise routine including intensity and duration.  On exam, pain to palpation may be noted over the posterior medial tibia. Pain may increase with resisted plantar flexion and with toe raises.  MRI or bone scan will likely show increased activity along the posteriormedial tibia. Treatment is usually successful with a period of rest, immobilization, and NSAIDs.

    Stress fractures of the tibia occur when repetitive stress is placed on bone that fails to respond to mechanical loads placed upon it. This failure results in the accumulation of microfractures that can result in a stress fracture.  Women are 12 times more likely to have a stress fracture than males and 72% of stress fractures occur in runners. Athletes with stress fractures complain of increasing pain over the middle to distal posteriormedial tibia. Physical exam reveals localized boney tenderness and patients may be unable to perform a single leg hop. Radiographs are often negative, particularly early in presentation. MRI will show bone edema, and in some cases, a fracture line. Treatment is usually successful with a period of rest, immobilization, and NSAIDs. Athletes should not return to play until daily activities are pain free and no tenderness to palpation is present over the previous fracture site.

    Popliteal artery entrapement syndrome (PAES) is a rare condition caused by a partial or completely obstructed popliteal artery. This may be caused by a congenitally abnormal medial head of the gastrocnemius muscle compressing on the popliteal artery. Functional popliteal artery compression has also been found to be caused by compression of the popliteus muscle during repetitive use. Symptoms include intermittent claudication that worsens with activity and is relieved with rest.  PAES occurs bilaterally in up to 25% of patients which unlike chronic compartment syndrome which occurs in up to 95% bilaterally. Decreased pulses and vascular skin changes are rarely found with PAES. Computed tomographic angiogram findings help to confirm the diagnosis. Vascular surgery consultation is the most appropriate initial step when PAES is suspected.

    References
    Meininger AK, Turnipseed WD, Hutchinson MR. Recognition and treatment of leg pain in athletes. Orthopedic Online Journal 2009. 7(2).

    Fraipoint MJ, Adamson GJ. Chronic Exertional Compartment Syndrome. J Am Acad Orthop Surg 2003; 11; 268-276.

  2. Question 2 of 50
    2. Question

    Which meniscal tear is the most amenable to being repaired?

    Correct

    The main obstacle that orthopedic surgeons encounter when deciding upon a surgical approach is the location and type of the meniscal tear.  The meniscus is divided into two regions based on local blood supply. The peripheral one third of the meniscus is referred to as the red zone and the central portion is referred to as the white zone. The two zones merge at the red-white junction. The red zone is supplied by branches of the popliteal artery, collectively known as the geniculate arteries. Therefore, this region is composed of highly vascularized tissue. Central to the red-white junction is the white zone. This region received its name due to its lack of arterial blood supply. The white zone was thought to be completely avascular receiving absolutely no contact with oxygenated blood.  However, studies have proven that this theory is not entirely true. Although the white zone has no arterial supply, it is believed that it does absorb a rather limited amount of blood from the circulating synovial fluid. Because proper vascularization promotes tissue healing, simple meniscal tears located within the red zone have been deemed prime candidates for repair. Such lesions carry a surgical success rate of 90% percent. Whereas, lesions located greater than 4 mm from the vascularized peripheral ridge have a success rate of only 50% percent. Historically, when a lesion is located within the avascular central two thirds of the meniscus, or greater than 3 mm from the peripheral rim, surgical repair is contraindicated and arthroscopic partial debridement becomes the treatment of choice.

    In addition to the vascular characteristics of a meniscal lesion, clinicians must also formulate the appropriate surgical approach based on the tear pattern. Meniscal tears are classified as vertical, horizontal, oblique, or complex; some being more amendable to repair than others. Vertical tears are defined as meniscal lesions that run in a perpendicular plane to the tibial plateau. Vertical tears can be further subdivided into vertical longitudinal, vertical radial, and bucket handle tears. A vertical longitudinal tear, which is the most commonly encountered tear pattern in adolescents, is a vertical tear that runs circumferentially with the long axis of meniscal fiber arrangement. Vertical radial tears originate from the central avascular zone and travel toward the periphery in a perpendicular plane relative to the tibial plateau. A bucket handle tear is defined as a complete vertical longitudinal tear with displacement into the tibiofemoral joint, often causing complete mechanical obstruction and requiring urgent surgical intervention. Vertical longitudinal tears, especially those complete in nature and located in the vascular periphery carry the greatest repair potential. However, as long as complexity and white-white zone involvement is minimal, all vertical meniscal tears have some degree of repair potential. Horizontal tears are seen more commonly in later adulthood and are classified as lesions running along the transverse plane, parallel to the tibial plateau. Oblique tears are simply meniscal flaps that are not confined to a particular plane (vertical or horizontal). Lastly, complex tears are intricate lesions containing at least two different tear patterns. Complex tears are often the result of long term degenerative changes and are the least amendable to repair of all tear patterns.

    References
    Noyes F.R. , Barber-Westin S.D. Management of meniscus tears that extend into the avascular region. Clinics in sports Medicine. 2012; 31: 65-90.

    Fox  A.J.S, Bedi A., Rodeo S.A. The basic science of human knee menisci structure, composition, and function. Sports Health: a multidisciplinary approach. 2012; 4: 340- 351.

    Shybut T., Strauss E.J.. Surgical Management of Meniscal Tears. Bulletin of the NYU Hospital for Joint Disease. 2011: 69 (1): 56-52.

    Krych A.J. et al. Arthroscopic repair of isolated meniscal tears in patients 18 years and younger. The American journal of sports medicine. 2008; 36 (7) 1283-1289.

    Maffulli N., Giuseppe Longo U., Campi S., Denaro V. Meniscal Tears. Dove Press Journal: open access journal of sports medicine. 2010; 45-54.

    Incorrect

    The main obstacle that orthopedic surgeons encounter when deciding upon a surgical approach is the location and type of the meniscal tear.  The meniscus is divided into two regions based on local blood supply. The peripheral one third of the meniscus is referred to as the red zone and the central portion is referred to as the white zone. The two zones merge at the red-white junction. The red zone is supplied by branches of the popliteal artery, collectively known as the geniculate arteries. Therefore, this region is composed of highly vascularized tissue. Central to the red-white junction is the white zone. This region received its name due to its lack of arterial blood supply. The white zone was thought to be completely avascular receiving absolutely no contact with oxygenated blood.  However, studies have proven that this theory is not entirely true. Although the white zone has no arterial supply, it is believed that it does absorb a rather limited amount of blood from the circulating synovial fluid. Because proper vascularization promotes tissue healing, simple meniscal tears located within the red zone have been deemed prime candidates for repair. Such lesions carry a surgical success rate of 90% percent. Whereas, lesions located greater than 4 mm from the vascularized peripheral ridge have a success rate of only 50% percent. Historically, when a lesion is located within the avascular central two thirds of the meniscus, or greater than 3 mm from the peripheral rim, surgical repair is contraindicated and arthroscopic partial debridement becomes the treatment of choice.

    In addition to the vascular characteristics of a meniscal lesion, clinicians must also formulate the appropriate surgical approach based on the tear pattern. Meniscal tears are classified as vertical, horizontal, oblique, or complex; some being more amendable to repair than others. Vertical tears are defined as meniscal lesions that run in a perpendicular plane to the tibial plateau. Vertical tears can be further subdivided into vertical longitudinal, vertical radial, and bucket handle tears. A vertical longitudinal tear, which is the most commonly encountered tear pattern in adolescents, is a vertical tear that runs circumferentially with the long axis of meniscal fiber arrangement. Vertical radial tears originate from the central avascular zone and travel toward the periphery in a perpendicular plane relative to the tibial plateau. A bucket handle tear is defined as a complete vertical longitudinal tear with displacement into the tibiofemoral joint, often causing complete mechanical obstruction and requiring urgent surgical intervention. Vertical longitudinal tears, especially those complete in nature and located in the vascular periphery carry the greatest repair potential. However, as long as complexity and white-white zone involvement is minimal, all vertical meniscal tears have some degree of repair potential. Horizontal tears are seen more commonly in later adulthood and are classified as lesions running along the transverse plane, parallel to the tibial plateau. Oblique tears are simply meniscal flaps that are not confined to a particular plane (vertical or horizontal). Lastly, complex tears are intricate lesions containing at least two different tear patterns. Complex tears are often the result of long term degenerative changes and are the least amendable to repair of all tear patterns.

    References
    Noyes F.R. , Barber-Westin S.D. Management of meniscus tears that extend into the avascular region. Clinics in sports Medicine. 2012; 31: 65-90.

    Fox  A.J.S, Bedi A., Rodeo S.A. The basic science of human knee menisci structure, composition, and function. Sports Health: a multidisciplinary approach. 2012; 4: 340- 351.

    Shybut T., Strauss E.J.. Surgical Management of Meniscal Tears. Bulletin of the NYU Hospital for Joint Disease. 2011: 69 (1): 56-52.

    Krych A.J. et al. Arthroscopic repair of isolated meniscal tears in patients 18 years and younger. The American journal of sports medicine. 2008; 36 (7) 1283-1289.

    Maffulli N., Giuseppe Longo U., Campi S., Denaro V. Meniscal Tears. Dove Press Journal: open access journal of sports medicine. 2010; 45-54.

  3. Question 3 of 50
    3. Question

    Which exam test is the most sensitive for diagnosing rotator cuff tendinitis caused by impingement syndrome?

    Correct

    Yergason’s test is performed by having the patients arm in an adducted  pronated position with the elbow in 90 degrees of flexion.  The examiner resists the patients attempt to actively supinate forearm and externally rotate the humerus. A positive finding is pain palpated over the bicipital groove indicating possible bicipital tendinitis.  Neer Impingement test is performed with the examiner holding one hand stabilizing the scapula and the other passively and maximally forward flexing the test shoulder.  Pain with this maneuver may indicate impingement syndrome.  Speeds test is performed with the patients shoulder at 90 degrees of forward flexion and the elbow in full extension. The examiner provides active resistance against forward flexion of the humerus. Pain along the bicipital grove may indicate bicipital tendinitis. O’brians test is performed with the patients should in 90 degrees of forward flexion, 30 to 45 degrees of horizontal adduction, and maximal internal rotation.  The patient adducts and flexes the shoulder against resistance. The test is than repeated with the patients arm in external rotation. Pain that is elicited with the arm internally rotated but absent when repeated in external rotation may indicate a SLAP lesion.

    Reference
    Konin JG, Wilksten DL, Isear JA, Brader H. Special Tests for Orthopedic Examination.  Third Edition. SLACK Incorporated. Thorofare, NJ. 2006.

    Incorrect

    Yergason’s test is performed by having the patients arm in an adducted  pronated position with the elbow in 90 degrees of flexion.  The examiner resists the patients attempt to actively supinate forearm and externally rotate the humerus. A positive finding is pain palpated over the bicipital groove indicating possible bicipital tendinitis.  Neer Impingement test is performed with the examiner holding one hand stabilizing the scapula and the other passively and maximally forward flexing the test shoulder.  Pain with this maneuver may indicate impingement syndrome.  Speeds test is performed with the patients shoulder at 90 degrees of forward flexion and the elbow in full extension. The examiner provides active resistance against forward flexion of the humerus. Pain along the bicipital grove may indicate bicipital tendinitis. O’brians test is performed with the patients should in 90 degrees of forward flexion, 30 to 45 degrees of horizontal adduction, and maximal internal rotation.  The patient adducts and flexes the shoulder against resistance. The test is than repeated with the patients arm in external rotation. Pain that is elicited with the arm internally rotated but absent when repeated in external rotation may indicate a SLAP lesion.

    Reference
    Konin JG, Wilksten DL, Isear JA, Brader H. Special Tests for Orthopedic Examination.  Third Edition. SLACK Incorporated. Thorofare, NJ. 2006.

  4. Question 4 of 50
    4. Question

    Which choice below is thought to be a potential cause of femoroacetabular impingement?

    Correct

    A definitive cause of FAI has not yet been established, however several theories have been proposed. Childhood hip diseases including Perthes and SCFE have been associated with an increased incidence of FAI. Most patients, however, do not show growth plate abnormalities. Acetabular retroversion, coxa profunda, protrusio acetaubli, and post-traumatic deformity are also known factors that can lead to impingement. Increased stresses across a developing physis of the femoral head during particular sports or hypermobility of the hip joint have been proposed as possible etiology for abnormalities of the proximal femur. For example, hockey players may be at higher risk of developing CAM type impingement due to the forceful lateral push off while skating.

    References
    Michael Leunig MD, Paul E. Beaule´ MD, Reinhold Ganz MD: The Concept of Femoroacetabular Impingement, Current Status and Future Perspectives. Clin Orthop Relat Res (2009) 467:616–622.

    Jack G. Skendzel, MD, Alexander E. Weber, MD, James R. Ross, MD, Christopher M. Larson, MD, Michael Leunig, MD, Bryan T. Kelly, MD, and Asheesh Bedi, MD: The Approach to the Evaluation and Surgical Treatment of Mechanical Hip Pain in the Young Patient. The Journal of Bone and Joint Surgery (2013) 95A.

    Justin D. Stull, Marc J. Philippon and Robert F. LaPrade ”At-Risk” Positioning and Hip Biomechanics of the Peewee Ice Hockey Sprint Start. Am J Sports Med 2011 39: 29S.

    Incorrect

    A definitive cause of FAI has not yet been established, however several theories have been proposed. Childhood hip diseases including Perthes and SCFE have been associated with an increased incidence of FAI. Most patients, however, do not show growth plate abnormalities. Acetabular retroversion, coxa profunda, protrusio acetaubli, and post-traumatic deformity are also known factors that can lead to impingement. Increased stresses across a developing physis of the femoral head during particular sports or hypermobility of the hip joint have been proposed as possible etiology for abnormalities of the proximal femur. For example, hockey players may be at higher risk of developing CAM type impingement due to the forceful lateral push off while skating.

    References
    Michael Leunig MD, Paul E. Beaule´ MD, Reinhold Ganz MD: The Concept of Femoroacetabular Impingement, Current Status and Future Perspectives. Clin Orthop Relat Res (2009) 467:616–622.

    Jack G. Skendzel, MD, Alexander E. Weber, MD, James R. Ross, MD, Christopher M. Larson, MD, Michael Leunig, MD, Bryan T. Kelly, MD, and Asheesh Bedi, MD: The Approach to the Evaluation and Surgical Treatment of Mechanical Hip Pain in the Young Patient. The Journal of Bone and Joint Surgery (2013) 95A.

    Justin D. Stull, Marc J. Philippon and Robert F. LaPrade ”At-Risk” Positioning and Hip Biomechanics of the Peewee Ice Hockey Sprint Start. Am J Sports Med 2011 39: 29S.

  5. Question 5 of 50
    5. Question

    Which choice describes the location of pincer lesion in femoroacetabular impingement?

    Correct

    The two types of FAI are pincer and CAM. Pincer impingement is the result of mechanical impingement on the acetabular side of the hip joint, typically in the anterior-superior location.  A prominence or overhang of acetabular bone causes mechanical impingement between the acetabulum and femoral head – neck junction.  As mentioned, acetabular retroversion, coxa profunda, protrusio, can contribute to pincer type impingement. The continued contact will often lead to labral tears or failure of the labrum.  Calcific deposits can occur within the labrum, which are often visible on x-ray.  Chondral defects can follow, as well as ossification of the acetabular rim known as os acetabuli.  Fibrocystic changes or synovial herniation pits can be seen at the femoral head-neck junction both on x-ray and MRI.  Active women aged 30-40 have the highest incidence of pincer type impingement.

    CAM type impingement is an abnormality or prominence of the femoral head, typically in the anterior-superior orientation, causing decreased clearance for full range of motion, particularly high flexion and internal rotation.  The subsequent contact between the femoral head and acetabular rim causes a shearing of the adjacent chondral surface and labrum.   CAM type impingement has a higher incidence in young athletic males aged 20-30.  Some patients have CAM and pincer characteristics on imaging, resulting in a combined impingment.

    Several other types of extra-articular hip impingement have been described including ischiofemoral impingement, psoas impingement, subspine impingement, greater trochanteric/pelvic impingement.6  These are less common than the CAM and pincer types of femoroacetabular impingement, and therefore will not be discussed in detail in this paper.

    References
    Asheesh Bedi, MD, and Bryan T. Kelly, MD: Femoroacetabular Impingement.  The Journal of Bone and Joint Surgery (2013).

    Darren de SA, M.D., Hussain Alradwan, M.B.B.S., F.R.C.S.(C), Stephanie Cargnelli, M.D. Cand., Zoyah Thawer, M.D. Cand., Nicole Simunovic, M.Sc., Edwin Cadet, M.D., Nicolas Bonin, M.D., Christopher Larson, M.D., and Olufemi R. Ayeni, M.D., M.Sc., F.R.C.S.(C).  Extra-Articular Hip Impingement: A Systematic Review Examining Operative Treatment of Psoas, Subspine, Ischiofemoral, and Greater Trochanteric/Pelvic Impingement: Arthroscopy (2014)

    Incorrect

    The two types of FAI are pincer and CAM. Pincer impingement is the result of mechanical impingement on the acetabular side of the hip joint, typically in the anterior-superior location.  A prominence or overhang of acetabular bone causes mechanical impingement between the acetabulum and femoral head – neck junction.  As mentioned, acetabular retroversion, coxa profunda, protrusio, can contribute to pincer type impingement. The continued contact will often lead to labral tears or failure of the labrum.  Calcific deposits can occur within the labrum, which are often visible on x-ray.  Chondral defects can follow, as well as ossification of the acetabular rim known as os acetabuli.  Fibrocystic changes or synovial herniation pits can be seen at the femoral head-neck junction both on x-ray and MRI.  Active women aged 30-40 have the highest incidence of pincer type impingement.

    CAM type impingement is an abnormality or prominence of the femoral head, typically in the anterior-superior orientation, causing decreased clearance for full range of motion, particularly high flexion and internal rotation.  The subsequent contact between the femoral head and acetabular rim causes a shearing of the adjacent chondral surface and labrum.   CAM type impingement has a higher incidence in young athletic males aged 20-30.  Some patients have CAM and pincer characteristics on imaging, resulting in a combined impingment.

    Several other types of extra-articular hip impingement have been described including ischiofemoral impingement, psoas impingement, subspine impingement, greater trochanteric/pelvic impingement.6  These are less common than the CAM and pincer types of femoroacetabular impingement, and therefore will not be discussed in detail in this paper.

    References
    Asheesh Bedi, MD, and Bryan T. Kelly, MD: Femoroacetabular Impingement.  The Journal of Bone and Joint Surgery (2013).

    Darren de SA, M.D., Hussain Alradwan, M.B.B.S., F.R.C.S.(C), Stephanie Cargnelli, M.D. Cand., Zoyah Thawer, M.D. Cand., Nicole Simunovic, M.Sc., Edwin Cadet, M.D., Nicolas Bonin, M.D., Christopher Larson, M.D., and Olufemi R. Ayeni, M.D., M.Sc., F.R.C.S.(C).  Extra-Articular Hip Impingement: A Systematic Review Examining Operative Treatment of Psoas, Subspine, Ischiofemoral, and Greater Trochanteric/Pelvic Impingement: Arthroscopy (2014).

  6. Question 6 of 50
    6. Question

    Which choice below describes the most common pathological position of femoroacetabular impingement (FAI)?

    Correct

    Irritability of the hip joint can be present with log rolling, however pain is usually elicited with passive hyperflexion, adduction, and internal rotation (FADIR).  This manuever places the hip in the most common pathologic position of FAI.   FABER (Flexion, ABduction, External Rotation), also called the Patrick test will often elicit pain as well.  Posterior impingement can be tested by extending the leg off the exam table, allowing the hip to fully extend, then abducted and externally rotated.  Range of motion should be assessed, looking especially for a decrease in internal rotation.  Patients will often guard against internal rotation with pelvic tilt. Pain with resisted hip flexion with the knee extended should raise concern for psoas tendinitis/ bursitis. Valsalva should not reproduce symptoms, and pain with resisted sit-up is more often associated with sports hernia.  Palpation should be performed at the sites of pain identified by the patient.  Particular areas of interest include abdomen, greater trochanter, psoas tendon, ASIS, SI joint, ischial tuberosity, and piriformis tendon.

    References
    Marc J. Philippon Æ R. Brian Maxwell, Todd L. Johnston Æ Mara Schenker Æ Karen K. Briggs Clinical presentation of femoroacetabular impingement, Knee Surg Sports Traumatol Arthrosc (2007) 15:1041–1047.

    Hip labral tear. http://www.orthobullets.com. 12/7/14.

    Incorrect

    Irritability of the hip joint can be present with log rolling, however pain is usually elicited with passive hyperflexion, adduction, and internal rotation (FADIR).  This manuever places the hip in the most common pathologic position of FAI.   FABER (Flexion, ABduction, External Rotation), also called the Patrick test will often elicit pain as well.  Posterior impingement can be tested by extending the leg off the exam table, allowing the hip to fully extend, then abducted and externally rotated.  Range of motion should be assessed, looking especially for a decrease in internal rotation.  Patients will often guard against internal rotation with pelvic tilt. Pain with resisted hip flexion with the knee extended should raise concern for psoas tendinitis/ bursitis. Valsalva should not reproduce symptoms, and pain with resisted sit-up is more often associated with sports hernia.  Palpation should be performed at the sites of pain identified by the patient.  Particular areas of interest include abdomen, greater trochanter, psoas tendon, ASIS, SI joint, ischial tuberosity, and piriformis tendon.

    References
    Marc J. Philippon Æ R. Brian Maxwell, Todd L. Johnston Æ Mara Schenker Æ Karen K. Briggs Clinical presentation of femoroacetabular impingement, Knee Surg Sports Traumatol Arthrosc (2007) 15:1041–1047.

    Hip labral tear. http://www.orthobullets.com. 12/7/14.

  7. Question 7 of 50
    7. Question

    A 33-year old female presents to the office with activity related hip pain for 6 months. She is an avid runner and coaches field hockey at a high school level.  She experiences sharp pain with hip flexion and internal rotation. X-rays taken in the office reveal a small cam lesion. An MRI arthrogram is performed showing a tear of the anterior superior labrum.  The patient would like to know about her current treatment options. Which statement is true regarding the treatment of FAI and labral tears of the hip?

    Correct

    Non-operative treatment including a trial of activity modification, physical therapy, and NSAIDS are the first line treatment for FAI and labral tears of the hip.  Since young and active patients are often affected, any prolonged activity modification may not be acceptable.  Physical therapy can be beneficial in correcting overall mechanics, however range of motion exercises can make symptoms worse or aggravate FAI.  As previously described, continued mechanical impingement can lead to chondral wear and arthritis. Therefore, patients are instructed not to push through pain and avoid positions during activities that cause discomfort. Intra-articular cortico-steriods can be used, however the age of patient and associated risk factors should be taken into consideration.  Intra-articular injections with local anesthetic can be used for diagnostic value, and serve to confirm the hip as the pain generator, since the differential diagnosis, as previously mentioned, can be extensive.

    In the prescence of FAI with an associate labral tear, bony impingement must be addressed to prevent further damage to the labrum. Repair of isolated labral tears without bony impingement lesions is also recommended if conservative treatment has failed. Isolated labral tears require a detailed history and physical ensuring other conditions in the differential are excluded. Clinical correlation is important with hip MRI, as some studies have reported high incidence (up to73%) of asymptomatic labral tears.  In other words, a labral tear on MRI may not be the source of the patient’s pain.

    Operative Treatment
    Both open and arthroscopic treatments are available for FAI, each with specific indications.  The open procedure involves a surgical dislocation with trochanteric osteotomy, making this a significant operation.  The advantage is that it does allow for 360 degree visualization of the femoral head. Large posterior or posterolateral CAM deformities, extra-articular impingement (ischiofemoral impingement or trochanteric impingement), and morphology that is associated with hip instability including coxa valga are often treated by open approach.  Acetabular sided morphology including dysplasia, severe acetabular retroversion, coxa profunda, and protrusio are also sometimes best addressed through an open surgical approach.

    Arthroscopy is a minimally invasive approach, allowing for quicker rehab. It is typically done on an outpatient basis.  Technique can be challenging and has an associated learning curve, and does require specialized equipment.  This paper will focus on arthroscopic treatment, as this is growing in popularity.

    Hip Arthroscopy
    Hip Arthroscopy is one of the fastest growing fields in orthopedics.  This minimally invasive technique can be effective for treating intra-articular pathology of the hip including loose body removal, labral debridement/repair, and correction of FAI bony changes, and is usually favored to open surgical dislocation.  Hip arthroscopy has been shown to have higher functional outcome scores than open procedures with lower rates of complications.

    Surgical Technique
    The patient is placed supine on the operating table.  Using a hip distractor system and well-padded perineal post, approximately 10-12 mm of distraction is achieved.  Fluoroscopy is used to evaluate distraction, as well as to guide spinal needles to obtain intra-articular access. A series of dilators are used to create arthroscopic portals, taking care to avoid injury to neurovascular structures.  The most commonly described portal sites are anterior, anterolateral and distal lateral accessory portal.  A 70 degree arthroscope allows for visualization of intra-articular structures. Labral debridement may be performed if tissue is not viable, however long term results appear to be better with labral repair. Labral repair is performed using anchors and suture .  In cases of significant labral deficiency, labral reconstruction may be performed to restore labral function.  Pincer lesions can be corrected, again using fluoroscopy to evaluate bony resection .  Care is taken to keep traction time under 2 hours to avoid neurovascular injury. CAM deformities are addressed via the peripheral compartment, which can be accessed off traction, resecting bone to create a more spherical femoral head.

    Rehabilitation Protocol
    The post-operative course following hip arthroscopy will vary by the procedure performed and surgeon preferences.  Rehab for isolated labral debridement typically includes immediate weight bearing as tolerated and a gradual return to activities to tolerance. FAI and labral repair patients are typically on crutches, allowing for partial weight bearing for 4-6 weeks. Flexion with internal rotation should be approached cautiously.  Some surgeons elect to use post operative bracing systems, however patients will often self regulate motion.  Range of motion is progressed through weeks 2-4, along with early strengthening of quad, hamstring and gluteal muscle groups.  Lumbopelvic muscle dysfunction is often present in these patients, and should be addressed.  Functional progression occurs as weight bearing, strength, and stability allow.  Elliptical machines and jogging are often incorporated at 12-16 weeks.  Return to play for athletes varies from 12-32 weeks.  Patient education is very important when discussing returning to high level sports and repetitive impact activities, such as distance running.  The risk of exacerbating underlying chondral pathology should be taken into consideration, with emphasis on long term chondral health.

    Patients should be advised of the risks and potential complications of hip arthroscopy.  Major risks that are unique to hip arthroscopy include; femoral neck fracture during application of traction, sciatic nerve injury/footdrop, pudendal nerve palsy, compartment syndrome of the leg, compartment syndrome of the abdomen from arthroscopic fluid extravasation, subsequent hip instability, AVN of the femoral head and septic arthritis.  A recent review of 6962 hip arthroscopies reported a 0.58% major complication rate, and minor complication rate of 7.5%.  Minor complications include:  re-tear of the labrum, persistent pain from degenerative arthritis, transient numbness in foot from traction boots, lateral femoral cutaneous nerve irritation, heterotopic ossification, and iatrogenic chondral injury.

    Positive clinical outcomes have been reported following hip arthroscopy, ranging from 67-93% good to excellent results at 26 months. Return to play rates for athletes have been reported at 92%.  As mentioned, results of labral repair tend to be better than debridement.  Outcomes are less favorable when other underlying conditions are present including degenerative osteoarthritis, acetabular dysplasia, large posterior CAM lesions, extra-articular impingement, significant coxa valga, coxa profunda and protrusio.While these are not necessarily contraindications to arthroscopy, care should be taken to thoroughly evaluate these conditions and adjust treatment accordingly.

    References
    Philippon, MJ Prevalence of Abnormal Hip Findings in Asymptomatic Participants American Journal of Sports Medicine 2012 Dec;40(12):2720-4.

    Beaule Three-dimensional computed tomography of the hip in the assessment of femoroacetabular impingement Journal of Orthopedic Research Nov 2005 23(6) 1286-92
    Parvizi, Leunig, Ganz Femoroacetabular Impingement Journal American Academy of Orthopedic Surgeons 2007 (15) 561-70.

    Pre-operative intra-articular hip injection as a predictor of short term outcome following arthroscopic management of femoroacetabular impingement  Knee surgery, sports traumatology, arthroscopy April 2014 22(4) 801-5.

    Boster IB, Smith TJ, Nasser R, Domb B  Open Surgical Dislocation Versus Arthroscopy for Femoroacetabular Impingment: A Comparison of Clinical Outcomes American Journal or Orthopedics May 2014 43(5) 209-14.

    Zaltz I, Kelly B, Larson C, Leunig M, Bedi A  Surgical Treatments of Femoroacetabular Impingement: What are the Limits of Hip Arthroscopy? Arthroscopy Jan 2014 30(1) 99-110.

    Boster IB, Jackson TJ, Smith TW, Leonard JP, Stake CE, Domb BG Open Surgical Dislocation versus arthroscopic Treatment of FAI American Journal of Orthopedics 2014 May 43(5) 209-14.

    The Learning Curve for Hip Arthroscopy: A Systematic Review Hoppe DJ, Larson CM  Arthroscopy 2014 March 30(3) 389-97.

    Trends in Hip Arthroscopy Colvin AC, Harrast J, Harner C Journal of Bone and Joint Surgery 2012 Feb 15, 94(4).

    Arthroscopic Labral Repair in the Hip: Surgical Technique and Review of the Literature Kelly BT, Weiland DE, Schenker ML, Philippon MJ Arthrosocpy Dec 2005 21(12) 1496-504.

    Arthroscopic debridement versus refixation of the acetabular labrum associated with femoroacetabular impingement: mean 3.5-year follow-up  Larson CM, Giveans MR, Stone RM American Journal of Sports Medicine 2012 May 40 (5) 1015-21.

    Arthroscopic Management of Femoroacetabular Impingement Osteoplasty Technique and Literature Review Philippon, MJ, Stubbs AJ, Schenker Ml, Maxwell BR, Ganz R, Leunig M  The American Journal of Sports Medicine 2007 Sept 35(9) 1571-80.

    Rehabilitation After Arthroscopic Decompression for Femoroacetabular Impingement Enseki KR, Martin R, Kelly BT. Clin Sports Med. 2010 Apr;29(2):247-55.

    Abdominal Compartment Syndrome After Hip Arthroscopy Justin Fowler, M.D., and Brett D. Owens, M.D. Arthroscopy 2010 Jan 26 (1) 128-30.

    Osteonecrosis of the Femoral Head after Hip Arthroscopy Danielle L. Scher MD, Philip J. Belmont Jr MD, Brett D. Owens MD Clinical Orthopedics and Related Research 2010 Nov 468(11) 3121-5.

    Hip Subluxation as a Complication of Arthroscopic Debridement Benali Y, Katthagen BD Arthroscopy 2009 Apr 25(4) 405-7.

    A review of femoroacetabular impingement and hip arthroscopy in the athlete Tranovich MJ, Salzler MJ, Enseki KR, Wright VJ The Physician and sports medicine 201 Feb 42(1) 75-87.

    Femoroacetabular impingement. Bedi A1, Kelly BT. Journal of Bone and Joint Surgery Jan 2 95(1) 82-92.

    Return to Preinjury Activity Levels After Surgical Management of Femoroacetabular Impingement in Athletes.  Alradwan H, Philippon MJ, Farrokhyar F, Chu R  Arthrosocpy 2012 Oct 28(10) 1567-76.

     

    Incorrect

    Non-operative treatment including a trial of activity modification, physical therapy, and NSAIDS are the first line treatment for FAI and labral tears of the hip.  Since young and active patients are often affected, any prolonged activity modification may not be acceptable.  Physical therapy can be beneficial in correcting overall mechanics, however range of motion exercises can make symptoms worse or aggravate FAI.  As previously described, continued mechanical impingement can lead to chondral wear and arthritis. Therefore, patients are instructed not to push through pain and avoid positions during activities that cause discomfort. Intra-articular cortico-steriods can be used, however the age of patient and associated risk factors should be taken into consideration.  Intra-articular injections with local anesthetic can be used for diagnostic value, and serve to confirm the hip as the pain generator, since the differential diagnosis, as previously mentioned, can be extensive.

    In the prescence of FAI with an associate labral tear, bony impingement must be addressed to prevent further damage to the labrum. Repair of isolated labral tears without bony impingement lesions is also recommended if conservative treatment has failed. Isolated labral tears require a detailed history and physical ensuring other conditions in the differential are excluded. Clinical correlation is important with hip MRI, as some studies have reported high incidence (up to73%) of asymptomatic labral tears.  In other words, a labral tear on MRI may not be the source of the patient’s pain.

    Operative Treatment
    Both open and arthroscopic treatments are available for FAI, each with specific indications.  The open procedure involves a surgical dislocation with trochanteric osteotomy, making this a significant operation.  The advantage is that it does allow for 360 degree visualization of the femoral head. Large posterior or posterolateral CAM deformities, extra-articular impingement (ischiofemoral impingement or trochanteric impingement), and morphology that is associated with hip instability including coxa valga are often treated by open approach.  Acetabular sided morphology including dysplasia, severe acetabular retroversion, coxa profunda, and protrusio are also sometimes best addressed through an open surgical approach.

    Arthroscopy is a minimally invasive approach, allowing for quicker rehab. It is typically done on an outpatient basis.  Technique can be challenging and has an associated learning curve, and does require specialized equipment.  This paper will focus on arthroscopic treatment, as this is growing in popularity.

    Hip Arthroscopy
    Hip Arthroscopy is one of the fastest growing fields in orthopedics.  This minimally invasive technique can be effective for treating intra-articular pathology of the hip including loose body removal, labral debridement/repair, and correction of FAI bony changes, and is usually favored to open surgical dislocation.  Hip arthroscopy has been shown to have higher functional outcome scores than open procedures with lower rates of complications.

    Surgical Technique
    The patient is placed supine on the operating table.  Using a hip distractor system and well-padded perineal post, approximately 10-12 mm of distraction is achieved.  Fluoroscopy is used to evaluate distraction, as well as to guide spinal needles to obtain intra-articular access. A series of dilators are used to create arthroscopic portals, taking care to avoid injury to neurovascular structures.  The most commonly described portal sites are anterior, anterolateral and distal lateral accessory portal.  A 70 degree arthroscope allows for visualization of intra-articular structures. Labral debridement may be performed if tissue is not viable, however long term results appear to be better with labral repair. Labral repair is performed using anchors and suture .  In cases of significant labral deficiency, labral reconstruction may be performed to restore labral function.  Pincer lesions can be corrected, again using fluoroscopy to evaluate bony resection .  Care is taken to keep traction time under 2 hours to avoid neurovascular injury. CAM deformities are addressed via the peripheral compartment, which can be accessed off traction, resecting bone to create a more spherical femoral head.

    Rehabilitation Protocol
    The post-operative course following hip arthroscopy will vary by the procedure performed and surgeon preferences.  Rehab for isolated labral debridement typically includes immediate weight bearing as tolerated and a gradual return to activities to tolerance. FAI and labral repair patients are typically on crutches, allowing for partial weight bearing for 4-6 weeks. Flexion with internal rotation should be approached cautiously.  Some surgeons elect to use post operative bracing systems, however patients will often self regulate motion.  Range of motion is progressed through weeks 2-4, along with early strengthening of quad, hamstring and gluteal muscle groups.  Lumbopelvic muscle dysfunction is often present in these patients, and should be addressed.  Functional progression occurs as weight bearing, strength, and stability allow.  Elliptical machines and jogging are often incorporated at 12-16 weeks.  Return to play for athletes varies from 12-32 weeks.  Patient education is very important when discussing returning to high level sports and repetitive impact activities, such as distance running.  The risk of exacerbating underlying chondral pathology should be taken into consideration, with emphasis on long term chondral health.

    Patients should be advised of the risks and potential complications of hip arthroscopy.  Major risks that are unique to hip arthroscopy include; femoral neck fracture during application of traction, sciatic nerve injury/footdrop, pudendal nerve palsy, compartment syndrome of the leg, compartment syndrome of the abdomen from arthroscopic fluid extravasation, subsequent hip instability, AVN of the femoral head and septic arthritis.  A recent review of 6962 hip arthroscopies reported a 0.58% major complication rate, and minor complication rate of 7.5%.  Minor complications include:  re-tear of the labrum, persistent pain from degenerative arthritis, transient numbness in foot from traction boots, lateral femoral cutaneous nerve irritation, heterotopic ossification, and iatrogenic chondral injury.

    Positive clinical outcomes have been reported following hip arthroscopy, ranging from 67-93% good to excellent results at 26 months. Return to play rates for athletes have been reported at 92%.  As mentioned, results of labral repair tend to be better than debridement.  Outcomes are less favorable when other underlying conditions are present including degenerative osteoarthritis, acetabular dysplasia, large posterior CAM lesions, extra-articular impingement, significant coxa valga, coxa profunda and protrusio.While these are not necessarily contraindications to arthroscopy, care should be taken to thoroughly evaluate these conditions and adjust treatment accordingly.

    References
    Philippon, MJ Prevalence of Abnormal Hip Findings in Asymptomatic Participants American Journal of Sports Medicine 2012 Dec;40(12):2720-4.

    Beaule Three-dimensional computed tomography of the hip in the assessment of femoroacetabular impingement Journal of Orthopedic Research Nov 2005 23(6) 1286-92
    Parvizi, Leunig, Ganz Femoroacetabular Impingement Journal American Academy of Orthopedic Surgeons 2007 (15) 561-70.

    Pre-operative intra-articular hip injection as a predictor of short term outcome following arthroscopic management of femoroacetabular impingement  Knee surgery, sports traumatology, arthroscopy April 2014 22(4) 801-5.

    Boster IB, Smith TJ, Nasser R, Domb B  Open Surgical Dislocation Versus Arthroscopy for Femoroacetabular Impingment: A Comparison of Clinical Outcomes American Journal or Orthopedics May 2014 43(5) 209-14.

    Zaltz I, Kelly B, Larson C, Leunig M, Bedi A  Surgical Treatments of Femoroacetabular Impingement: What are the Limits of Hip Arthroscopy? Arthroscopy Jan 2014 30(1) 99-110.

    Boster IB, Jackson TJ, Smith TW, Leonard JP, Stake CE, Domb BG Open Surgical Dislocation versus arthroscopic Treatment of FAI American Journal of Orthopedics 2014 May 43(5) 209-14.

    The Learning Curve for Hip Arthroscopy: A Systematic Review Hoppe DJ, Larson CM  Arthroscopy 2014 March 30(3) 389-97.

    Trends in Hip Arthroscopy Colvin AC, Harrast J, Harner C Journal of Bone and Joint Surgery 2012 Feb 15, 94(4).

    Arthroscopic Labral Repair in the Hip: Surgical Technique and Review of the Literature Kelly BT, Weiland DE, Schenker ML, Philippon MJ Arthrosocpy Dec 2005 21(12) 1496-504.

    Arthroscopic debridement versus refixation of the acetabular labrum associated with femoroacetabular impingement: mean 3.5-year follow-up  Larson CM, Giveans MR, Stone RM American Journal of Sports Medicine 2012 May 40 (5) 1015-21.

    Arthroscopic Management of Femoroacetabular Impingement Osteoplasty Technique and Literature Review Philippon, MJ, Stubbs AJ, Schenker Ml, Maxwell BR, Ganz R, Leunig M  The American Journal of Sports Medicine 2007 Sept 35(9) 1571-80.

    Rehabilitation After Arthroscopic Decompression for Femoroacetabular Impingement Enseki KR, Martin R, Kelly BT. Clin Sports Med. 2010 Apr;29(2):247-55.

    Abdominal Compartment Syndrome After Hip Arthroscopy Justin Fowler, M.D., and Brett D. Owens, M.D. Arthroscopy 2010 Jan 26 (1) 128-30.

    Osteonecrosis of the Femoral Head after Hip Arthroscopy Danielle L. Scher MD, Philip J. Belmont Jr MD, Brett D. Owens MD Clinical Orthopedics and Related Research 2010 Nov 468(11) 3121-5.

    Hip Subluxation as a Complication of Arthroscopic Debridement Benali Y, Katthagen BD Arthroscopy 2009 Apr 25(4) 405-7.

    A review of femoroacetabular impingement and hip arthroscopy in the athlete Tranovich MJ, Salzler MJ, Enseki KR, Wright VJ The Physician and sports medicine 201 Feb 42(1) 75-87.

    Femoroacetabular impingement. Bedi A1, Kelly BT. Journal of Bone and Joint Surgery Jan 2 95(1) 82-92.

    Return to Preinjury Activity Levels After Surgical Management of Femoroacetabular Impingement in Athletes.  Alradwan H, Philippon MJ, Farrokhyar F, Chu R  Arthrosocpy 2012 Oct 28(10) 1567-76.

  8. Question 8 of 50
    8. Question

    Injury or entrapment of which peripheral nerve would cause winging of the scapula?

    Correct

    The long thoracic nerve evolves from cervical spine nerve roots C5-C7 and supplies the serratous anterior muscle. The serratous anterior muscle functions to pull the scapula forward and rotate the arm upward. The muscle is important with forward arm motion like boxing and overhead activities like swimming and tennis. Injury to the long thoracic nerve may result from trauma and overhead activities that cause over stretching or repetitive movements of the arm.  Patients typically present with shoulder or neck pain that worsens with overhead activities. Physical exam findings include weakness with forward flexion of the arm and winging of the scapula due to a weak serratous anterior muscle.

    Spinal accessory nerve injury supplies the sternocleidomastoid and trapezius muscles. Injury may result from traumatic or iatrogenic causes; iatrogenic injury may be a postoperative complication from radical neck dissection, carotid endarterectomy, and cervical node biopsy. Exam may reveal weak shoulder elevation and should asymmetry.

    The suprascapular nerve supplies supraspinatus and infraspinatus muscles. Injury may result from repetitive overhead loading and cyst formation in the suprascapular notch that compresses the nerve. Physical exam findings may include weakness with weak arm elevation and/or external rotation.

    Injury to the axillary nerve is commonly caused by a glenohumeral dislocation or proximal humerus fracture.  An anterior dislocation causes traction of the axillary nerve in the posterior aspect of the shoulder. Tension due to stretching may result in injury to the nerve. Physical exam findings after an axillary nerve injury include weak lateral abduction and paraesthesias in the deltoid distribution.

    Reference
    Neal SA, Fields KB. Peripheral Nerve Entrapment and Injury in the Upper Extremity. Am Fam Physician. 2010 Jan 15;81(2):147-155.

    Incorrect

    The long thoracic nerve evolves from cervical spine nerve roots C5-C7 and supplies the serratous anterior muscle. The serratous anterior muscle functions to pull the scapula forward and rotate the arm upward. The muscle is important with forward arm motion like boxing and overhead activities like swimming and tennis. Injury to the long thoracic nerve may result from trauma and overhead activities that cause over stretching or repetitive movements of the arm.  Patients typically present with shoulder or neck pain that worsens with overhead activities. Physical exam findings include weakness with forward flexion of the arm and winging of the scapula due to a weak serratous anterior muscle.

    Spinal accessory nerve injury supplies the sternocleidomastoid and trapezius muscles. Injury may result from traumatic or iatrogenic causes; iatrogenic injury may be a postoperative complication from radical neck dissection, carotid endarterectomy, and cervical node biopsy. Exam may reveal weak shoulder elevation and should asymmetry.

    The suprascapular nerve supplies supraspinatus and infraspinatus muscles. Injury may result from repetitive overhead loading and cyst formation in the suprascapular notch that compresses the nerve. Physical exam findings may include weakness with weak arm elevation and/or external rotation.

    Injury to the axillary nerve is commonly caused by a glenohumeral dislocation or proximal humerus fracture.  An anterior dislocation causes traction of the axillary nerve in the posterior aspect of the shoulder. Tension due to stretching may result in injury to the nerve. Physical exam findings after an axillary nerve injury include weak lateral abduction and paraesthesias in the deltoid distribution.

    Reference
    Neal SA, Fields KB. Peripheral Nerve Entrapment and Injury in the Upper Extremity. Am Fam Physician. 2010 Jan 15;81(2):147-155.

  9. Question 9 of 50
    9. Question

    Which physical exam test is used to reproduce the symptoms associated with sacroiliac (SI) joint?

    Correct

    FABER’s or Patrick test is used to assess sacroiliac and hip joint pathology. The patient’s hip is placed in flexion, abduction, and external rotation in a supine position. A posterior directed force is then placed on the knee and transferred through the femur to stress the sacroiliac joint.

    Ober’s test is used to determine tightness of the iliotibial band. During the exam the patient test lies in the lateral position with the affected hip up. The affected hip is slowly taken into extension and adduction. Inability of the leg to adduct or drop to the contralateral leg is a positive finding indicating a tight iliotibial band and tensor fascia latae.

    Ely’s test is used for assessing flexibility of the rectus femoris muscle. During the exam test the patient lies prone while the patient’s knee is passively flexed. A tight rectus femoris is noted if the hip flexes when the knee is flexed.

    Thomas test is used for assessing the iliopsoas muscle group and rectus femoris muscle for tightness. The patient lies in a supine position and maximally flexes both knees against the chest using both arms with the buttocks near the table edge. The patient then lowers the test leg until either pelvic tilting or an increased lumbar lordosis occurs. Limited hip extension with the knee in greater than 45 degrees of flexion indicates iliopsoas muscle tightness.

    Reference
    Konin JG, Wilksten DL, Isear JA, Brader H. Special Tests for Orthopedic Examination.  Third Edition. SLACK Incorporated. Thorofare, NJ. 2006.

    Incorrect

    FABER’s or Patrick test is used to assess sacroiliac and hip joint pathology. The patient’s hip is placed in flexion, abduction, and external rotation in a supine position. A posterior directed force is then placed on the knee and transferred through the femur to stress the sacroiliac joint.

    Ober’s test is used to determine tightness of the iliotibial band. During the exam the patient test lies in the lateral position with the affected hip up. The affected hip is slowly taken into extension and adduction. Inability of the leg to adduct or drop to the contralateral leg is a positive finding indicating a tight iliotibial band and tensor fascia latae.

    Ely’s test is used for assessing flexibility of the rectus femoris muscle. During the exam test the patient lies prone while the patient’s knee is passively flexed. A tight rectus femoris is noted if the hip flexes when the knee is flexed.

    Thomas test is used for assessing the iliopsoas muscle group and rectus femoris muscle for tightness. The patient lies in a supine position and maximally flexes both knees against the chest using both arms with the buttocks near the table edge. The patient then lowers the test leg until either pelvic tilting or an increased lumbar lordosis occurs. Limited hip extension with the knee in greater than 45 degrees of flexion indicates iliopsoas muscle tightness.

    Reference
    Konin JG, Wilksten DL, Isear JA, Brader H. Special Tests for Orthopedic Examination.  Third Edition. SLACK Incorporated. Thorofare, NJ. 2006.

  10. Question 10 of 50
    10. Question

    finger

    A 16-year old male presents to your office with pain and deformity of his right middle finger.  He injured the finger playing football 2 days ago while making a tackle. On exam you notice a flexion deformity at the distal interphalangeal joint (DIP) and he is unable to extend his distal phalanx. AP and lateral x-rays are shown above. Which choice is the best treatment option for this patient?

    Correct

    The above x-rays show a bony mallet finger. Bony mallet fractures result from a forceful flexion to an extended distal phalanx causing the extensor tendon to pull off a piece of bone.  Patients with a mallet finger may also present with a swan neck deformity as disruption of the terminal extensor tendon causes increased extensor tone at the DIP.

    Several types of splints and surgical techniques have been utilized to treat bony mallet fractures. The majority of injuries are treated with continuous splinting of the DIP alone for 6-8 weeks.  DIP splinting alone is also advised with an associated swan neck deformity as combined DIP and PIP splinting has not been shown to improve a swan neck deformity.  At no point should the DIP be flexed during the splinting period. If flexion occurs during the 6-8 weeks of splinting then the immobilization period is restarted. Dorsal and volar splints are used with no definitive benefit of one over the other.  Splinting in full extension or slight hyperextension of the DIP is recommended. However, overextension of the DIP should be avoided as this may result in dorsal skin compromise.

    Bony mallet fractures do well with nonsurgical management when patients can be compliant with strict immobilization. This patient was treated in a volar splint at all times for 6 weeks. Although the fracture fragment is displaced in the above x-rays, studies have shown that the articular surface of the distal phalanx has significant remodeling potential.

    Relative surgical indications include inability to tolerate splinting, volar subluxation of the distal phalanx, greater than 50% articular surface involvement, and open injury. Surgical techniques involve placing a k-wire to immobilize the DIP joint and may include reduction of the fracture fragment with a transarticular pin through the DIP.

    References
    Mallet Finger. http://www.orthobullets.com. Accessed on 11/1/14.

    Bendre AA, Hartigan BJ, Kalainov DM. Mallet Finger. JAAOS 2005; 13: 336-344.

    Incorrect

    The above x-rays show a bony mallet finger. Bony mallet fractures result from a forceful flexion to an extended distal phalanx causing the extensor tendon to pull off a piece of bone.  Patients with a mallet finger may also present with a swan neck deformity as disruption of the terminal extensor tendon causes increased extensor tone at the DIP.

    Several types of splints and surgical techniques have been utilized to treat bony mallet fractures. The majority of injuries are treated with continuous splinting of the DIP alone for 6-8 weeks.  DIP splinting alone is also advised with an associated swan neck deformity as combined DIP and PIP splinting has not been shown to improve a swan neck deformity.  At no point should the DIP be flexed during the splinting period. If flexion occurs during the 6-8 weeks of splinting then the immobilization period is restarted. Dorsal and volar splints are used with no definitive benefit of one over the other.  Splinting in full extension or slight hyperextension of the DIP is recommended. However, overextension of the DIP should be avoided as this may result in dorsal skin compromise.

    Bony mallet fractures do well with nonsurgical management when patients can be compliant with strict immobilization. This patient was treated in a volar splint at all times for 6 weeks. Although the fracture fragment is displaced in the above x-rays, studies have shown that the articular surface of the distal phalanx has significant remodeling potential.

    Relative surgical indications include inability to tolerate splinting, volar subluxation of the distal phalanx, greater than 50% articular surface involvement, and open injury. Surgical techniques involve placing a k-wire to immobilize the DIP joint and may include reduction of the fracture fragment with a transarticular pin through the DIP.

    References
    Mallet Finger. http://www.orthobullets.com. Accessed on 11/1/14.

    Bendre AA, Hartigan BJ, Kalainov DM. Mallet Finger. JAAOS 2005; 13: 336-344.

  11. Question 11 of 50
    11. Question

    Which choice below best describes the pathoanatomy of a sports hernia?

    Correct

    Sports hernia (athletic pubalgia) causes groin pain in athletes and occurs more common in sports that require fast twisting motion of the hips. The condition is almost exclusively found in men. The pain is often unilateral, located deep into the groin, and relieved with rest. Several pathological findings have been reported in patients with sports hernias including tearing of the internal and external oblique muscles as they attach to the pubic bone and tearing of the hip adductor muscles.

    A sports hernia or athletic pubalgia is actually not a hernia as the injury does not create a hole in the abdominal wall.  On exam of a patient with athletic pubalgia no bulge or inguinal hernia is palpated. A true hernia is defined as a protrusion of a periotoneal sac through the abdominal wall. A direct inguinal hernia is a protrusion of the peritoneal sac within the floor of Hasselbach’s triangle which consists of the inguinal ligament, the lateral boarder of the inferior epigastric vessels, and the medial boarder of the lateral rectus sheath. The more common indirect inguinal hernia is a protrusion of the peritoneal sac through the internal ring of the inguinal canal.  Inflammation of the symphasis pubis is called osteitis pubis.

    References
    Blackbourne LH. Surgical Recall 4th Edition. Lippincott Williams & Wilkens 2006. Baltimore, MD.

    Farber AJ, Wilckens JH. Sports Hernia. Diagnosis and Therapeutic Approach. JAAOS 2007; 15; 507-515.

    Incorrect

    Sports hernia (athletic pubalgia) causes groin pain in athletes and occurs more common in sports that require fast twisting motion of the hips. The condition is almost exclusively found in men. The pain is often unilateral, located deep into the groin, and relieved with rest. Several pathological findings have been reported in patients with sports hernias including tearing of the internal and external oblique muscles as they attach to the pubic bone and tearing of the hip adductor muscles.

    A sports hernia or athletic pubalgia is actually not a hernia as the injury does not create a hole in the abdominal wall.  On exam of a patient with athletic pubalgia no bulge or inguinal hernia is palpated. A true hernia is defined as a protrusion of a periotoneal sac through the abdominal wall. A direct inguinal hernia is a protrusion of the peritoneal sac within the floor of Hasselbach’s triangle which consists of the inguinal ligament, the lateral boarder of the inferior epigastric vessels, and the medial boarder of the lateral rectus sheath. The more common indirect inguinal hernia is a protrusion of the peritoneal sac through the internal ring of the inguinal canal.  Inflammation of the symphasis pubis is called osteitis pubis.

    References
    Blackbourne LH. Surgical Recall 4th Edition. Lippincott Williams & Wilkens 2006. Baltimore, MD.

    Farber AJ, Wilckens JH. Sports Hernia. Diagnosis and Therapeutic Approach. JAAOS 2007; 15; 507-515.

  12. Question 12 of 50
    12. Question

    hip aiis

    A 13-year old male presents to the office with right hip pain after being tackled during a football game two days ago. He had difficulty ambulating after the injury and was unable to continue playing. He presents to the office on crutches with continued right hip pain. X-rays taken in the office are shown above. What is the most appropriate treatment option for this patient’s injury?

    Correct

    This patients x-rays show an anterior inferior iliac spine (AIIS) avulsion fracture.  An AIIS avulsion fracture represents an apophyseal injury at the insertion site of the rectus femoris. This injury generally occurs with a sudden forceful extension of the hip causing a pull of the rectus femoris.  Patients often complain of feeling a “pop” during the injury and increased pain with hip flexion. Exam findings may include localized pain and tenderness over the AIIS. Increased pain is elicited with resistive hip flexion and knee extension. An AP pelvis x-ray should be obtained to confirm an avulsion fracture vs. a normal adolescent variant.   Treatment consists of partial weight bearing for 2-3 weeks on crutches then weight bearing as tolerated thereafter. The patient is instructed to avoid active hip flexion and passive hip extension for 2-3 weeks.  Physical therapy is initiated at 3 weeks with a focus on a gradual increase in strength training. Return to sports without restrictions can begin at 8 weeks if the patient is back to full strength.  Open reduction and internal fixation is rarely indicated with AIIS avulsion fractures; surgical indications may include displacement > 2 cm, painful nonunion, and exostosis formation.

    References
    Beaty JH, Kasser JR. Rockwood and Wilkin’s Fractures in Children. 6th Edition. Lippincott Williams and Wilkins. Philadelphia, PA. 2006.

    AIIS Avulsion Fracture. http://www.orthobullets.com/sports/3131/anterior-inferior-iliac-spine-avulsion-aiis. 11/15/2014.

    Incorrect

    This patients x-rays show an anterior inferior iliac spine (AIIS) avulsion fracture.  An AIIS avulsion fracture represents an apophyseal injury at the insertion site of the rectus femoris. This injury generally occurs with a sudden forceful extension of the hip causing a pull of the rectus femoris.  Patients often complain of feeling a “pop” during the injury and increased pain with hip flexion. Exam findings may include localized pain and tenderness over the AIIS. Increased pain is elicited with resistive hip flexion and knee extension. An AP pelvis x-ray should be obtained to confirm an avulsion fracture vs. a normal adolescent variant.   Treatment consists of partial weight bearing for 2-3 weeks on crutches then weight bearing as tolerated thereafter. The patient is instructed to avoid active hip flexion and passive hip extension for 2-3 weeks.  Physical therapy is initiated at 3 weeks with a focus on a gradual increase in strength training. Return to sports without restrictions can begin at 8 weeks if the patient is back to full strength.  Open reduction and internal fixation is rarely indicated with AIIS avulsion fractures; surgical indications may include displacement > 2 cm, painful nonunion, and exostosis formation.

    References
    Beaty JH, Kasser JR. Rockwood and Wilkin’s Fractures in Children. 6th Edition. Lippincott Williams and Wilkins. Philadelphia, PA. 2006.

    AIIS Avulsion Fracture. http://www.orthobullets.com/sports/3131/anterior-inferior-iliac-spine-avulsion-aiis. 11/15/2014.

  13. Question 13 of 50
    13. Question

    Complete disruption of the conoid and trapezoid (CC) ligaments is associated with which type of acromioclavicular joint injuries?

    Correct

    The conoid and the trapezoid ligaments originate from the base of the corocoid and attached to the clavicle.  The CC ligaments provide stability of the AC joint in all directions with the primary function being to prevent superior migration of the clavicle. In a type I AC joint sprain the CC ligaments remain intact and x-rays findings are normal. In a type II AC joint sprain the CC ligaments are strained but remain intact. X-ray findings show incomplete AC joint displacement. In a type III injury the CC ligaments are completely disrupted and x-rays findings will show complete displacement of the AC joint.  Type IV injuries have the same soft tissue injury as type III in addition to the clavicle being displaced posteriorly and entrapped within the trapezius. Remember “type IV is out the back door”. Type V injuries have more extensive soft tissue disruption including complete tearing of the CC ligaments with trapezial and deltoid fascia disruption. Clincially, there is drooping of the shoulder and the clavicle cannot be closed reduced unlike type III injuries.  Remember “high V” as these the clavicle will be significantly displaced. Type IV are rarely seen injuries that have inferior dislocation of the distal clavicle under the subacromial space or subcoracoid behind the conjoin tendon.

    Reference
    Mostif A, Clifford RG, et al. Acromioclavicular Joint Injuries. Orthopedic Knowledge Online Journal 2008 6(4). Accessed April 21, 2008.

    Incorrect

    The conoid and the trapezoid ligaments originate from the base of the corocoid and attached to the clavicle.  The CC ligaments provide stability of the AC joint in all directions with the primary function being to prevent superior migration of the clavicle. In a type I AC joint sprain the CC ligaments remain intact and x-rays findings are normal. In a type II AC joint sprain the CC ligaments are strained but remain intact. X-ray findings show incomplete AC joint displacement. In a type III injury the CC ligaments are completely disrupted and x-rays findings will show complete displacement of the AC joint.  Type IV injuries have the same soft tissue injury as type III in addition to the clavicle being displaced posteriorly and entrapped within the trapezius. Remember “type IV is out the back door”. Type V injuries have more extensive soft tissue disruption including complete tearing of the CC ligaments with trapezial and deltoid fascia disruption. Clincially, there is drooping of the shoulder and the clavicle cannot be closed reduced unlike type III injuries.  Remember “high V” as these the clavicle will be significantly displaced. Type IV are rarely seen injuries that have inferior dislocation of the distal clavicle under the subacromial space or subcoracoid behind the conjoin tendon.

    Reference
    Mostif A, Clifford RG, et al. Acromioclavicular Joint Injuries. Orthopedic Knowledge Online Journal 2008 6(4). Accessed April 21, 2008.

  14. Question 14 of 50
    14. Question

    Which choice below best describes how grade III acromioclavicular are treated?

    Correct

    Treatment of grade III AC joint separations is controversial and may vary between surgeons. Most surgeons agree that conservative treatment for at least 3 months is recommended before surgical intervention should be considered. Conservative treatment involves rest, immobilization, and physical therapy. Patients may return to activities as tolerated when full pain free range of motion is demonstrated. Some surgeons recommend early surgical intervention in high level athletes and heavy laborers hoping to decrease painful instability. However, studies have not proven that surgery has any better results both long and short term. Improvements in pain, strength, and range of motion have been found to be equal for both surgical and nonsurgical treatments. Patients with continued pain and instability after 3 months may be considered for surgery. Athletes and heavy laborers may be the most likely to feel AC joint instability and pain after a period of conservative treatment.

    Reference
    Mostif A, Clifford RG, et al. Acromioclavicular Joint Injuries. Orthopedic Knowledge Online Journal 2008 6(4). Accessed April 21, 2008.

    Incorrect

    Treatment of grade III AC joint separations is controversial and may vary between surgeons. Most surgeons agree that conservative treatment for at least 3 months is recommended before surgical intervention should be considered. Conservative treatment involves rest, immobilization, and physical therapy. Patients may return to activities as tolerated when full pain free range of motion is demonstrated. Some surgeons recommend early surgical intervention in high level athletes and heavy laborers hoping to decrease painful instability. However, studies have not proven that surgery has any better results both long and short term. Improvements in pain, strength, and range of motion have been found to be equal for both surgical and nonsurgical treatments. Patients with continued pain and instability after 3 months may be considered for surgery. Athletes and heavy laborers may be the most likely to feel AC joint instability and pain after a period of conservative treatment.

    Reference
    Mostif A, Clifford RG, et al. Acromioclavicular Joint Injuries. Orthopedic Knowledge Online Journal 2008 6(4). Accessed April 21, 2008.

  15. Question 15 of 50
    15. Question

    You are the covering provider for a high school football game when the athletic trainer asks you to evaluate an athlete who sustained a concussion during the game. The athlete states he didn’t experience any loss of consciousness, nausea, vomiting, vision, or speech changes.  He said he “saw stars” and had a headache for a few minutes after the incident.  He denies any pre or post concussive amnesia. According to the International Conference on Concussion in Sport, when should the athlete be given permission to resume full competition?

    Correct

    Concussion is described as a direct or indirect force to the head that results in a rapid onset of short lived neurological impairment.  Symptoms may resolve over minutes to hours with the majority concussions resolving in 7-10 days.   Symptoms may include headache, amnesia, cognitive impairment, and unsteadiness. Concussion can occur with or without loss consciousness.
    Athletes will sometimes try to withhold, hide, or disguise symptoms so they can continue to play, making a thorough exam critical. Continually checking symptoms until the athlete is in stable condition is vital. Athletes who experience concussion symptoms should not return to play on the same day. Sideline examination should include comprehensive neurological and cognitive function testing. Sideline assessment tools like the Sideline Concussion Assessment Tool (SCAT3) can help standardize and document a complete exam.

    The International Conference on Concussion in Sport developed a six step protocol as a guideline for providers to follow when returning athletes back to play. Each step takes at least 24 hours and the entire protocol takes nearly a week to complete. Athletes must remain asymptomatic through each step to progress. If symptoms do occur, the athlete returns to the previous level. The six step protocol includes: Day 1) no activity, full cognitive recovery, Day 2) light aerobic exercise to increase heart rate, Day 3) add movement with sport specific exercise, Day 4) Non-contact athletic drills, Day 5) full contact practice, and Day 6) if previous levels are asymptomatic the athlete may return to play.

    When taking the athlete’s history, previous and recent concussions must be carefully considered. Second-impact syndrome (SIS) happens when an athlete sustains a second concussion before symptoms of a prior concussion have subsided. SIS is more common in teenagers and is characterized by rapid swelling of the brain that causes sudden and severe loss of neurological function. Nearly 50% of SIS cases lead to death, and 100% result in brain damage. Thus, recognizing concussion symptoms and preventing an injured athlete from returning too early is crucial.

    References
    Durand P, & Adamson GJ. On-the-field management of athletic head injuries.  J Am Acad Orthop Surg. 2004;12(3):191-195.

    McCrory, P., Meeuwisse, W., Johnston, K., Dvorak, J., Aubry, M., Molloy, M., & Cantu, R. (2013). Consensus statement on concussion in sport-the 4th International Conference on Concussion in Sport held in Zurich, November 2012. Br J Sports Med 2013;47:250-258.

    Incorrect

    Concussion is described as a direct or indirect force to the head that results in a rapid onset of short lived neurological impairment.  Symptoms may resolve over minutes to hours with the majority concussions resolving in 7-10 days.   Symptoms may include headache, amnesia, cognitive impairment, and unsteadiness. Concussion can occur with or without loss consciousness.
    Athletes will sometimes try to withhold, hide, or disguise symptoms so they can continue to play, making a thorough exam critical. Continually checking symptoms until the athlete is in stable condition is vital. Athletes who experience concussion symptoms should not return to play on the same day. Sideline examination should include comprehensive neurological and cognitive function testing. Sideline assessment tools like the Sideline Concussion Assessment Tool (SCAT3) can help standardize and document a complete exam.

    The International Conference on Concussion in Sport developed a six step protocol as a guideline for providers to follow when returning athletes back to play. Each step takes at least 24 hours and the entire protocol takes nearly a week to complete. Athletes must remain asymptomatic through each step to progress. If symptoms do occur, the athlete returns to the previous level. The six step protocol includes: Day 1) no activity, full cognitive recovery, Day 2) light aerobic exercise to increase heart rate, Day 3) add movement with sport specific exercise, Day 4) Non-contact athletic drills, Day 5) full contact practice, and Day 6) if previous levels are asymptomatic the athlete may return to play.

    When taking the athlete’s history, previous and recent concussions must be carefully considered. Second-impact syndrome (SIS) happens when an athlete sustains a second concussion before symptoms of a prior concussion have subsided. SIS is more common in teenagers and is characterized by rapid swelling of the brain that causes sudden and severe loss of neurological function. Nearly 50% of SIS cases lead to death, and 100% result in brain damage. Thus, recognizing concussion symptoms and preventing an injured athlete from returning too early is crucial.

    References
    Durand P, & Adamson GJ. On-the-field management of athletic head injuries.  J Am Acad Orthop Surg. 2004;12(3):191-195.

    McCrory, P., Meeuwisse, W., Johnston, K., Dvorak, J., Aubry, M., Molloy, M., & Cantu, R. (2013). Consensus statement on concussion in sport-the 4th International Conference on Concussion in Sport held in Zurich, November 2012. Br J Sports Med 2013;47:250-258.

  16. Question 16 of 50
    16. Question

    rotator

    A 64-year old male presents with a 5-year history of right shoulder pain. The pain started after falling on ice and has been off and on since. He believes an MRI after the injury showed a rotator cuff tear but he did not want surgery at that time.  He presents to the office now with worsening pain and significant weakness with overhead lifting after painting his house 3 months ago. Coronal (figure 1) and sagittal (figure 2) images of a recent MRI ordered by the patients PCP are shown above. MRI findings include complete tearing of the supraspinatus and infraspinatus tendons with approximately 4 cm of retraction. Figure 2 also shows a tangent line from the superior border of the scapula spine to the superior border of the coracoid process. This tangent line is used to determine degree of supraspinatus fatty infiltration using Goutallier classification as the muscle belly does not cross this line. There is approximately 60% fatty infiltration of the supraspinatus tendon. Which Goutallier stage of rotator cuff fatty infiltration does this describe?

    Correct

    Rotator cuff tears are common injuries that require early diagnosis and treatment to optimize outcome.  Full thickness rotator cuff tears will most likely increase over time and become more symptomatic.  When a tendon of the rotator cuff tears the muscle undergoes degenerative changes from disuse. These changes include muscle atrophy, fatty infiltration, and tendon retraction. Early surgical repair reduces the time period of rotator cuff degeneration and optimizes patient outcome.

    Radiographic evaluation of suspected rotator cuff tear should include a true AP of the glenohumeral joint, an axillary view, and a supraspinatus outlet view. The infraspinatus and supraspinatus are active depressors of the humeral head. A decrease in the acromiohumeral interval strongly suggests full-thickness rotator cuff tearing and muscle atrophy.

    MRI provides detailed information on the size, location, and chronicity of a rotator cuff tear. Goutallier classification grading system is used to quantify degree of fatty infiltration seen on sagittal MRI.  Goutallier classification describes the integrity of the rotator cuff muscles and helps predict clinical outcomes after rotator cuff repair surgery. Supraspinatus fatty atrophy is best seen where the corocoid and scapula join to form a Y on a sagittal view (figure 2). Goutallier classification is broken into 5 stages; stage 0 describes normal muscle, stage 1 has streaks of fatty infiltration, stage 2 has more fatty infiltration but still more muscle than fat, stage 3 has equal amounts of fat and muscle, and stage 4 has more fat than muscle.

    Tendon retraction and muscle atrophy are two of several factors that help predict healing and re-tear rates after rotator cuff surgery. One study reported a 93% failure rate in healing when the tendon length was less than 15 mm long with Goutallier stage two or three fatty infiltration. Other factors that influence tendon healing include tear size, tension of the repair, advanced age, tobacco use, and poor health.

    References
    Pill SG, Phillips J, Kissenberth MJ, Hawkins RJ. Decision making in large rotator cuff tears. Inst Course Lect 2012; 61: 97-111.

    “What can be done about rotator cuff failure” http://www.mltj.org/materiale_cic/724_3_4/6265_fatty/article.htm. 1/4/15.

    Incorrect

    Rotator cuff tears are common injuries that require early diagnosis and treatment to optimize outcome.  Full thickness rotator cuff tears will most likely increase over time and become more symptomatic.  When a tendon of the rotator cuff tears the muscle undergoes degenerative changes from disuse. These changes include muscle atrophy, fatty infiltration, and tendon retraction. Early surgical repair reduces the time period of rotator cuff degeneration and optimizes patient outcome.

    Radiographic evaluation of suspected rotator cuff tear should include a true AP of the glenohumeral joint, an axillary view, and a supraspinatus outlet view. The infraspinatus and supraspinatus are active depressors of the humeral head. A decrease in the acromiohumeral interval strongly suggests full-thickness rotator cuff tearing and muscle atrophy.

    MRI provides detailed information on the size, location, and chronicity of a rotator cuff tear. Goutallier classification grading system is used to quantify degree of fatty infiltration seen on sagittal MRI.  Goutallier classification describes the integrity of the rotator cuff muscles and helps predict clinical outcomes after rotator cuff repair surgery. Supraspinatus fatty atrophy is best seen where the corocoid and scapula join to form a Y on a sagittal view (figure 2). Goutallier classification is broken into 5 stages; stage 0 describes normal muscle, stage 1 has streaks of fatty infiltration, stage 2 has more fatty infiltration but still more muscle than fat, stage 3 has equal amounts of fat and muscle, and stage 4 has more fat than muscle.

    Tendon retraction and muscle atrophy are two of several factors that help predict healing and re-tear rates after rotator cuff surgery. One study reported a 93% failure rate in healing when the tendon length was less than 15 mm long with Goutallier stage two or three fatty infiltration. Other factors that influence tendon healing include tear size, tension of the repair, advanced age, tobacco use, and poor health.

    References
    Pill SG, Phillips J, Kissenberth MJ, Hawkins RJ. Decision making in large rotator cuff tears. Inst Course Lect 2012; 61: 97-111.

    “What can be done about rotator cuff failure” http://www.mltj.org/materiale_cic/724_3_4/6265_fatty/article.htm. 1/4/15.

  17. Question 17 of 50
    17. Question

    Question 17

    A 16-year old patient presents to the office with a 2 day history of right knee pain after a gymnastics injury. She admits to feeling a “pop” as she fell awkwardly during a landing.  She has had difficulty with ambulation since the injury.  On exam the patient is guarded and difficult to evaluate. She is unable to fully extend her right knee during the exam. Sagittal MRI images of the right knee are shown above. What is the most likely diagnosis?

    Correct

    The patient’s MRI findings and inability to fully extend her knee are consistent with a large bucket handle tear of the medial meniscus. The sagittal MRI (figure 2) shows a large displaced meniscal fragment in the intercondylar notch or a “double PCL sign”. The displaced fragment is being impinged and is blocking her from achieving full extension. Figure 1 shows intact anterior cruciate and posterior cruciate ligaments.

    The treatment of choice for a large bucket handle meniscus tear in a 16-year old is an arthroscopic meniscal repair. Meniscal tears that are stable after reduction and occur in the vascular zone, or within 3-5 mm of the periphery, are considered for repair. Unfortunately, unstable tears may require partial meniscectomy and later result in arthritic changes proportional to the amount of meniscus removed. Patients should be educated preoperatively on the significant difference in postoperative limitations between meniscal repair and meniscectomy. Meniscectomy patients may resume weight bearing as tolerated immediately and progress activities to tolerance postoperatively. Common postoperative protocols for meniscal repair include the avoidance of knee flexion during weight bearing for 4-6 weeks. Return to sports after repair varies significantly between providers; some recommend return as tolerated while others wait at least 4 months postoperatively. Currently there is no data to support one protocol over the other. The overall failure rate of meniscal repair can be as high as 30% with most failures occurring after 2 years.

    References

    1. Bucket handle meniscus tear. http://www.wheelessonline.com. Accessed on 1/20/15.

    2. Laible C, Stein DA, Kiridly DN. Meniscal Repair. JAAOS. 2013; 21: 204-213.

    3.  Nepple JJ, Dunn WR, Wright RW. Meniscal Repair Outcomes At Greater than Five Years. A systemic Literature Review and Meta-Analysis. JBJS. 2012; 94: 2222-7.

    Incorrect

    The patient’s MRI findings and inability to fully extend her knee are consistent with a large bucket handle tear of the medial meniscus. The sagittal MRI (figure 2) shows a large displaced meniscal fragment in the intercondylar notch or a “double PCL sign”. The displaced fragment is being impinged and is blocking her from achieving full extension. Figure 1 shows intact anterior cruciate and posterior cruciate ligaments.

    The treatment of choice for a large bucket handle meniscus tear in a 16-year old is an arthroscopic meniscal repair. Meniscal tears that are stable after reduction and occur in the vascular zone, or within 3-5 mm of the periphery, are considered for repair. Unfortunately, unstable tears may require partial meniscectomy and later result in arthritic changes proportional to the amount of meniscus removed. Patients should be educated preoperatively on the significant difference in postoperative limitations between meniscal repair and meniscectomy. Meniscectomy patients may resume weight bearing as tolerated immediately and progress activities to tolerance postoperatively. Common postoperative protocols for meniscal repair include the avoidance of knee flexion during weight bearing for 4-6 weeks. Return to sports after repair varies significantly between providers; some recommend return as tolerated while others wait at least 4 months postoperatively. Currently there is no data to support one protocol over the other. The overall failure rate of meniscal repair can be as high as 30% with most failures occurring after 2 years.

    References

    1. Bucket handle meniscus tear. http://www.wheelessonline.com. Accessed on 1/20/15.

    2. Laible C, Stein DA, Kiridly DN. Meniscal Repair. JAAOS. 2013; 21: 204-213.

    3.  Nepple JJ, Dunn WR, Wright RW. Meniscal Repair Outcomes At Greater than Five Years. A systemic Literature Review and Meta-Analysis. JBJS. 2012; 94: 2222-7.

  18. Question 18 of 50
    18. Question

    question 18

    A 16-year old male presents to your office with approximately 6 years of episodic left knee pain.  The pain started during football season but he can’t recall any injury. The pain was sharp in nature and he had difficulty ambulating after games. The pain improved in the off season but continued to bother him on occasion over the years. The pain has progressed over the past several months and now hurts with activities like climbing stairs, jumping, and running. He has occasional swelling but denies any locking or giving way of the knee. His primary care prescribed physical therapy which has not helped.  AP x-ray and coronal T2 MRI image of the patient’s left knee show a 2 cm by 3 cm osteochondral lesion of the medial femoral condyle. There is no linear high T2 signal intensity or cyst formation indicating the overlying hyaline cartilage is intact without gross instability. There is no step-off of the subchondral bone or overlying articular cartilage. During the knee athroscopy the lesion was probed and found to have a slight ridge over the posterior aspect of the OCD lesion. There was no softening or violation of the articular cartilage. Which choice is the next best step in treatment?

    Correct

    The patient’s left knee x-ray and MRI reveal osteochondritis dissecans with an in situ lesion of the medial femoral condyle. Osteochondritis dissecans (OCD) is a defect in the articular cartilage and subchondral bone that can present in varying degrees of severity. Injury can range from softening of the articular cartilage to detachment of an osteochondral lesion creating a displaced fragment or loose body. Typical presentation includes athletic males below the age of 18 with unilateral involvement. Lesions are most often located in the posterior lateral medial femoral condyle.  Pain is usually activity related and poorly localized. Although there is no single known cause of OCD the most widely accepted etiology is repetitive trauma. The pathology starts with softening of the articular cartilage, progressing to cartilage separation, then partial detachment of the lesion, and finally osteochondral detachment.

    Most OCD lesions can be seen on x-ray but an MRI is routinely ordered to determine size of the lesion and if the fragment is detached. The Clanton and Delee classification system is used to describe OCD lesions and is divided into 4 progressive stages. Stage 1 is a depression of the osteochondral fracture, stage 2 is a fragment attached to an osteochondral bridge, stage 3 is a detached non-displaced fragment, and stage 4 is a displaced fragment. Lesions assessed arthroscopically can be classified using the International Cartilage Repair Society (ICRS) scale of OCD lesions. Type 1 is a stable lesion with softened but intact articular cartilage, Type 2 has partial articular cartilage discontinuity but is stable when probed, Type 3 has complete articular cartilage discontinuity without detachment, and Type 4 is an empty defect with complete detachment or loose body.

    Juvenile OCD, occurring in patients with open growth plates, has a better prognosis in terms of healing with non-operative treatment compared to lesions in adults.  Stable lesions in children with open physes can be treated non-operatively with a period of non-weight bearing and bracing. The spectrum of non-operative treatment varies widely from non-weight bearing in a cylinder cast for 3 months to 6 weeks of crutch walking with immediate gentle range of motion. Healing rates approach 50-75% with conservative treatment.

    Surgery is indicated if non-operative treatment fails in patients with stable lesions. Arthroscopy and transarticular drilling are commonly performed to create vascular channels to the affected bone and cartilage to promote healing. Post operative rehabilitation may include a period of 4 weeks non-weight bearing and a slow, progressive return to activity thereafter.

    Unstable OCD lesions measuring greater than 2 cm require stabilization with some form of fixation. Arthroscopic techniques for internal fixation include compression screws, k-wire fixation, bone pegs, and bioabsorbable pins and nails.  The goal of fixation is to stabilize the lesion, promote healing, and halt the progression of arthritis. Lesions found to be loose during arthroscopy may be peeled back so that debridement can be performed prior to reduction and pinning.

    Unstable osteochondral lesions that are completely detached from underlying bone and irreducible should be treated with osteochondral autologous transplantation (OAT). Microfracture (MF) is generally used for smaller detached chondral fragments. OAT procedure has shown improved long term rates of return to sports over microfracture in patients with ICRS grade 3&4 OCD lesions.  Both OAT and MF have similar results at one year post operatively. However, due to factors including poor cartilage repair and decrease strength of fibrocartilage reparative tissue with MF, long term results are less favorable compared to an OAT procedure.

    References

    1. Schroeder GD, Lynch ST, Patel RM, Williams JR, Weathorford BM, Sarwark JF. Current concepts in the management of juvenile osteochondritis dissecans of the knee. Orthopedic Knowledge Online Journal. Accesed March 1, 2013.

    2. Osteochondritis dissecans. http://www.orthobullets. Accessed on 5/19/2015.

    Incorrect

    The patient’s left knee x-ray and MRI reveal osteochondritis dissecans with an in situ lesion of the medial femoral condyle. Osteochondritis dissecans (OCD) is a defect in the articular cartilage and subchondral bone that can present in varying degrees of severity. Injury can range from softening of the articular cartilage to detachment of an osteochondral lesion creating a displaced fragment or loose body. Typical presentation includes athletic males below the age of 18 with unilateral involvement. Lesions are most often located in the posterior lateral medial femoral condyle.  Pain is usually activity related and poorly localized. Although there is no single known cause of OCD the most widely accepted etiology is repetitive trauma. The pathology starts with softening of the articular cartilage, progressing to cartilage separation, then partial detachment of the lesion, and finally osteochondral detachment.

    Most OCD lesions can be seen on x-ray but an MRI is routinely ordered to determine size of the lesion and if the fragment is detached. The Clanton and Delee classification system is used to describe OCD lesions and is divided into 4 progressive stages. Stage 1 is a depression of the osteochondral fracture, stage 2 is a fragment attached to an osteochondral bridge, stage 3 is a detached non-displaced fragment, and stage 4 is a displaced fragment. Lesions assessed arthroscopically can be classified using the International Cartilage Repair Society (ICRS) scale of OCD lesions. Type 1 is a stable lesion with softened but intact articular cartilage, Type 2 has partial articular cartilage discontinuity but is stable when probed, Type 3 has complete articular cartilage discontinuity without detachment, and Type 4 is an empty defect with complete detachment or loose body.

    Juvenile OCD, occurring in patients with open growth plates, has a better prognosis in terms of healing with non-operative treatment compared to lesions in adults.  Stable lesions in children with open physes can be treated non-operatively with a period of non-weight bearing and bracing. The spectrum of non-operative treatment varies widely from non-weight bearing in a cylinder cast for 3 months to 6 weeks of crutch walking with immediate gentle range of motion. Healing rates approach 50-75% with conservative treatment.

    Surgery is indicated if non-operative treatment fails in patients with stable lesions. Arthroscopy and transarticular drilling are commonly performed to create vascular channels to the affected bone and cartilage to promote healing. Post operative rehabilitation may include a period of 4 weeks non-weight bearing and a slow, progressive return to activity thereafter.

    Unstable OCD lesions measuring greater than 2 cm require stabilization with some form of fixation. Arthroscopic techniques for internal fixation include compression screws, k-wire fixation, bone pegs, and bioabsorbable pins and nails.  The goal of fixation is to stabilize the lesion, promote healing, and halt the progression of arthritis. Lesions found to be loose during arthroscopy may be peeled back so that debridement can be performed prior to reduction and pinning.

    Unstable osteochondral lesions that are completely detached from underlying bone and irreducible should be treated with osteochondral autologous transplantation (OAT). Microfracture (MF) is generally used for smaller detached chondral fragments. OAT procedure has shown improved long term rates of return to sports over microfracture in patients with ICRS grade 3&4 OCD lesions.  Both OAT and MF have similar results at one year post operatively. However, due to factors including poor cartilage repair and decrease strength of fibrocartilage reparative tissue with MF, long term results are less favorable compared to an OAT procedure.

    References

    1. Schroeder GD, Lynch ST, Patel RM, Williams JR, Weathorford BM, Sarwark JF. Current concepts in the management of juvenile osteochondritis dissecans of the knee. Orthopedic Knowledge Online Journal. Accesed March 1, 2013.

    2. Osteochondritis dissecans. http://www.orthobullets. Accessed on 5/19/2015.

  19. Question 19 of 50
    19. Question

    Question 19

    A 38-year old male presents to the office with a history of severe left hamstring pain after an injury during a softball game 5 days ago. He was stretching for a catch at first base when his foot slipped causing him to do a near full split. He collapsed to the ground after the injury and had difficulty ambulating after. On exam he has moderate tenderness over the mid portion of the left hamstring but is able to extend his hip and flex his knee against resistance. There is no palpable or visual defect of the hamstring muscle compared to the contralateral leg. He has moderate ecchymosis to the area as shown in the figures above. X-rays obtained are negative for fracture. What is the next best step in treating this patient?

    Correct

    The hamstring consists of three muscles including the medial and superficial semitendinosus, deep medial semimembranosus, and the lateral biceps femoris. The biceps femoris is divided into two muscles; the superficial long head and deep short head. The biceps femoris attaches proximally to the ischial tuberosity and distally to the head of the fibula. The semimembranosus originates from the ischial tuberosity and attaches to the medial condyle of the tibia. The semitendinosus originates from the ischial tuberosity and attaches to the medial aspect of the tibia at the same attachment sites as the sartorius and gracilis tendons.  The area where these three tendons come together is call the Pes anserinus or “goose foot”. The hamstring muscles cross the hip and knee joint and function to extend the thigh and flex the knee.

    Hamstring injuries occur as a result of sudden hip flexion and knee extension. These injuries commonly occur in sprinting athletes and recreational “weekend warriors”. Any sport that requires rapid starts and stops may cause a hamstring tear. The hamstring can tear at any level with the proximal myotendinous junction being the most common location. An acute proximal hamstring rupture may tear all three tendons off the ischial tuberosity. Pediatric patients may avulse the ischial tuberosity apophysis which is often diagnosed by an acute displaced fracture on x-ray.
    Symptoms of a hamstring tear include a sudden sharp pain and occasionally, an audible “pop” when the injury occurs. Patients often have persistent pain and difficult ambulating after the injury. Acute hamstring tears can be diagnosed by physical exam alone.  Ecchymosis may develop several days to one week after the injury.  Knee flexion and hip extension strength should be assessed. Mild injuries may have no noticeable weakness.  A palpable and visible defect may be seen in complete retracted proximal tears of the myotendinous junction.  Significant ecchymosis and pain to palpation proximally warrants additional work-up including an MRI to determine displacement of the tear.  Pain in the mid to distal hamstrings without deformity does not require further imaging in most cases as midsubtance tears are treated conservatively.

    X-rays may be ordered to rule out an associated boney avulsion which can occur in the pediatric population. MRI is the modality of choice in patients with significant pain proximally. MRI can differentiate complete vs. partial tearing and helps measure the degree myotendinous displacement.

    Most hamstring injuries can be treated non-operatively with 4-6 weeks of protected weight bearing as tolerated, physical therapy, and pain control. Patients are allowed to begin gradual return to athletic activities when the hamstring strength is at 90% of the contralateral side. Patients with severe proximal injuries should have an MRI to determine tendon displacement as clinical assessment can be unreliable in judging tendon injury. MRI evaluation is particularly important in athletes who desire to return to sports as severity of injury seen on MRI can help predict expected length of rehabilitation. Acute proximal hamstring tears with > 2 cm of retraction in two out of three tendons and any displacement of all three tendons together should be treated surgically.  However, surgical decision making is largely based on the patient’s age and activity level and may vary between surgeons.  Surgical treatment for acute displaced proximal hamstring tears has shown improved rates of pre-injury return to sport and increased hamstring strength compared to non-operative treatment. Other indications for surgery include gluteal sciatica resulting from tendon displacement and failure of non-operative treatment.

    References

    1. Mohammad M. Alzahrani, MD, MSc(C); Sultan Aldebeyan, MD, MSc(C); Fahad Abduljabbar, MBBS; Paul A. Martineau, MD, FRCSC. Hamstring Injuries in Athletes: Diagnosis and Treatment. JBJS Reviews, 2015 Jun; 3 (6).

    2. Hamstring Injury. http://www.orthobullets.com. Accessed on 7/29/2015.

    Incorrect

    The hamstring consists of three muscles including the medial and superficial semitendinosus, deep medial semimembranosus, and the lateral biceps femoris. The biceps femoris is divided into two muscles; the superficial long head and deep short head. The biceps femoris attaches proximally to the ischial tuberosity and distally to the head of the fibula. The semimembranosus originates from the ischial tuberosity and attaches to the medial condyle of the tibia. The semitendinosus originates from the ischial tuberosity and attaches to the medial aspect of the tibia at the same attachment sites as the sartorius and gracilis tendons.  The area where these three tendons come together is call the Pes anserinus or “goose foot”. The hamstring muscles cross the hip and knee joint and function to extend the thigh and flex the knee.

    Hamstring injuries occur as a result of sudden hip flexion and knee extension. These injuries commonly occur in sprinting athletes and recreational “weekend warriors”. Any sport that requires rapid starts and stops may cause a hamstring tear. The hamstring can tear at any level with the proximal myotendinous junction being the most common location. An acute proximal hamstring rupture may tear all three tendons off the ischial tuberosity. Pediatric patients may avulse the ischial tuberosity apophysis which is often diagnosed by an acute displaced fracture on x-ray.
    Symptoms of a hamstring tear include a sudden sharp pain and occasionally, an audible “pop” when the injury occurs. Patients often have persistent pain and difficult ambulating after the injury. Acute hamstring tears can be diagnosed by physical exam alone.  Ecchymosis may develop several days to one week after the injury.  Knee flexion and hip extension strength should be assessed. Mild injuries may have no noticeable weakness.  A palpable and visible defect may be seen in complete retracted proximal tears of the myotendinous junction.  Significant ecchymosis and pain to palpation proximally warrants additional work-up including an MRI to determine displacement of the tear.  Pain in the mid to distal hamstrings without deformity does not require further imaging in most cases as midsubtance tears are treated conservatively.

    X-rays may be ordered to rule out an associated boney avulsion which can occur in the pediatric population. MRI is the modality of choice in patients with significant pain proximally. MRI can differentiate complete vs. partial tearing and helps measure the degree myotendinous displacement.

    Most hamstring injuries can be treated non-operatively with 4-6 weeks of protected weight bearing as tolerated, physical therapy, and pain control. Patients are allowed to begin gradual return to athletic activities when the hamstring strength is at 90% of the contralateral side. Patients with severe proximal injuries should have an MRI to determine tendon displacement as clinical assessment can be unreliable in judging tendon injury. MRI evaluation is particularly important in athletes who desire to return to sports as severity of injury seen on MRI can help predict expected length of rehabilitation. Acute proximal hamstring tears with > 2 cm of retraction in two out of three tendons and any displacement of all three tendons together should be treated surgically.  However, surgical decision making is largely based on the patient’s age and activity level and may vary between surgeons.  Surgical treatment for acute displaced proximal hamstring tears has shown improved rates of pre-injury return to sport and increased hamstring strength compared to non-operative treatment. Other indications for surgery include gluteal sciatica resulting from tendon displacement and failure of non-operative treatment.

    References

    1. Mohammad M. Alzahrani, MD, MSc(C); Sultan Aldebeyan, MD, MSc(C); Fahad Abduljabbar, MBBS; Paul A. Martineau, MD, FRCSC. Hamstring Injuries in Athletes: Diagnosis and Treatment. JBJS Reviews, 2015 Jun; 3 (6).

    2. Hamstring Injury. http://www.orthobullets.com. Accessed on 7/29/2015.

  20. Question 20 of 50
    20. Question

    Question 20

    A 33-year old male presents with left shoulder pain for three weeks. He denies injuring the shoulder and admits the pain is getting worse with physical activities. He is employed as a security guard and works out at the gym on a daily basis. He particularly notices the pain when doing bench presses. On exam the patient has full passive and active range of motion of the left shoulder. He has a negative Neer’s sign and O’brien’s test. He has increased pain with horizontal adduction of the left arm. X-rays are shown above. What is the most likely diagnosis?

    Correct

    Distal clavicle osteolysis is strongly associated with male weight trainers.  Osteolysis is thought to be caused by repetitive use causing microfracture of the subchondral bone. Weight lifters that work out frequently and don’t rest their shoulders in order to heal the microfractures will begin to have the bone of the AC joint break down causing pain. The AP shoulder x-ray above shows classic findings of subchondral bone loss, slight enlargement of the distal clavicle, and widening of the AC joint.

    Patients with distal clavicle osteolysis present with pain over the AC joint that is made worse with horizontal adduction. Common x-ray findings may also include osteopenia and cyst formation of the distal clavicle. The acromial side typically shows no changes on x-ray which helps differentiate osteolysis form AC joint arthritis. First line treatment for distal clavicle osteolysis includes ice, NSAIDS, and modifying the patient’s workout routine. Patients should keep their arms further apart while doing push-ups or bench press exercises and avoid dips, flies, and other lifts that elicit pain. Many patients unwilling to modify their work out routine will have continued pain. A corticosteroid injection to the AC joint is the next line treatment.  A steroid injection also helps confirm the diagnosis if the patient experiences pain relief after. An arthroscopic distal clavicle excision, also known as a Mumford procedure, is indicated in patients with persistent symptoms despite conservative treatment. During the procedure, approximately .5 to 1 cm of the distal clavicle is removed. Weight lifters can expect a return to full activities without pain approximately 6-8 weeks after the procedure.

    The patient has a negative Neer’s sign likely ruling out subacromial impingement, a negative O’brien’s test likely ruling out a labral tear, and full passive range of motion ruling out adhesive capsulitis.

    References

    1. Schwarzkopf R1, Ishak C, Elman M, Gelber J, Strauss DN, Jazrawi LM. Distal clavicular osteolysis: a review of the literature. Bull NYU Hosp Jt Dis. 2008;66(2):94-101.

    2. Cadet E, Ahmad CS, Levine WN. The management of acromioclavicular joint ostearthrosis: debride, resect, or leave it alone. AAOS Instructional Course Lectures, Volume 55, 2006.

    Incorrect

    Distal clavicle osteolysis is strongly associated with male weight trainers.  Osteolysis is thought to be caused by repetitive use causing microfracture of the subchondral bone. Weight lifters that work out frequently and don’t rest their shoulders in order to heal the microfractures will begin to have the bone of the AC joint break down causing pain. The AP shoulder x-ray above shows classic findings of subchondral bone loss, slight enlargement of the distal clavicle, and widening of the AC joint.

    Patients with distal clavicle osteolysis present with pain over the AC joint that is made worse with horizontal adduction. Common x-ray findings may also include osteopenia and cyst formation of the distal clavicle. The acromial side typically shows no changes on x-ray which helps differentiate osteolysis form AC joint arthritis. First line treatment for distal clavicle osteolysis includes ice, NSAIDS, and modifying the patient’s workout routine. Patients should keep their arms further apart while doing push-ups or bench press exercises and avoid dips, flies, and other lifts that elicit pain. Many patients unwilling to modify their work out routine will have continued pain. A corticosteroid injection to the AC joint is the next line treatment.  A steroid injection also helps confirm the diagnosis if the patient experiences pain relief after. An arthroscopic distal clavicle excision, also known as a Mumford procedure, is indicated in patients with persistent symptoms despite conservative treatment. During the procedure, approximately .5 to 1 cm of the distal clavicle is removed. Weight lifters can expect a return to full activities without pain approximately 6-8 weeks after the procedure.

    The patient has a negative Neer’s sign likely ruling out subacromial impingement, a negative O’brien’s test likely ruling out a labral tear, and full passive range of motion ruling out adhesive capsulitis.

    References

    1. Schwarzkopf R1, Ishak C, Elman M, Gelber J, Strauss DN, Jazrawi LM. Distal clavicular osteolysis: a review of the literature. Bull NYU Hosp Jt Dis. 2008;66(2):94-101.

    2. Cadet E, Ahmad CS, Levine WN. The management of acromioclavicular joint ostearthrosis: debride, resect, or leave it alone. AAOS Instructional Course Lectures, Volume 55, 2006.

  21. Question 21 of 50
    21. Question

    question 21

    A 9-year old boy presents with a 2 month history of right heel pain that has worsened over the past two weeks. He is an avid soccer player who plays at least 2 hours a day 6-7 days a week.  His parents state he is a very talented player who plays on a number of all-star teams throughout the year.  On exam of the heel he has tenderness over the calcaneal apophysis that is made worse with active plantar flexion and passive dorsiflexion. A lateral x-ray is shown above. What is the most likely diagnosis?

    Correct

    Based on the patient’s history of playing soccer on a daily basis without regular days of rest, an overuse injury should be suspected.  Calcaneal apophysitis, or Sever’s disease, is a common source of activity related heel pain in skeletally immature athletes. The calcaneal apophysis experiences strong pulling forces from both the Achilles and the plantar fascia during running. These repetitive forces can cause a traction apophysitis and heel pain, particularly in athletes who don’t rest or stretch enough.  Most patients will have pain to palpation over the apophysis on exam and a history of progressive, activity related heel pain. A lateral radiograph can help differentiate Sever’s disease from other sources of heel pain including bone tumors or stress fracture. Treatment involves activity modification, immobilization, physical therapy for Achilles tendon stretching and local modalities, NSAIDs, and heel cups. Casting may be necessary in non-compliant patients or if other treatments fail.  Recurrence is common until patients reach skeletal maturity. Osgood-Schlatter disease is a traction apophysitis of the tibial tubercle.  Sinding-Larsen-Johansson syndrome is a traction apophysitis of the distal pole of the patella. Iselin’s disease is a traction apophysitis at the base of the fifth metatarsal.

    References

    1. Frank JB, Jarit GJ, Bravman JT, Rosen JE. Lower Extremity Injuries in the Skeletally Immature Athlete. JAAOS 2007; 15: 356-366.

    2. Sever’s disease. http://www.orthobullets.com. Accessed on 12/20/15.

    Incorrect

    Based on the patient’s history of playing soccer on a daily basis without regular days of rest, an overuse injury should be suspected.  Calcaneal apophysitis, or Sever’s disease, is a common source of activity related heel pain in skeletally immature athletes. The calcaneal apophysis experiences strong pulling forces from both the Achilles and the plantar fascia during running. These repetitive forces can cause a traction apophysitis and heel pain, particularly in athletes who don’t rest or stretch enough.  Most patients will have pain to palpation over the apophysis on exam and a history of progressive, activity related heel pain. A lateral radiograph can help differentiate Sever’s disease from other sources of heel pain including bone tumors or stress fracture. Treatment involves activity modification, immobilization, physical therapy for Achilles tendon stretching and local modalities, NSAIDs, and heel cups. Casting may be necessary in non-compliant patients or if other treatments fail.  Recurrence is common until patients reach skeletal maturity. Osgood-Schlatter disease is a traction apophysitis of the tibial tubercle.  Sinding-Larsen-Johansson syndrome is a traction apophysitis of the distal pole of the patella. Iselin’s disease is a traction apophysitis at the base of the fifth metatarsal.

    References

    1. Frank JB, Jarit GJ, Bravman JT, Rosen JE. Lower Extremity Injuries in the Skeletally Immature Athlete. JAAOS 2007; 15: 356-366.

    2. Sever’s disease. http://www.orthobullets.com. Accessed on 12/20/15.

  22. Question 22 of 50
    22. Question

    figure 1 adn 2figure 3 and 4

    A 20 year-old soccer player presents to your clinic with two weeks of right-sided leg pain after a soccer injury. She strained the leg during a kick and developed swelling and pain shortly after. The patient is having a difficult time with walking and reports significant weakness with hip flexion and knee extension. She has a mass on the anterior thigh that has grown larger since the injury but has been the same size over the last week (figure 1). MRI was performed showing there to be a distal rectus femoris tear at the myotendinous junction. Sagittal, axial, and coronal MRI images are shown in figures 2, 3, and 4, respectively. What is the best treatment option for this patient?

     

    Correct

    The quadriceps muscle consists of a group of four muscles including the rectus femoris, vastus medialis, vastus lateralis, and vastus intermedius. The rectus femoris arises from two heads; a direct head that attaches to the anterior inferior iliac spine (AIIS) and a reflected head that attaches just above the acetabulum. The rectus inserts at the patella and acts to flex the thigh and extend the knee. The vastus lateralis, vastus intermedius, and vastus medialis attach at the proximal femur, insert at the patella, and act to extend the knee.

    The myotendinous junction of the rectus femoris is the most common site of a quadriceps injury. More often the injury is a strain rather than a tear. Because the rectus crosses both the hip and knee joints it is thought to be more prone to injury. The most common mechanism of injury includes a kicking motion when the hip is hyperextended and the knee is flexed causing combined loads of stretch and eccentric muscle activation of the quadriceps muscle. Avulsion of the rectus femoris can also occur but is rare in skeletally mature athletes. AIIS physeal avulsion fractures are commonly seen in the adolescent and pediatric populations. Rectus femoris avulsions in the adult population are more often reported in high level athletes such as profession football and soccer players.

    A discrete soft tissue mass on the anterior thigh may be seen in patients with rectus tears (figure 1). On physical exam, flexing the knee with the hip in extension can reproduce the pain by stretching the rectus (Ely’s test). Femur and pelvis radiographs are often performed after the injury but are usually negative. MRI is the study of choice to confirm the diagnosis of a rectus femoris tear.

    Rectus femoris tears are generally treated conservatively with a period rest, NSAIDS, and rehabilitation. A short period, usually 1-2 weeks, of protected weight bearing with crutches may be necessary before formal physical therapy is initiated. Return to sports can vary based on patient symptoms but generally occurs at 6-12 weeks after the injury. Minimally displaced tears will likely heal with a fibrous union and patients can return to competitive sports with minimal to no pain once healed. Surgical intervention may be considered in the setting of prolonged weakness and functional limitations. 3

    References
    1. Murphey, MD, Kuklo, TR, Sweet DE, Gibbons MH, Murphey, MD. Rectus Femoris Muscle Tear Appearing as a Pseudotumor. Am J Sports Med July 1998 vol. 26 no. 4 544-548

    2. Adler KL, Cook P, Giordano BD. Rehabilitation Following Proximal Rectus Femoris Repair: A Case Report. International Journal of Athletic Therapy & Training . May2015, Vol. 20 Issue 3, p25-30. 6p.

    3. Gamradt SC, Brophy RH, Barnes R, Warren RF, Byrd TW, Kelly BT. Nonoperative Treatment for Proximal Avulsion of the Rectus Femoris in Professional American Football. Am J Sports Med July 2009 vol. 37 no. 7 1370-1374

     

    Incorrect

    The quadriceps muscle consists of a group of four muscles including the rectus femoris, vastus medialis, vastus lateralis, and vastus intermedius. The rectus femoris arises from two heads; a direct head that attaches to the anterior inferior iliac spine (AIIS) and a reflected head that attaches just above the acetabulum. The rectus inserts at the patella and acts to flex the thigh and extend the knee. The vastus lateralis, vastus intermedius, and vastus medialis attach at the proximal femur, insert at the patella, and act to extend the knee.

    The myotendinous junction of the rectus femoris is the most common site of a quadriceps injury. More often the injury is a strain rather than a tear. Because the rectus crosses both the hip and knee joints it is thought to be more prone to injury. The most common mechanism of injury includes a kicking motion when the hip is hyperextended and the knee is flexed causing combined loads of stretch and eccentric muscle activation of the quadriceps muscle. Avulsion of the rectus femoris can also occur but is rare in skeletally mature athletes. AIIS physeal avulsion fractures are commonly seen in the adolescent and pediatric populations. Rectus femoris avulsions in the adult population are more often reported in high level athletes such as profession football and soccer players.

    A discrete soft tissue mass on the anterior thigh may be seen in patients with rectus tears (figure 1). On physical exam, flexing the knee with the hip in extension can reproduce the pain by stretching the rectus (Ely’s test). Femur and pelvis radiographs are often performed after the injury but are usually negative. MRI is the study of choice to confirm the diagnosis of a rectus femoris tear.

    Rectus femoris tears are generally treated conservatively with a period rest, NSAIDS, and rehabilitation. A short period, usually 1-2 weeks, of protected weight bearing with crutches may be necessary before formal physical therapy is initiated. Return to sports can vary based on patient symptoms but generally occurs at 6-12 weeks after the injury. Minimally displaced tears will likely heal with a fibrous union and patients can return to competitive sports with minimal to no pain once healed. Surgical intervention may be considered in the setting of prolonged weakness and functional limitations. 3

    References
    1. Murphey, MD, Kuklo, TR, Sweet DE, Gibbons MH, Murphey, MD. Rectus Femoris Muscle Tear Appearing as a Pseudotumor. Am J Sports Med July 1998 vol. 26 no. 4 544-548

    2. Adler KL, Cook P, Giordano BD. Rehabilitation Following Proximal Rectus Femoris Repair: A Case Report. International Journal of Athletic Therapy & Training . May2015, Vol. 20 Issue 3, p25-30. 6p.

    3. Gamradt SC, Brophy RH, Barnes R, Warren RF, Byrd TW, Kelly BT. Nonoperative Treatment for Proximal Avulsion of the Rectus Femoris in Professional American Football. Am J Sports Med July 2009 vol. 37 no. 7 1370-1374

     

  23. Question 23 of 50
    23. Question

    double pcl sign

    A 31 year-old male presents to your office with right knee pain after a soccer injury one week ago. He was running and made a cutting movement when he felt something in his knee “pop”. He was unable to continue playing and his knee has been swollen since. He is also having trouble bearing weight and cannot fully extend his right knee. MRI images (figures 1 and 2) show a “double PCL” sign.  What does this sign likely indicate?

    Correct

    The patient presents with a large bucket handle medial meniscus tear with a large meniscal fragment displaced into the intercondylar notch. This appears as a second smaller posterior cruciate ligament (PCL) on sagittal MRI images (double PCL sign). The displaced meniscus lies parallel and anterior inferior to the PCL.  A double PCL sign is highly specific for a displaced bucket handle meniscus tear. The coronal MRI image (figure 3) also shows absence of the medial meniscus in the medial compartment and a meniscal fragment in the inferior medial intercondylar notch.

    A displaced bucket handle tear often causes a mechanical block to knee extension, as seen in this patient’s presentation. Bucket handle tears commonly occur in conjunction with other ligamentous injuries including medial collateral and anterior cruciate ligament tears. A true vertical bucket handle meniscus tear that occurs in the red zone or vascular zone is usually amenable to repair. The decision to repair or resect the tear is often made at the time of arthroscopic surgery. Meniscus repair, often times done in conjunction with ACL reconstruction, is preferable in the younger population (age below 35-40 years old) to preserve articular cartilage over time. Meniscectomy in generally preferred in middle-aged patients with chondral wear already present.

    References

    1. Double PCL Sign. http://www.radiopaedia.org Accessed on 12/13/2016.
    2. Sommers, J. Meniscus Tears: Repair or Resect. JOPA. Volume 1, Issue 2. Pages 11-15.
    Incorrect

    The patient presents with a large bucket handle medial meniscus tear with a large meniscal fragment displaced into the intercondylar notch. This appears as a second smaller posterior cruciate ligament (PCL) on sagittal MRI images (double PCL sign). The displaced meniscus lies parallel and anterior inferior to the PCL.  A double PCL sign is highly specific for a displaced bucket handle meniscus tear. The coronal MRI image (figure 3) also shows absence of the medial meniscus in the medial compartment and a meniscal fragment in the inferior medial intercondylar notch.

    A displaced bucket handle tear often causes a mechanical block to knee extension, as seen in this patient’s presentation. Bucket handle tears commonly occur in conjunction with other ligamentous injuries including medial collateral and anterior cruciate ligament tears. A true vertical bucket handle meniscus tear that occurs in the red zone or vascular zone is usually amenable to repair. The decision to repair or resect the tear is often made at the time of arthroscopic surgery. Meniscus repair, often times done in conjunction with ACL reconstruction, is preferable in the younger population (age below 35-40 years old) to preserve articular cartilage over time. Meniscectomy in generally preferred in middle-aged patients with chondral wear already present.

    References

    1. Double PCL Sign. http://www.radiopaedia.org Accessed on 12/13/2016.
    2. Sommers, J. Meniscus Tears: Repair or Resect. JOPA. Volume 1, Issue 2. Pages 11-15.
  24. Question 24 of 50
    24. Question

    mcl

    A 16 year-old male presents to the office with right knee pain for one week after a lacrosse injury. He was struck on the outside of the knee by an opposing player and felt a sharp pain in the knee after. He was unable to continue playing and has had pain in the medial knee since. On exam he has grade 2 laxity with valgus stress to the right knee. He has a negative Lachman’s and no joint effusion. AP and lateral x-rays and a coronal MRI image of the right knee are shown above. You diagnose the patient with a medial collateral ligament (MCL) tear. Which statement is FALSE regarding MCL tears?

     

    Correct

    The medial collateral ligament (MCL), otherwise known as the tibial collateral ligament, stabilizes the knee against valgus forces. The MCL is the most commonly injured ligament in the knee. The mechanism of injury includes a direct blow to the lateral knee creating a valgus stress. The MCL most commonly tears at the femoral insertion, which has improved healing rates over tears occurring distally at the tibial side. The most common grading system used to define MCL ruptures describes the degree of laxity with valgus stress. A valgus load is placed with the knee in 20 to 30 degrees of flexion and laxity is described as Grade I, Grade II, or Grade III. Grade I is a first degree tear with tenderness to palpation over the MCL but without laxity. Grade II is a second degree tear with partial tearing of the MCL. In Grade II tears, laxity can be felt with valgus stress but the fibers are still opposed so a firm end point is noted. Grade III is a complete rupture of the MCL with gross laxity and no endpoint felt with valgus stress.

    An MRI is often ordered after MCL injuries if associated injuries, such as bone bruising, meniscal tears, and ACL tears, cannot be ruled out. Isolated Grade 1-2 MCL tears are treated non-operatively in a hinged knee brace and protected weight bearing. Physical therapy is initiated for supervised strengthening and stretching. Grade 1 tears may return to sports in 7-10 days with a brace if tolerated. Grade II injuries generally stay out of sports for 4-6 weeks, and occasionally longer if the tear occurs at the tibial side. Most Grade III injuries are treated non-operatively in a hinged brace for 6 weeks. Progressive weight bearing and strength training is continued throughout the period of bracing. A custom MCL brace may be used longer in athletes at a high risk of re-injury including sports like hockey and football. Operative treatment may be considered in patients with a complete rupture on the tibial side, patients with multi-ligament knee injuries, and continued laxity and disability despite conservative treatment.

    References

    1. MCL Tears. http://www.orthobullets.com. Accessed on 8/29/16.
    2. Wijdicks CA, Griffith CJ, Johansen S, Engebretsen L, LaPrade RF. Current Concepts Review: Injuries to the Medial Collateral Ligament and Associated Injuries to the Medial Side of the Knee. JBJS 2010; 92: 1266-80.
    Incorrect

    The medial collateral ligament (MCL), otherwise known as the tibial collateral ligament, stabilizes the knee against valgus forces. The MCL is the most commonly injured ligament in the knee. The mechanism of injury includes a direct blow to the lateral knee creating a valgus stress. The MCL most commonly tears at the femoral insertion, which has improved healing rates over tears occurring distally at the tibial side. The most common grading system used to define MCL ruptures describes the degree of laxity with valgus stress. A valgus load is placed with the knee in 20 to 30 degrees of flexion and laxity is described as Grade I, Grade II, or Grade III. Grade I is a first degree tear with tenderness to palpation over the MCL but without laxity. Grade II is a second degree tear with partial tearing of the MCL. In Grade II tears, laxity can be felt with valgus stress but the fibers are still opposed so a firm end point is noted. Grade III is a complete rupture of the MCL with gross laxity and no endpoint felt with valgus stress.

    An MRI is often ordered after MCL injuries if associated injuries, such as bone bruising, meniscal tears, and ACL tears, cannot be ruled out. Isolated Grade 1-2 MCL tears are treated non-operatively in a hinged knee brace and protected weight bearing. Physical therapy is initiated for supervised strengthening and stretching. Grade 1 tears may return to sports in 7-10 days with a brace if tolerated. Grade II injuries generally stay out of sports for 4-6 weeks, and occasionally longer if the tear occurs at the tibial side. Most Grade III injuries are treated non-operatively in a hinged brace for 6 weeks. Progressive weight bearing and strength training is continued throughout the period of bracing. A custom MCL brace may be used longer in athletes at a high risk of re-injury including sports like hockey and football. Operative treatment may be considered in patients with a complete rupture on the tibial side, patients with multi-ligament knee injuries, and continued laxity and disability despite conservative treatment.

    References

    1. MCL Tears. http://www.orthobullets.com. Accessed on 8/29/16.
    2. Wijdicks CA, Griffith CJ, Johansen S, Engebretsen L, LaPrade RF. Current Concepts Review: Injuries to the Medial Collateral Ligament and Associated Injuries to the Medial Side of the Knee. JBJS 2010; 92: 1266-80.
  25. Question 25 of 50
    25. Question

    A 45 year-old female presents to the office with a history of left heel pain for several months. She has a past medical history of diabetes, high cholesterol, hypertension, and depression. Her pain is made worse with running and stairs. Recent x-rays taken include an AP mortise and lateral view of her ankle show no abnormalities. She has been diagnosed with Achilles tendonitis and has tried ice, NSAIDs, rest, and physical therapy without much relief. Which medication could be contributing to her tendonitis?

    Correct

    Statins, along with fluoroquinolones, are known to cause systemic side effects including tendinitis and tendon rupture. For patients with hypercholesterolemia, a statin is by far the most common medication used. Statins are the most effective medication in reducing LDL cholesterol which reduces risk of coronary artery disease, stroke, and death. Statins can cause a number of side effects including myalgias, myopathy, tendinitis, and elevated liver enzymes. The etiology of why statins cause tendinopathy is unclear, although the most widely accepted theory is that statins interfere with healing after microtrauma to the tendon. The Achilles tendon is the most susceptible to injury due to size and repetitive use. In effected patients, most will experience symptoms within four months after starting a statin. Symptoms usually resolve within 2-3 weeks after stopping the statin. Bottom line: patients at higher risk of tendinitis, including those with diabetes, hyperuricemia, and who participate in sports, should be monitored closely while on a statin. 1,2
    Answer D.
    References
    1. Deren ME, Klinge SA, Mukand NH, Mukand JA. Tendinopathy and Tendon Rupture Associated with Statins. JBJS Reviews 2016; 4 (5). 1-6.
    2. 1.Marie I, Delafenêtre H, Massy N, et al. Tendinous disorders attributed to statins: A study on ninety-six spontaneous reports in the period 1990-2005 and review of the literature. Arthritis Care Res. 2008;59:367-372.

    Incorrect

    Statins, along with fluoroquinolones, are known to cause systemic side effects including tendinitis and tendon rupture. For patients with hypercholesterolemia, a statin is by far the most common medication used. Statins are the most effective medication in reducing LDL cholesterol which reduces risk of coronary artery disease, stroke, and death. Statins can cause a number of side effects including myalgias, myopathy, tendinitis, and elevated liver enzymes. The etiology of why statins cause tendinopathy is unclear, although the most widely accepted theory is that statins interfere with healing after microtrauma to the tendon. The Achilles tendon is the most susceptible to injury due to size and repetitive use. In effected patients, most will experience symptoms within four months after starting a statin. Symptoms usually resolve within 2-3 weeks after stopping the statin. Bottom line: patients at higher risk of tendinitis, including those with diabetes, hyperuricemia, and who participate in sports, should be monitored closely while on a statin. 1,2
    Answer D.
    References
    1. Deren ME, Klinge SA, Mukand NH, Mukand JA. Tendinopathy and Tendon Rupture Associated with Statins. JBJS Reviews 2016; 4 (5). 1-6.
    2. 1.Marie I, Delafenêtre H, Massy N, et al. Tendinous disorders attributed to statins: A study on ninety-six spontaneous reports in the period 1990-2005 and review of the literature. Arthritis Care Res. 2008;59:367-372.

  26. Question 26 of 50
    26. Question

    A 17 year-old male presents to the office with right elbow pain for several months. He plays baseball 7 months out of the year and either pitches or plays third base. His pain is made worse when throwing and he has lost velocity on his fastball. Ice and NSIADs provide little relief. He tried taking a few months off from pitching but the pain returned as he started throwing again. On exam you note tenderness over the radiocapitellar joint and pain during the lateral compression test. He has no medial sided laxity or pain with valgus stress. AP and lateral x-rays of the right elbow are normal. What is the most likely diagnosis?

    Correct

    The overhead throwing motion places significant stress on the elbow which can often lead to overuse injury. Determining pathology in the overhead athlete starts with the basic understanding that a valgus force is placed on the elbow during the throwing motion. This causes tensile forces to the medial elbow and compression forces to the lateral elbow. Common sources of medial elbow pain in the throwing athlete are often attributed to a stretch of the medial structures including ulnar collateral ligament tearing, ulnar neuritis, flexor pronator strain, and medial apophysitis. Repetitive compressive forces at the lateral elbow may cause a condition called osteochondritis dissecans (OCD) of the capitellum. 1
    OCD of the capitellum occurs when repetitive trauma starts to damage the articular cartilage and subchondral bone of the capitellum. The condition generally occurs between the ages of 11 and 21 and is more common in males. Pain is often described as worse with activities and better with rest, with the occasional catching or locking occurring during throwing motion. On exam tenderness along the radiocapitellar articulation and pain with the lateral compression test is often noted. Radiographs are generally negative early in the disease but may show irregularity to the articular surface as the disease advances. MRI is the most sensitive study for picking up OCD early. Early stable OCD can be treated successfully with 3-6 weeks of rest followed by a gradual return to throwing. Large or unstable OCD lesions often require operative debridement and lesion excision, debridement and bone marrow stimulation, or fragment fixation. (Note: Panner disease occurs almost exclusively in males under 10 years of age and is not associated with trauma.) 1,2
    References
    1. Patel RM, Lynch TS, Amin NH, Gryzlo S, Schickenantz M. Elbow Injuries in the Throwing Athlete. JBJS Reviews 2014; 2 (11). Pages 1-11.
    2. Ruchelsman DE, Hall MP, Youm T. Osteochondritis Dissecans of the Capitellum: Current Concepts. JAAOS 2010; 18: 557-567.

    Incorrect

    The overhead throwing motion places significant stress on the elbow which can often lead to overuse injury. Determining pathology in the overhead athlete starts with the basic understanding that a valgus force is placed on the elbow during the throwing motion. This causes tensile forces to the medial elbow and compression forces to the lateral elbow. Common sources of medial elbow pain in the throwing athlete are often attributed to a stretch of the medial structures including ulnar collateral ligament tearing, ulnar neuritis, flexor pronator strain, and medial apophysitis. Repetitive compressive forces at the lateral elbow may cause a condition called osteochondritis dissecans (OCD) of the capitellum. 1
    OCD of the capitellum occurs when repetitive trauma starts to damage the articular cartilage and subchondral bone of the capitellum. The condition generally occurs between the ages of 11 and 21 and is more common in males. Pain is often described as worse with activities and better with rest, with the occasional catching or locking occurring during throwing motion. On exam tenderness along the radiocapitellar articulation and pain with the lateral compression test is often noted. Radiographs are generally negative early in the disease but may show irregularity to the articular surface as the disease advances. MRI is the most sensitive study for picking up OCD early. Early stable OCD can be treated successfully with 3-6 weeks of rest followed by a gradual return to throwing. Large or unstable OCD lesions often require operative debridement and lesion excision, debridement and bone marrow stimulation, or fragment fixation. (Note: Panner disease occurs almost exclusively in males under 10 years of age and is not associated with trauma.) 1,2
    References
    1. Patel RM, Lynch TS, Amin NH, Gryzlo S, Schickenantz M. Elbow Injuries in the Throwing Athlete. JBJS Reviews 2014; 2 (11). Pages 1-11.
    2. Ruchelsman DE, Hall MP, Youm T. Osteochondritis Dissecans of the Capitellum: Current Concepts. JAAOS 2010; 18: 557-567.

  27. Question 27 of 50
    27. Question

    A 35 year-old male presents to the office with right hip pain after a twist and split type fall while working construction 3 months ago. He has experienced sharp pain when rotating his hip since the injury. On exam he has pain with hip flexion and internal rotation. AP and lateral x-rays of the hip show no signs of femoroacetabular impingement or arthritis. MRI arthrogram shows a degenerative labral tear. Which statement below is accurate regarding treatment with an arthroscopic labral debridement?

    Correct

    Hip arthroscopy is an increasingly utilized procedure in orthopedic practice. Debridement of labral tears remains the most common indication for hip arthroscopy. Indications for hip arthroscopy include hip pain beyond one month, positive findings on physical exam, and MRI confirmation of labral pathology. Hip arthroscopy with debridement is an effective treatment option for patients with degenerative labral tears. Arthroscopic debridement with partial labrectomy has been found to reduce hip pain and resolve mechanical symptoms in 91% of patients with labral tears. Poor prognostic indicators after hip arthroscopy include pre-existing arthritis, femoroacetabular impingement, and age over 45 years old. During an arthroscopic labral debridement loose flaps of labrum and calcifications are shaved away leaving as much normal labrum intact as possible. Patients with an unstable degenerative flap type labral tear, like this patient, will likely do well with an arthroscopic debridement. A labral repair is performed only if the labrum is non-viable after debridement.

    References
    1. Wilkin G, March G, Beaule PE. Athroscopic Acetabular Labral Debridement in Patients Forty Five Years of Age or Older Has Minimal Benefit for Pain and Function. JBJS 2014; 96: 113-8.
    2. Domb BG, Hartigan DE, Perets I. Decision Making for Labral Treatment in the Hip: Repair Versus Debridement Versus Reconstruction. JAAOS 2017; 25: e53-e62

    Incorrect

    Hip arthroscopy is an increasingly utilized procedure in orthopedic practice. Debridement of labral tears remains the most common indication for hip arthroscopy. Indications for hip arthroscopy include hip pain beyond one month, positive findings on physical exam, and MRI confirmation of labral pathology. Hip arthroscopy with debridement is an effective treatment option for patients with degenerative labral tears. Arthroscopic debridement with partial labrectomy has been found to reduce hip pain and resolve mechanical symptoms in 91% of patients with labral tears. Poor prognostic indicators after hip arthroscopy include pre-existing arthritis, femoroacetabular impingement, and age over 45 years old. During an arthroscopic labral debridement loose flaps of labrum and calcifications are shaved away leaving as much normal labrum intact as possible. Patients with an unstable degenerative flap type labral tear, like this patient, will likely do well with an arthroscopic debridement. A labral repair is performed only if the labrum is non-viable after debridement.

    References
    1. Wilkin G, March G, Beaule PE. Athroscopic Acetabular Labral Debridement in Patients Forty Five Years of Age or Older Has Minimal Benefit for Pain and Function. JBJS 2014; 96: 113-8.
    2. Domb BG, Hartigan DE, Perets I. Decision Making for Labral Treatment in the Hip: Repair Versus Debridement Versus Reconstruction. JAAOS 2017; 25: e53-e62

  28. Question 28 of 50
    28. Question

    A 49 year-old female presents with a 4 month history of left shoulder pain. She enjoys going to the gym and cross training but her shoulder has been bothering her with overhead activities. She received a subacromial injection 2 months ago then did several weeks of physical therapy with minimal relief. AP and outlet radiographs show no abnormalities. MRI shows a partial thickness articular sided rotator cuff tear of the left shoulder. The patient is taken to the operating for a left shoulder arthroscopy which reveals a partial tear through 60% of the depth of the supraspinatus tendon. What would be the next best step in treatment?

    Correct

    Partial thickness tears of the rotator cuff present a treatment dilemma for clinicians who treat shoulder pain. These patients often fail conservative treatment with rest, NSAIDS, subacromial injections, and physical therapy before electing for surgery. Partial thickness tears have limited healing potential and will likely progress over time. Despite this, most patients will continue to improve with conservative treatment up to 18 months. However, it is reasonable to consider surgery when conservative treatment has failed for a period of 3-6 months. Many partial tears that are treated with arthroscopy, rotator cuff debridement, and acromioplasty have good to excellent results at 2 to 5 years. However, several factors must be considered during shoulder arthroscopically that help the surgeon determine the best course of treatment. Such factors include location of the partial rotator cuff tear, size of the tear, and other should pathology seen during arthroscopy. Arthroscopic debridement of partial tears is essential in determining the true depth of the tear and may stimulate healing and remove inflammatory cells. Partial thickness tears can occur on the articular side, intratendinous, bursal side, or combination thereof. Articular sided supraspinatus tendon tears are the most common location in older patients and bursal sided tears are typically more symptomatic. Bursal sided tears are more likely to progress to symptomatic full thickness tears than articular sided tears. Partial tears greater than 50% of the tendon depth usually do poorly with conservative treatment. The thickness of a rotator cuff measures approximately 11-12 mm so a tear >6 mm in depth on the articular side and >3 mm in depth on the bursal side will do poorly with observation and should be treated with a rotator cuff repair. When the partial tear is >50% of the tendon depth the tear is often completed then repaired back down to the humerus with suture anchors. Younger overhead athletes with partial rotator cuff tears are the exception to these rules as non-surgical treatment should be the mainstay of treatment. These patients often fail to return to the same level of activity after rotator cuff surgery, particularly repair. Adequate rehab, rest, NSAIDS, and possible steroid injections can effectively resolve symptoms in this population. 1,2

    References:
    1. Wolff AB, Sethi P, Sutton KM, Covey AS, Magit DP, Medvecky M. Partial Thickness Rotator Cuff Tears. JAAOS 2006; 14: 715-725.
    2. Shaffer B, Huttman D. Rotator Cuff Tears in the Throwing Athlete. 2014. Sports Med Arthrosc Rev. Volume 22, number 2. 101-109.

    Incorrect

    Partial thickness tears of the rotator cuff present a treatment dilemma for clinicians who treat shoulder pain. These patients often fail conservative treatment with rest, NSAIDS, subacromial injections, and physical therapy before electing for surgery. Partial thickness tears have limited healing potential and will likely progress over time. Despite this, most patients will continue to improve with conservative treatment up to 18 months. However, it is reasonable to consider surgery when conservative treatment has failed for a period of 3-6 months. Many partial tears that are treated with arthroscopy, rotator cuff debridement, and acromioplasty have good to excellent results at 2 to 5 years. However, several factors must be considered during shoulder arthroscopically that help the surgeon determine the best course of treatment. Such factors include location of the partial rotator cuff tear, size of the tear, and other should pathology seen during arthroscopy. Arthroscopic debridement of partial tears is essential in determining the true depth of the tear and may stimulate healing and remove inflammatory cells. Partial thickness tears can occur on the articular side, intratendinous, bursal side, or combination thereof. Articular sided supraspinatus tendon tears are the most common location in older patients and bursal sided tears are typically more symptomatic. Bursal sided tears are more likely to progress to symptomatic full thickness tears than articular sided tears. Partial tears greater than 50% of the tendon depth usually do poorly with conservative treatment. The thickness of a rotator cuff measures approximately 11-12 mm so a tear >6 mm in depth on the articular side and >3 mm in depth on the bursal side will do poorly with observation and should be treated with a rotator cuff repair. When the partial tear is >50% of the tendon depth the tear is often completed then repaired back down to the humerus with suture anchors. Younger overhead athletes with partial rotator cuff tears are the exception to these rules as non-surgical treatment should be the mainstay of treatment. These patients often fail to return to the same level of activity after rotator cuff surgery, particularly repair. Adequate rehab, rest, NSAIDS, and possible steroid injections can effectively resolve symptoms in this population. 1,2

    References:
    1. Wolff AB, Sethi P, Sutton KM, Covey AS, Magit DP, Medvecky M. Partial Thickness Rotator Cuff Tears. JAAOS 2006; 14: 715-725.
    2. Shaffer B, Huttman D. Rotator Cuff Tears in the Throwing Athlete. 2014. Sports Med Arthrosc Rev. Volume 22, number 2. 101-109.

  29. Question 29 of 50
    29. Question

    A 17-year old male presents to your clinic after sustaining a right knee injury during a lacrosse game. He felt a “pop” in the knee as he pivoted to avoid an opponent. His primary care physician ordered an MRI after the injury which showed a complete anterior cruciate ligament rupture. You discuss with the patient the recommendations of ACL reconstruction including which graft types are available. For this patient, which would be the recommended graft choice to reconstruct his ACL?

    Correct

    The anterior cruciate ligament (ACL) is a crucial stabilizing structure in the knee that primarily provides restraint to anterior translation and rotation of the tibia. A deficient ACL causes the knee to give out or become unstable with activities that require running and cutting such as athletics. A deficient ACL may also cause buckling of the knee with day to day activities such as walking down stairs or changing direction quickly. In most active patients an ACL deficient knee must be treated acutely as damage to the menisci and articular cartilage can occur with instability episodes. This involves an arthroscopic procedure that takes out the torn tissue and reconstructs the ligament with a graft, either from the patient’s own tissue (autograft) or donated cadaver tissue (allograft).
    There are several factors to be considered as patients and orthopedic surgeons choose which graft to use. The differences of the success rates between graft choices are small so the benefits and downfalls of each should be discussed and considered. Autografts have the advantage of using the patient’s own tissue which tend to heal a little faster than donated tissue. The most commonly used graft type in the US is a bone patella bone autograft with the second type being hamstring autograft. Professional athletes most commonly choose bone patella bone autograft followed by hamstring autograft. The downside of autografts is the additional pain postoperatively that is associated with harvesting the graft. Allografts have a higher failure rate in young athletes but have less pain after surgery and tend to rehab a little quicker. This is why recreational athletes over 35 to 40 years of age generally choose an allograft for a quicker return to work. Surgeon’s preference also plays a major role when patients select graft choice. Graft choice can come with its own unique surgical approach/technique and often surgeons choose the graft and technique they are most familiar with.

    References
    Furio N, Yin KL, Marx RG. Graft Selection and Preparation in Anterior Cruciate Ligament Reconstruction. JBJS Journal of Orthopaedics for Physician Assistants: January – March 2017 – Volume 5 – Issue 1 – p e6
    ACL Tear. http://www.orthobullets.com. Accessed on 4/24/2017.

    Incorrect

    The anterior cruciate ligament (ACL) is a crucial stabilizing structure in the knee that primarily provides restraint to anterior translation and rotation of the tibia. A deficient ACL causes the knee to give out or become unstable with activities that require running and cutting such as athletics. A deficient ACL may also cause buckling of the knee with day to day activities such as walking down stairs or changing direction quickly. In most active patients an ACL deficient knee must be treated acutely as damage to the menisci and articular cartilage can occur with instability episodes. This involves an arthroscopic procedure that takes out the torn tissue and reconstructs the ligament with a graft, either from the patient’s own tissue (autograft) or donated cadaver tissue (allograft).
    There are several factors to be considered as patients and orthopedic surgeons choose which graft to use. The differences of the success rates between graft choices are small so the benefits and downfalls of each should be discussed and considered. Autografts have the advantage of using the patient’s own tissue which tend to heal a little faster than donated tissue. The most commonly used graft type in the US is a bone patella bone autograft with the second type being hamstring autograft. Professional athletes most commonly choose bone patella bone autograft followed by hamstring autograft. The downside of autografts is the additional pain postoperatively that is associated with harvesting the graft. Allografts have a higher failure rate in young athletes but have less pain after surgery and tend to rehab a little quicker. This is why recreational athletes over 35 to 40 years of age generally choose an allograft for a quicker return to work. Surgeon’s preference also plays a major role when patients select graft choice. Graft choice can come with its own unique surgical approach/technique and often surgeons choose the graft and technique they are most familiar with.

    References
    Furio N, Yin KL, Marx RG. Graft Selection and Preparation in Anterior Cruciate Ligament Reconstruction. JBJS Journal of Orthopaedics for Physician Assistants: January – March 2017 – Volume 5 – Issue 1 – p e6
    ACL Tear. http://www.orthobullets.com. Accessed on 4/24/2017.

  30. Question 30 of 50
    30. Question


    An 18 year-old male presents to the office one week after sustaining a patella dislocation while playing football. He admits to falling awkwardly and noticing a deformity to his knee cap. He was able to reduce the patella with his hand and was able to walk after the reduction. He now presents with swelling and significant pain when flexing his right knee. Sunrise view x-ray (figure 1) shows a reduced patella located centrally within the trochlea groove and a small avulsion type fracture off the medial patella. What is the next best step in this patient’s treatment?

    Correct

    The patient presents with swelling and pain after a patella dislocation. Given his swelling and x-ray findings, an MRI should be ordered to rule out an osteochondral defect and injury to the medial patellofemoral ligament (MPFL). A traumatic patella dislocation can break off an osteochondral fragment, generally off the lateral femoral condyle as the patella dislocates laterally. An osteochondral fragment can break off and float around the knee causing symptoms of a loose body. Symptoms include recurrent effusions and mechanical symptoms of catching or locking of the knee. The incidence of loose body formation after patella dislocation was found to be 22% in one study. 1
    MPFL disruption may occur in up to 96% of patients that sustain a traumatic patella dislocation. The MPFL is the primary soft tissue stabilizer to lateral dislocation of the patella. The MPFL also helps the patella stay located centrally within the trochlea during knee flexion. MRI is the study of choice to determine if the MPFL is intact after a traumatic patella dislocation with a sensitivity of 85% and an accuracy of 80%. The standard of care for patella dislocations with MPFL disruption and without an OCD lesion is non-operative treatment. Conservative treatment is usually successfully with redislocation rates ranging from 15% to 44%. 2

    References
    1. Farr J, Covell DJ, Lattermann C. Cartilage lesions in patellofemoral dislocations: Incidents/locations/when to treat. Sports Med Arthrosc. 2012 Sep; 20(3): 181–186.
    2. Amin NH, Lynch TS, Patel RM, Patel N, Saluan P. Medial Patellafemoral ligament reconstruction. JBJS Reviews 2015; 3 (7): e3

    Incorrect

    The patient presents with swelling and pain after a patella dislocation. Given his swelling and x-ray findings, an MRI should be ordered to rule out an osteochondral defect and injury to the medial patellofemoral ligament (MPFL). A traumatic patella dislocation can break off an osteochondral fragment, generally off the lateral femoral condyle as the patella dislocates laterally. An osteochondral fragment can break off and float around the knee causing symptoms of a loose body. Symptoms include recurrent effusions and mechanical symptoms of catching or locking of the knee. The incidence of loose body formation after patella dislocation was found to be 22% in one study. 1
    MPFL disruption may occur in up to 96% of patients that sustain a traumatic patella dislocation. The MPFL is the primary soft tissue stabilizer to lateral dislocation of the patella. The MPFL also helps the patella stay located centrally within the trochlea during knee flexion. MRI is the study of choice to determine if the MPFL is intact after a traumatic patella dislocation with a sensitivity of 85% and an accuracy of 80%. The standard of care for patella dislocations with MPFL disruption and without an OCD lesion is non-operative treatment. Conservative treatment is usually successfully with redislocation rates ranging from 15% to 44%. 2

    References
    1. Farr J, Covell DJ, Lattermann C. Cartilage lesions in patellofemoral dislocations: Incidents/locations/when to treat. Sports Med Arthrosc. 2012 Sep; 20(3): 181–186.
    2. Amin NH, Lynch TS, Patel RM, Patel N, Saluan P. Medial Patellafemoral ligament reconstruction. JBJS Reviews 2015; 3 (7): e3

  31. Question 31 of 50
    31. Question

    A 14 year old male presents to the office with left knee pain after playing basketball 2 days ago. He is diagnosed with a grade I medial collateral ligament (MCL) sprain but a large 1.6 cm osteochondral (OCD) lesion on the medial femoral condyle is found incidentally on an AP x-ray (figure 1). Coronal MRI of the knee shows intact articular cartilage with no reactive edema around the OCD lesion. What is the best treatment option?

    Correct

    The patient has a large asymptomatic osteochondral lesion of the medial femoral condyle. MRI shows intact articular cartilage over the OCD lesion and no increase signal intensity in the surrounding bone which represents a stable lesion. This lesion was found incidentally on x-ray and is not causing the patients symptoms. Skeletally immature (juvenile) patients with an open physis and a stable lesion usually heal with non-operative treatment. Stable OCD lesions often become asymptomatic long before bone marrow edema subsides on MRI and x-rays shows signs of healing. For that reason, asymptomatic patients with stable lesions are usually not followed with serial x-rays or additional MRI. Skeletally immature patients that are symptomatic and have unstable lesions are unlikely to heal with conservative treatment and usually require surgery to prevent early onset arthritis. Osteochondral lesions in adults usually require operative treatment to prevent detachment of the OCD lesion and resulting early onset arthritis. Restricting running and jumping activities in juvenile patients with asymptomatic lesions has not been shown to improve outcomes. 1,2
    Answer A
    References
    1. Eric J. Wall, MD,1 Jason Vourazeris, BS,1 Gregory D. Myer, MS, CSCS,1 Kathleen H. Emery, MD,1 Jon G. Divine, MD,1 Todd G. Nick, PhD,1 and Timothy E. Hewett, PhD1 The Healing Potential of Stable Juvenile Osteochondritis Dissecans Knee Lesions. J Bone Joint Surg Am. 2008 Dec 1; 90(12): 2655–2664
    2. Osteochondral lesions. http://www.orthobullets.com. Accessed on 12/22/2019.

    Incorrect

    The patient has a large asymptomatic osteochondral lesion of the medial femoral condyle. MRI shows intact articular cartilage over the OCD lesion and no increase signal intensity in the surrounding bone which represents a stable lesion. This lesion was found incidentally on x-ray and is not causing the patients symptoms. Skeletally immature (juvenile) patients with an open physis and a stable lesion usually heal with non-operative treatment. Stable OCD lesions often become asymptomatic long before bone marrow edema subsides on MRI and x-rays shows signs of healing. For that reason, asymptomatic patients with stable lesions are usually not followed with serial x-rays or additional MRI. Skeletally immature patients that are symptomatic and have unstable lesions are unlikely to heal with conservative treatment and usually require surgery to prevent early onset arthritis. Osteochondral lesions in adults usually require operative treatment to prevent detachment of the OCD lesion and resulting early onset arthritis. Restricting running and jumping activities in juvenile patients with asymptomatic lesions has not been shown to improve outcomes. 1,2
    Answer A
    References
    1. Eric J. Wall, MD,1 Jason Vourazeris, BS,1 Gregory D. Myer, MS, CSCS,1 Kathleen H. Emery, MD,1 Jon G. Divine, MD,1 Todd G. Nick, PhD,1 and Timothy E. Hewett, PhD1 The Healing Potential of Stable Juvenile Osteochondritis Dissecans Knee Lesions. J Bone Joint Surg Am. 2008 Dec 1; 90(12): 2655–2664
    2. Osteochondral lesions. http://www.orthobullets.com. Accessed on 12/22/2019.

  32. Question 32 of 50
    32. Question

    A 21 year-old female volleyball player presents with persistent left shoulder pain with overhead activities. She has failed conservative treatment with several weeks of physical therapy, rest, and anti-inflammatories. MRI shows a superior labral anterior-posterior (SLAP) tear. Which physical exam test would most likely be positive for this patient?

    Correct

    Superior labral anterior-posterior (SLAP) lesions occur where the biceps tendon attaches to the superior labrum. The injury occurs as the superior labrum tears off the glenoid attachment. SLAP lesions place strain on the anterior band of the inferior glenohumeral ligament which compromises shoulder stability. In patients under 40, SLAP lesions can develop from repetitive overhead sports or a fall on an outstretched hand that pulls on the biceps anchor on the labrum. SLAP lesions can also occur over time and are generally considered a degenerative condition in patients over 40 years old. An O’Brien’s test, also known as the active compression test, is commonly used to diagnose SLAP tears. The test involves placing the arm in 90 degrees of forward flexion, 20 degrees of horizontal adduction, and internal rotation (thumb pointing down). The patient is then instructed to forward flex the arm against resistance. The test is repeated with the arm internally rotated and then externally rotated. Pain with internal rotation but not external rotation is a positive test. The O’Brien’s test stresses the labrum by tightening the posterior capsule and posteriorly translating the humeral head. This stress causes pain and weakness during the O’Brien’s test when the superior labrum is torn. MRI arthrogram is the diagnostic study of choice for SLAP tears. The arthrogram dye can be seen underneath and into the superior labral tear in figure 1.
    Answer D
    References
    1. Owen M, Boulter T, Walton M, Funk L, Mackenzie T. Reinterpretation of O’Brien test in posterior labral tears of the shoulder. Int J Shoulder Surg. 2015 Jan-Mar; 9(1): 6–8.
    2. Marx R, Camp C. Labral Tears of the Shoulder. Clinical Summary. http://www.JBJS.org. Retreived on 12/20/18.

    Incorrect

    Superior labral anterior-posterior (SLAP) lesions occur where the biceps tendon attaches to the superior labrum. The injury occurs as the superior labrum tears off the glenoid attachment. SLAP lesions place strain on the anterior band of the inferior glenohumeral ligament which compromises shoulder stability. In patients under 40, SLAP lesions can develop from repetitive overhead sports or a fall on an outstretched hand that pulls on the biceps anchor on the labrum. SLAP lesions can also occur over time and are generally considered a degenerative condition in patients over 40 years old. An O’Brien’s test, also known as the active compression test, is commonly used to diagnose SLAP tears. The test involves placing the arm in 90 degrees of forward flexion, 20 degrees of horizontal adduction, and internal rotation (thumb pointing down). The patient is then instructed to forward flex the arm against resistance. The test is repeated with the arm internally rotated and then externally rotated. Pain with internal rotation but not external rotation is a positive test. The O’Brien’s test stresses the labrum by tightening the posterior capsule and posteriorly translating the humeral head. This stress causes pain and weakness during the O’Brien’s test when the superior labrum is torn. MRI arthrogram is the diagnostic study of choice for SLAP tears. The arthrogram dye can be seen underneath and into the superior labral tear in figure 1.
    Answer D
    References
    1. Owen M, Boulter T, Walton M, Funk L, Mackenzie T. Reinterpretation of O’Brien test in posterior labral tears of the shoulder. Int J Shoulder Surg. 2015 Jan-Mar; 9(1): 6–8.
    2. Marx R, Camp C. Labral Tears of the Shoulder. Clinical Summary. http://www.JBJS.org. Retreived on 12/20/18.

  33. Question 33 of 50
    33. Question

    A 19 year-old male presents to your office with a 2 month history of left knee pain. He denies injury or precipitating event. He feels the knee “locking up” at times and giving way. He feels a sharp stabbing pain when the knee gives way. Coronal view MRI image is shown above. What is the most likely diagnosis?

    Correct

    The patient’s MRI findings are consistent with a discoid lateral meniscus tear. MRI findings include an abnormally wide and thick lateral meniscus. Figure 2 shows the classic “bow-tie sign”. A normal meniscus is a crescent shaped cartilaginous structure that acts as a shock absorber to protect the cartilage in the knee. A discoid meniscus is an abnormal development that results in a larger than normal meniscus. Most discoid meniscus are asymptomatic and found incidentally on MRI or during knee arthroscopy. However, the larger meniscus is prone to cause symptoms of pain, clicking, and mechanical locking that is more pronounced in knee extension. Patients with a symptomatic discoid meniscus often start to experience symptoms in adolescence during physical activities. MRI is the study of choice to diagnose a discoid meniscus, which is more prone to tearing than a normal shaped meniscus. Up to 70% of symptomatic discoid meniscus have a tear seen during arthroscopy. Most discoid meniscus can be treated successfully with observation. Symptomatic discoid meniscus are often treated with knee arthroscopy which may include saucerization to create a normal C-shaped meniscus. 1,2
    Answer B.
    References
    1. Discoid Lateral Meniscus. JOPA- Image Quiz: 14 June 2016 – Volume 4 – Issue 2 – p. 13-15
    2. Discoid lateral meniscus. http://www.orthobullets.com. Accessed on 10/30/19

    Incorrect

    The patient’s MRI findings are consistent with a discoid lateral meniscus tear. MRI findings include an abnormally wide and thick lateral meniscus. Figure 2 shows the classic “bow-tie sign”. A normal meniscus is a crescent shaped cartilaginous structure that acts as a shock absorber to protect the cartilage in the knee. A discoid meniscus is an abnormal development that results in a larger than normal meniscus. Most discoid meniscus are asymptomatic and found incidentally on MRI or during knee arthroscopy. However, the larger meniscus is prone to cause symptoms of pain, clicking, and mechanical locking that is more pronounced in knee extension. Patients with a symptomatic discoid meniscus often start to experience symptoms in adolescence during physical activities. MRI is the study of choice to diagnose a discoid meniscus, which is more prone to tearing than a normal shaped meniscus. Up to 70% of symptomatic discoid meniscus have a tear seen during arthroscopy. Most discoid meniscus can be treated successfully with observation. Symptomatic discoid meniscus are often treated with knee arthroscopy which may include saucerization to create a normal C-shaped meniscus. 1,2
    Answer B.
    References
    1. Discoid Lateral Meniscus. JOPA- Image Quiz: 14 June 2016 – Volume 4 – Issue 2 – p. 13-15
    2. Discoid lateral meniscus. http://www.orthobullets.com. Accessed on 10/30/19

  34. Question 34 of 50
    34. Question


    A 34 year-old female presents to the emergency room with left elbow pain and deformity after an injury during a martial arts competition. AP and lateral x-rays of the left elbow show a posterior elbow dislocation. The elbow is reduced and splinted in the ER. Which choice represents the typical treatment for elbow dislocations

    Correct

    The elbow is second to only the shoulder as the most common major joint to dislocate. The injury pattern typically results from a fall on an outstretched hand. The elbow is hyperextended and the forearm dislocates in a posterior lateral direction relative to the humerus. Acute management of an elbow dislocation includes closed reduction and splinting in 90 degrees of elbow flexion to maintain the reduction. The technique for closed reduction involves traction of the forearm, forearm supination, and elbow flexion. Simple dislocations without fracture usually have some degree of osteochondral injury, although surgical treatment is rarely necessary. Neurovascular injury is rare but a detailed exam should be performed post-reduction to rule out this complication. Recurrent instability after simple dislocations of the elbow is rare with an incidence of less than 1%. Treatment includes a brief period of immobilization, usually 5-10 days, to relieve pain and swelling. Immobilization for greater than 3 weeks is associated with chronic elbow stiffness and should be avoided. Early elbow motion can start a week from injury with the goal of slowly regaining full flexion and extension. Surgery is indicated when there are associated unstable fractures around the joint or the elbow fails to stay reduced. 1,2
    Answer C.
    References
    1. Cohen M, Hastings H. Acute Elbow Dislocation: Evaluation and Management. J Am Acad Orthop Surg 1998;6:15-23
    2. Elbow Dislocation. http://www.orthobullets.com. Accessed on 12/19/18.

    Incorrect

    The elbow is second to only the shoulder as the most common major joint to dislocate. The injury pattern typically results from a fall on an outstretched hand. The elbow is hyperextended and the forearm dislocates in a posterior lateral direction relative to the humerus. Acute management of an elbow dislocation includes closed reduction and splinting in 90 degrees of elbow flexion to maintain the reduction. The technique for closed reduction involves traction of the forearm, forearm supination, and elbow flexion. Simple dislocations without fracture usually have some degree of osteochondral injury, although surgical treatment is rarely necessary. Neurovascular injury is rare but a detailed exam should be performed post-reduction to rule out this complication. Recurrent instability after simple dislocations of the elbow is rare with an incidence of less than 1%. Treatment includes a brief period of immobilization, usually 5-10 days, to relieve pain and swelling. Immobilization for greater than 3 weeks is associated with chronic elbow stiffness and should be avoided. Early elbow motion can start a week from injury with the goal of slowly regaining full flexion and extension. Surgery is indicated when there are associated unstable fractures around the joint or the elbow fails to stay reduced. 1,2
    Answer C.
    References
    1. Cohen M, Hastings H. Acute Elbow Dislocation: Evaluation and Management. J Am Acad Orthop Surg 1998;6:15-23
    2. Elbow Dislocation. http://www.orthobullets.com. Accessed on 12/19/18.

  35. Question 35 of 50
    35. Question

    A 54 year-old male presents to the clinic with right shoulder pain after a fall 2 days prior. He fell while skiing and landed directly on the right shoulder. He has had significant pain and weakness since the injury. AP radiograph of the clavicle is shown in figure 1. Which ligament or ligaments have been injured?

    Correct

    Acromioclavicular (AC) joint separation is a common injury resulting in a direct blow to the shoulder. Anterior-posterior AC joint stability is provided by the acromioclavicular (AC) ligaments and superior-inferior stability is provided by the coracoclavicular (CC) ligaments. Injury to the two coracoclavicular ligaments (the medial conoid and the lateral trapezoid) can cause superior migration of the clavicle. The Rockwood classification divides AC joint injuries into 6 types. Type I AC joint injuries are characterized by sprains of the AC ligaments without superior migration of the clavicle. Type II injuries cause tearing of the AC ligaments but not the CC ligaments. Grade III injuries tear both the AC ligaments and CC ligaments causing 25% to 100% superior displacement of the clavicle. Type IV, V, and VI injuries are rare. Type IV injuries result in posterior displacement of the clavicle into the trapezial fascia. Type V injuries are Type III injuries with >100% superior displacement of the clavicle. Type VI injuries displace inferiorly in a subcoracoid position. With type III-V injuries up to 18% of patients will have concomitant pathology such as SLAP lesions, rotator cuff injuries, and fractures. 1,2
    This patient has a grade III/ borderline grade V AC joint separation indicating the acromioclavicular and corococlavicular ligaments have been injured.
    Answer E.
    References
    1. Li X, Ma R, MD; Bedi A, Dines DM, Altchek DW, Dines J. Current Concepts Review: Management Of Acromioclavicular Joint Injuries. JBJS. January 2014 – Volume 96 – Issue 1 – p. 73-84
    2. Spencer EE Jr. Treatment of grade III acromioclavicular joint injuries: a systematic review. Clin Orthop Relat Res, February 2007; 455(1); 38-44.

    Incorrect

    Acromioclavicular (AC) joint separation is a common injury resulting in a direct blow to the shoulder. Anterior-posterior AC joint stability is provided by the acromioclavicular (AC) ligaments and superior-inferior stability is provided by the coracoclavicular (CC) ligaments. Injury to the two coracoclavicular ligaments (the medial conoid and the lateral trapezoid) can cause superior migration of the clavicle. The Rockwood classification divides AC joint injuries into 6 types. Type I AC joint injuries are characterized by sprains of the AC ligaments without superior migration of the clavicle. Type II injuries cause tearing of the AC ligaments but not the CC ligaments. Grade III injuries tear both the AC ligaments and CC ligaments causing 25% to 100% superior displacement of the clavicle. Type IV, V, and VI injuries are rare. Type IV injuries result in posterior displacement of the clavicle into the trapezial fascia. Type V injuries are Type III injuries with >100% superior displacement of the clavicle. Type VI injuries displace inferiorly in a subcoracoid position. With type III-V injuries up to 18% of patients will have concomitant pathology such as SLAP lesions, rotator cuff injuries, and fractures. 1,2
    This patient has a grade III/ borderline grade V AC joint separation indicating the acromioclavicular and corococlavicular ligaments have been injured.
    Answer E.
    References
    1. Li X, Ma R, MD; Bedi A, Dines DM, Altchek DW, Dines J. Current Concepts Review: Management Of Acromioclavicular Joint Injuries. JBJS. January 2014 – Volume 96 – Issue 1 – p. 73-84
    2. Spencer EE Jr. Treatment of grade III acromioclavicular joint injuries: a systematic review. Clin Orthop Relat Res, February 2007; 455(1); 38-44.

  36. Question 36 of 50
    36. Question

    A 14 year girl presents to the office with left posterior buttock pain after a soccer injury 2 days ago. She is having trouble walking now and has pain from the buttock that radiates to the hamstring. AP pelvis shows a left ischial tuberosity avulsion fracture. The avulsion appears to be displaced 1.5 cm. What is the best treatment option?

    Correct

    The ischial tuberosity is an attachment site for the proximal hamstring tendons. The three muscles that make up the hamstring include the semimembranosus, the semitendinosus, and the biceps femoris. The hamstrings act to extend the hip and flex the knee. The hamstrings can be torn proximally, midsubstance, or distal. The most common site of rupture is the proximal myotendinous junction. Avulsion injuries are less common and are generally seen in skeletally immature patients. Hamstring injuries cause a sharp pain in the posterior buttock which can be associated with significant ecchymosis. All patients with suspected hamstring injuries should have an AP pelvis x-ray to look for an avulsion fracture of the ischial tuberosity. In general most all hamstring tendon injuries are treated non-operatively with a period of protective weight-bearing for 4-6 weeks. The hamstrings can tighten quickly after tearing so physical therapy should be initiated after 4 weeks to focus on hamstring stretches. Athletes are allowed to return to sports when the hamstring is 90% as strong as the contralateral side. Operative treatment is indicated when there is three tendons torn of the ischial tuberosity or there is >2 cm of displacement of an osseus avulsion. These patients generally have significant chronic weakness and pain when treated non-operatively. 1,2
    Answer B.
    References
    1. Hofman KJ, Paggi Adam, Conors D, Miller S. Complete Avulsion of the Proximal Hamstring Insertion: Functional Outcomes After Nonsurgical Treatment. The Journal Of Bone And Joint Surgery – Scientific Articles: 18 June 2014 – Volume 96 – Issue 12 – p. 1022-1025
    2. Alzahrani MM, Aldebeyan S, Abduljabbar F, Martineau PA. Hamstring Injuries in Athletes: Diagnosis and Treatment. Jbjs Reviews – Review Article: 30 June 2015 – Volume 3 – Issue 6 – p. e5

    Incorrect

    The ischial tuberosity is an attachment site for the proximal hamstring tendons. The three muscles that make up the hamstring include the semimembranosus, the semitendinosus, and the biceps femoris. The hamstrings act to extend the hip and flex the knee. The hamstrings can be torn proximally, midsubstance, or distal. The most common site of rupture is the proximal myotendinous junction. Avulsion injuries are less common and are generally seen in skeletally immature patients. Hamstring injuries cause a sharp pain in the posterior buttock which can be associated with significant ecchymosis. All patients with suspected hamstring injuries should have an AP pelvis x-ray to look for an avulsion fracture of the ischial tuberosity. In general most all hamstring tendon injuries are treated non-operatively with a period of protective weight-bearing for 4-6 weeks. The hamstrings can tighten quickly after tearing so physical therapy should be initiated after 4 weeks to focus on hamstring stretches. Athletes are allowed to return to sports when the hamstring is 90% as strong as the contralateral side. Operative treatment is indicated when there is three tendons torn of the ischial tuberosity or there is >2 cm of displacement of an osseus avulsion. These patients generally have significant chronic weakness and pain when treated non-operatively. 1,2
    Answer B.
    References
    1. Hofman KJ, Paggi Adam, Conors D, Miller S. Complete Avulsion of the Proximal Hamstring Insertion: Functional Outcomes After Nonsurgical Treatment. The Journal Of Bone And Joint Surgery – Scientific Articles: 18 June 2014 – Volume 96 – Issue 12 – p. 1022-1025
    2. Alzahrani MM, Aldebeyan S, Abduljabbar F, Martineau PA. Hamstring Injuries in Athletes: Diagnosis and Treatment. Jbjs Reviews – Review Article: 30 June 2015 – Volume 3 – Issue 6 – p. e5

  37. Question 37 of 50
    37. Question

    A 35 year old female presents to your office with a 4 month history of left hip pain. The pain is made worse during karate, especially when she is kicking or rotating the hip. She has failed conservative treatment with oral anti-inflammatories, physical therapy, and an intra-articular injection. MRI arthrogram of the left hip shows a tear of the anterior labrum with normal articular cartilage. What is the most likely treatment for this patient?

    Correct

    The labrum is a rim of soft tissue fibrocartilage that lines the periphery of the acetabulum. The labrum provides stability of the hip joint by deepening the hip socket and providing a suction seal. Labral tears are found in a high percentage of asymptomatic patients and therefore most labral tears are a result of degeneration over time. Anatomical variants such as hip dysplasia, pincer lesion (excessive bone on acetabulum, and cam lesions (excessive bone on femoral side) can cause early degeneration and tearing of the hip labrum. Sports with repetitive hip motion can also increase the likelihood of a labral tear. Symptoms that suggest a labral tear include groin related pain, snapping, or locking of the hip. If symptoms and physical exam findings suggest a labral tear, and all conservative treatments have failed, an arthroscopic labral repair is recommended. Patients may elect for a labral debridement for a faster postoperative return to activities. However, labral repair has shown to have improved patient outcomes over labral debridement. Labral debridement may also increase the likelihood of degenerative arthritis in the hip and the need for a total hip replacement. Labral calcification may be an indication for labral debridement alone but capsular-labral junction must remain intact to preserve the blood supply to the labrum. Advanced age and hip arthritis often change treatment in favor of labral debridement. Bottom line, labral tears in a non-arthritic hip should be repaired whenever possible. 1,2
    Answer C.
    References
    1. Domb BG, Hartigan DE, Perets I. Decision Making for Labral Treatment in the Hip: Repair Versus Débridement Versus Reconstruction. J Am Acad Orthop Surg. , March 2017; 25(3); e53-e62
    2. Krych AJ, Thompson M, Knutson Z, Scoon J, Coleman SH. Arthroscopic labral repair versus selective labral debridement in female patients with femoroacetabular impingement: a prospective randomized study.Arthroscopy. , January 2013; 29(1); 46-53

    Incorrect

    The labrum is a rim of soft tissue fibrocartilage that lines the periphery of the acetabulum. The labrum provides stability of the hip joint by deepening the hip socket and providing a suction seal. Labral tears are found in a high percentage of asymptomatic patients and therefore most labral tears are a result of degeneration over time. Anatomical variants such as hip dysplasia, pincer lesion (excessive bone on acetabulum, and cam lesions (excessive bone on femoral side) can cause early degeneration and tearing of the hip labrum. Sports with repetitive hip motion can also increase the likelihood of a labral tear. Symptoms that suggest a labral tear include groin related pain, snapping, or locking of the hip. If symptoms and physical exam findings suggest a labral tear, and all conservative treatments have failed, an arthroscopic labral repair is recommended. Patients may elect for a labral debridement for a faster postoperative return to activities. However, labral repair has shown to have improved patient outcomes over labral debridement. Labral debridement may also increase the likelihood of degenerative arthritis in the hip and the need for a total hip replacement. Labral calcification may be an indication for labral debridement alone but capsular-labral junction must remain intact to preserve the blood supply to the labrum. Advanced age and hip arthritis often change treatment in favor of labral debridement. Bottom line, labral tears in a non-arthritic hip should be repaired whenever possible. 1,2
    Answer C.
    References
    1. Domb BG, Hartigan DE, Perets I. Decision Making for Labral Treatment in the Hip: Repair Versus Débridement Versus Reconstruction. J Am Acad Orthop Surg. , March 2017; 25(3); e53-e62
    2. Krych AJ, Thompson M, Knutson Z, Scoon J, Coleman SH. Arthroscopic labral repair versus selective labral debridement in female patients with femoroacetabular impingement: a prospective randomized study.Arthroscopy. , January 2013; 29(1); 46-53

  38. Question 38 of 50
    38. Question


    A 56 year-old male presents to the office with left shoulder pain for several months. He notices the left shoulder isn’t as strong as the right shoulder but denies previous injury. AP x-ray of the left shoulder is shown in figure 1. What is the most likely cause of this patient chronic shoulder pain?

    Correct

    The patient has a decreased acromiohumeral interval noted on AP x-ray of the shoulder. This distance can be measured from the undersurface of the acromion to the superior portion of the humerus. An increased distance suggests inferior subluxation which can result from a shoulder dislocation or proximal humerus fracture. A decreased distance suggests a rotator cuff tear. An intact rotator cuff places an inferiorly directed force to the humeral head relative to its position on the glenoid. A balance of an intact supraspinatus, infraspinatus, teres minor, and subscapularis tendons allows for concentric rotation of humeral head in the glenoid. A massive rotator cuff tear causes superior humeral head migration from the unopposed contraction of the deltoid. A rotator cuff tear also causes loss of negative pressure in the glenohumeral joint and escape of normal synovial fluid out of the joint that is important for healthy articular cartilage. With chronic massive rotator cuff tearing the acromion can often articulate with the humeral head. A decreasing acromiohumeral interval on serial x-rays is a sign of progressive rotator cuff disease. A decreased acromiohumeral interval is the start of the rotator cuff arthropathy cascade where articular cartilage is gradually lost and arthritis becomes evident on x-ray. A loss of dynamic stabilization of the shoulder causes repetitive trauma to the articular cartilage resulting in arthritis. 1,2
    Answer B.
    References
    1. Nam DN, Maak TG, Raphael BS, Kepler CK, Cross MB, Warren RF. Rotator Cuff Tear Arthropathy: Evaluation, Diagnosis, And Treatment. The Journal Of Bone & Joint Surgery – Scientific Articles: 21 March 2012 – Volume 94 – Issue 6 – p. e34.
    2. Ecklund, Kier J.; Lee, Thay Q.; Tibone J. Rotator Cuff Tear Arthropathy. JAAOS – Journal of the American Academy of Orthopaedic Surgeons. 15(6):340-349, June 2007.

    Incorrect

    The patient has a decreased acromiohumeral interval noted on AP x-ray of the shoulder. This distance can be measured from the undersurface of the acromion to the superior portion of the humerus. An increased distance suggests inferior subluxation which can result from a shoulder dislocation or proximal humerus fracture. A decreased distance suggests a rotator cuff tear. An intact rotator cuff places an inferiorly directed force to the humeral head relative to its position on the glenoid. A balance of an intact supraspinatus, infraspinatus, teres minor, and subscapularis tendons allows for concentric rotation of humeral head in the glenoid. A massive rotator cuff tear causes superior humeral head migration from the unopposed contraction of the deltoid. A rotator cuff tear also causes loss of negative pressure in the glenohumeral joint and escape of normal synovial fluid out of the joint that is important for healthy articular cartilage. With chronic massive rotator cuff tearing the acromion can often articulate with the humeral head. A decreasing acromiohumeral interval on serial x-rays is a sign of progressive rotator cuff disease. A decreased acromiohumeral interval is the start of the rotator cuff arthropathy cascade where articular cartilage is gradually lost and arthritis becomes evident on x-ray. A loss of dynamic stabilization of the shoulder causes repetitive trauma to the articular cartilage resulting in arthritis. 1,2
    Answer B.
    References
    1. Nam DN, Maak TG, Raphael BS, Kepler CK, Cross MB, Warren RF. Rotator Cuff Tear Arthropathy: Evaluation, Diagnosis, And Treatment. The Journal Of Bone & Joint Surgery – Scientific Articles: 21 March 2012 – Volume 94 – Issue 6 – p. e34.
    2. Ecklund, Kier J.; Lee, Thay Q.; Tibone J. Rotator Cuff Tear Arthropathy. JAAOS – Journal of the American Academy of Orthopaedic Surgeons. 15(6):340-349, June 2007.

  39. Question 39 of 50
    39. Question

    A 24 year-old male presents to the office with chronic right shoulder pain and weakness after falling off his mountain bike two years ago. He was found to have a grade III acromioclavicular (AC) joint separation and was treated conservatively but he has never regained full shoulder function. On exam in the office you note an obvious deformity to the shoulder as the distal clavicle is seen tenting the skin. He also complains of weakness and pain with shoulder motion. A coronal MRI image shows a chronic grade III AC joint separation. He works as an accountant and enjoys recreation sports. Which choice below is a benefit of surgical reconstruction of the AC joint?

    Correct

    A grade III acromioclavicular joint separation is characterized by complete disruption of the acromioclavicular and corococlavicular ligaments (coronoid and trapezoid ligaments). The injury is often obvious with 25 to 100% displacement of the distal clavicle which can be seen tenting the skin. The acromioclavicular ligaments provide support with horizontal shoulder motion and anterior-posterior stability. The corococlavicular ligaments provide restraint against cephalad migration of the clavicle. Despite the obvious deformity of a grade III AC joint separation most can be treated conservatively without surgery. In general most patients will have good functional recovery without loss of strength or motion long term. Competitive athletes and heavy laborers have also been shown to have comparable results with and without surgery. Outcomes of non-surgical treatment largely depend on participation in physical therapy to regain strength and motion. Surgical reconstruction may offer high level throwing athletes some benefit as these patients were found to have altered throwing motion without surgery. Loss of AC stability is thought to decrease the stability of the scapula and subsequently weaken the shoulder girdle. With that said, the current literature overwhelmingly supports non-operative treatment in most, if not all patients, with a grade III AC joint separation. 1,2
    Answer D.
    References
    1. Virk M, Apostolakos J, Cote M, Baker B, Beitzel K, Mazzocca A, . Operative And Nonoperative Treatment Of Acromioclavicular Dislocation. JBJS Reviews, 27 October 2015; 3(10); e5.
    2. AC Seperation. http://www.orthobullets.com. Accessed on 9/21/18.

    Incorrect

    A grade III acromioclavicular joint separation is characterized by complete disruption of the acromioclavicular and corococlavicular ligaments (coronoid and trapezoid ligaments). The injury is often obvious with 25 to 100% displacement of the distal clavicle which can be seen tenting the skin. The acromioclavicular ligaments provide support with horizontal shoulder motion and anterior-posterior stability. The corococlavicular ligaments provide restraint against cephalad migration of the clavicle. Despite the obvious deformity of a grade III AC joint separation most can be treated conservatively without surgery. In general most patients will have good functional recovery without loss of strength or motion long term. Competitive athletes and heavy laborers have also been shown to have comparable results with and without surgery. Outcomes of non-surgical treatment largely depend on participation in physical therapy to regain strength and motion. Surgical reconstruction may offer high level throwing athletes some benefit as these patients were found to have altered throwing motion without surgery. Loss of AC stability is thought to decrease the stability of the scapula and subsequently weaken the shoulder girdle. With that said, the current literature overwhelmingly supports non-operative treatment in most, if not all patients, with a grade III AC joint separation. 1,2
    Answer D.
    References
    1. Virk M, Apostolakos J, Cote M, Baker B, Beitzel K, Mazzocca A, . Operative And Nonoperative Treatment Of Acromioclavicular Dislocation. JBJS Reviews, 27 October 2015; 3(10); e5.
    2. AC Seperation. http://www.orthobullets.com. Accessed on 9/21/18.

  40. Question 40 of 50
    40. Question


    A 33 year-old male presents to the office with chronic right hip pain from an injury 10 months ago. He was playing soccer at the time of injury when he went to kick a ball but was held up suddenly. When he held up to stop the forward movement of his leg he felt a pop, and he points to the hip as to where that pop occurred. Since that time he has not been able to kick a soccer ball, but activities of daily living do not bother him. On exam he has pain with resisted forward flexion of the right hip. AP pelvis and oblique view x-rays of the right hip are shown in figures 1 and 2. Which choice is the likely cause of this avulsion fracture?

    Correct

    The pelvis provides attachment sites for many of the muscles that flex, extend, and rotate the hip. Musculotendinous attachments can pull bone off these sites, or cause avulsion fractures, with high energy injuries. The most common sites of avulsion fractures in the pelvis include the ischial tuberosity, the anterior inferior iliac spine (AIIS), and the anterior superior iliac spine (ASIS). The AIIS is the attachment site for the rectus femoris which flexes the hip. The ASIS is the attachment site for the Sartorius and tensor fasciae latae which flex and medially rotate the hip, respectively. The hamstrings (biceps femoris, semitendinosus, and semimembranosus) attach to the ischial tuberosity and extend the hip. 1,2

    The AIIS is the most common site for all avulsion fractures of the pelvis. The AIIS apophysis fuses at age 16 years old on average and epiphyseal avulsion fractures typically occur between the ages of 14 and 17. As with this case of a 33 year-old patient, the AIIS can also avulse when the apophysis is fused. Avulsion injuries typically occur with sporting activities such as running, jumping, or kicking sports. Patients often recall a sudden “pop” or snapping sound followed by pain and weakness. The initial treatment is non-operative with a brief period of non-weight bearing and crutches. Physical therapy and weight bearing are initiated 1-2 weeks after the injury or when symptoms allow. Patients can often return to sport 2 months after avulsion fractures. 1,2

    Answer D.

    References

    1. Schiller J, Defroda S, Blood T. Lower Extremity Avulsion Fractures in the Pediatric and Adolescent Athlete. JAAOS 2017; 25: 251-259.

    2. AIIS Avulsion Fracture. http://www.orthobullets.com. Accessed on 5/13/2018.

     

    Incorrect

    The pelvis provides attachment sites for many of the muscles that flex, extend, and rotate the hip. Musculotendinous attachments can pull bone off these sites, or cause avulsion fractures, with high energy injuries. The most common sites of avulsion fractures in the pelvis include the ischial tuberosity, the anterior inferior iliac spine (AIIS), and the anterior superior iliac spine (ASIS). The AIIS is the attachment site for the rectus femoris which flexes the hip. The ASIS is the attachment site for the Sartorius and tensor fasciae latae which flex and medially rotate the hip, respectively. The hamstrings (biceps femoris, semitendinosus, and semimembranosus) attach to the ischial tuberosity and extend the hip. 1,2

    The AIIS is the most common site for all avulsion fractures of the pelvis. The AIIS apophysis fuses at age 16 years old on average and epiphyseal avulsion fractures typically occur between the ages of 14 and 17. As with this case of a 33 year-old patient, the AIIS can also avulse when the apophysis is fused. Avulsion injuries typically occur with sporting activities such as running, jumping, or kicking sports. Patients often recall a sudden “pop” or snapping sound followed by pain and weakness. The initial treatment is non-operative with a brief period of non-weight bearing and crutches. Physical therapy and weight bearing are initiated 1-2 weeks after the injury or when symptoms allow. Patients can often return to sport 2 months after avulsion fractures. 1,2

    Answer D.

    References

    1. Schiller J, Defroda S, Blood T. Lower Extremity Avulsion Fractures in the Pediatric and Adolescent Athlete. JAAOS 2017; 25: 251-259.

    2. AIIS Avulsion Fracture. http://www.orthobullets.com. Accessed on 5/13/2018.

     

  41. Question 41 of 50
    41. Question


    A 45 year-old female presents to the emergency department with right knee pain and deformity after falling down the stairs. A sunrise view x-ray (figure 1) shows a lateral patella dislocation. Which structure is most likely injured?

    Correct

    The medial patellofemoral ligament (MPFL) attaches the medial patella to the medial femur and is the primary restraint against lateral dislocation of the patella. Cadaveric studies show that the MPFL provides 50-60% of the total restraining force against lateral dislocation. Other structures that provide a lesser role in preventing lateral instability include the medial retinaculum and the medial patellotibial ligament. Patella dislocations generally occur with the knee in extension and the leg externally rotated. Risk factors for patella instability include patella alta, trochlea dysplasia, increased Q angle, and ligamentous laxity. 1,2
    The incidence of MPFL injury after patella dislocation varies widely in the literature from 40% to 90%. MRI is the study of choice to diagnose an MPFL injury. Other injuries that may occur with patella dislocations include osteochondral injuries, meniscus tears, and MCL tears. Osteochondral injuries of the patellofemoral joint and lateral femoral condyle occur in up to 40% of lateral patella dislocations. This correlates closely with studies showing a 35% incidence of patellofemoral arthritis 13 years after patellofemoral dislocation. 1,2

    References
    1. Shin-Jae Rhee, George Pavlou, Jeremy Oakley, David Barlow, and Farres Haddad. Modern management of patellar instability. Int Orthop. 2012 Dec; 36(12): 2447–2456.
    2. Sillanp PJ, Mattila VM, HeikkiM, Kiuru M, Visuri T, Pihlajam H. Treatment with and without Initial Stabilizing Surgery for Primary Traumatic Patellar Dislocation. JBJS 2009; 91: 263-273.

    Incorrect

    The medial patellofemoral ligament (MPFL) attaches the medial patella to the medial femur and is the primary restraint against lateral dislocation of the patella. Cadaveric studies show that the MPFL provides 50-60% of the total restraining force against lateral dislocation. Other structures that provide a lesser role in preventing lateral instability include the medial retinaculum and the medial patellotibial ligament. Patella dislocations generally occur with the knee in extension and the leg externally rotated. Risk factors for patella instability include patella alta, trochlea dysplasia, increased Q angle, and ligamentous laxity. 1,2
    The incidence of MPFL injury after patella dislocation varies widely in the literature from 40% to 90%. MRI is the study of choice to diagnose an MPFL injury. Other injuries that may occur with patella dislocations include osteochondral injuries, meniscus tears, and MCL tears. Osteochondral injuries of the patellofemoral joint and lateral femoral condyle occur in up to 40% of lateral patella dislocations. This correlates closely with studies showing a 35% incidence of patellofemoral arthritis 13 years after patellofemoral dislocation. 1,2

    References
    1. Shin-Jae Rhee, George Pavlou, Jeremy Oakley, David Barlow, and Farres Haddad. Modern management of patellar instability. Int Orthop. 2012 Dec; 36(12): 2447–2456.
    2. Sillanp PJ, Mattila VM, HeikkiM, Kiuru M, Visuri T, Pihlajam H. Treatment with and without Initial Stabilizing Surgery for Primary Traumatic Patellar Dislocation. JBJS 2009; 91: 263-273.

  42. Question 42 of 50
    42. Question


    A 42 year-old female presents to the office with a few months of right knee pain. The pain is worse when going up and down stairs. She also complains of start-up knee pain after sitting for a long period of time. On exam she has anterior knee pain over the lateral patellofemoral facet joint. She is noted to have lateral patella tilt on sunrise view x-ray (figure 1). Which choice below does not contribute to lateral tilt of the patella?

    Correct

    Patellofemoral pain is a common complaint associated with many ADL’s including sitting, going up and down stairs, kneeling, squatting. The most common cause of patellofemoral pain is lateral patella maltracking. Lateral maltracking may be due to several etiologies including vastus medialis weakness, hamstring tightness, lateral retinacula tightness, iliotibial band tightness, lower extremity malalignment, and trauma.
    The vastus medialis oblique (VMO) helps keep the patella medial in the femoral trochlea during knee motion. A weak VMO can cause lateral maltracking. Tight hamstrings increase knee flexion and may prevent terminal extension during gait. This puts more strain and lateral force to the patella. Hamstring stretching and VMO strengthening are the primary focus of physical therapy for rehabbing lateral patella femoral compression syndrome. Lateral retinacula tightness can also pull the patella laterally which can contribute to maltracking. Lateral retinacula release was a common surgical procedure performed in years past but recent data shows this to be ineffective for addressing patella femoral pain. Lower extremity deformities causing lateral maltracking of the patella include increased Q ankle, genu valgum, external tibial torsion, and excessive foot pronation. Trauma such as a patella dislocation can tear the medial supporting structures to the patella, including the medial patella femoral ligament (MPFL), which often results in chronic lateral patella instability. 1, 2
    References
    1. Douciette SA, Goble M. The effect of exercise on patella tracking for lateral patella compression syndrome. American Journal of Sports Medcine. 1992. Volume 24 (4). 435-440
    2. Larsen B, Andreason E. Patella Taping: A radiographic exam of the medial glide technique. Americal Journal of Sports Medicine. 1995 Volume 23. 465-471.

    Incorrect

    Patellofemoral pain is a common complaint associated with many ADL’s including sitting, going up and down stairs, kneeling, squatting. The most common cause of patellofemoral pain is lateral patella maltracking. Lateral maltracking may be due to several etiologies including vastus medialis weakness, hamstring tightness, lateral retinacula tightness, iliotibial band tightness, lower extremity malalignment, and trauma.
    The vastus medialis oblique (VMO) helps keep the patella medial in the femoral trochlea during knee motion. A weak VMO can cause lateral maltracking. Tight hamstrings increase knee flexion and may prevent terminal extension during gait. This puts more strain and lateral force to the patella. Hamstring stretching and VMO strengthening are the primary focus of physical therapy for rehabbing lateral patella femoral compression syndrome. Lateral retinacula tightness can also pull the patella laterally which can contribute to maltracking. Lateral retinacula release was a common surgical procedure performed in years past but recent data shows this to be ineffective for addressing patella femoral pain. Lower extremity deformities causing lateral maltracking of the patella include increased Q ankle, genu valgum, external tibial torsion, and excessive foot pronation. Trauma such as a patella dislocation can tear the medial supporting structures to the patella, including the medial patella femoral ligament (MPFL), which often results in chronic lateral patella instability. 1, 2
    References
    1. Douciette SA, Goble M. The effect of exercise on patella tracking for lateral patella compression syndrome. American Journal of Sports Medcine. 1992. Volume 24 (4). 435-440
    2. Larsen B, Andreason E. Patella Taping: A radiographic exam of the medial glide technique. Americal Journal of Sports Medicine. 1995 Volume 23. 465-471.

  43. Question 43 of 50
    43. Question

    A 23 year-old female presents to the office with left knee pain after falling while skiing. She has mild weight bearing pain and the knee feels unstable. On exam she has full passive and active range of motion and no effusion. She has grade II laxity with valgus stress to the knee. She has no instability with varus stress or with Lachman test. She has no tenderness over the medial or lateral joint line. She is able to straight leg raise against resistance. AP and lateral x-rays are negative. What is the next best step in treatment?

    Correct

    The patient has obvious valgus instability to the knee consistent with an MCL sprain. Her x-rays show no evidence of a fracture which would limit her weight bearing during treatment. She has no effusion or joint line tenderness that would indicate a possible meniscal tear or other intra-articular pathology. MRI is often ordered if associated injuries are suspected but based on her clinical exam she appears to have an isolated MCL injury. For this reason an MRI is not necessary. An MCL tear can be diagnosed on clinical exam alone. The patient should be treated with a hinged knee brace with weight bearing as tolerated. 1,2
    The MCL has an abundant blood supply that allows the torn tendon ends to heal together. Treatment with a hinged knee brace protects the knee from further valgus stress while allowing for knee motion. Early knee motion in a hinged knee brace helps enhance healing and improve biomechanical properties of the tendon. Weight bearing in a hinged knee brace doesn’t stress the MCL and should be encouraged early. A typical rehab protocol includes a hinged knee brace with physical therapy for continued strengthening of the knee. The brace is usually worn for 3-4 weeks for grade I injuries and 6 weeks for grade II and III injuries. A brace is used until the patient is able to perform activities without pain or instability. The MCL is also tested with valgus stress and any persistent laxity should be braced and followed closely before the brace is removed. 1,2

    References
    1. Wijdicks CA, Griffith CJ, Johansen S, Engebretsen L, Laprade RF. Injuries To The Medial Collateral Ligament And Associated Medial Structures Of The Knee. JBJS – Current Concepts Review: May 2010 – Volume 92 – Issue 5 – p. 1266-1280
    2. Miyamoto, Ryan G.; Bosco, Joseph A.; Sherman, Orrin H.Treatment of Medial Collateral Ligament Injuries. JAAOS. 2009. 17(3):152-161.

    Incorrect

    The patient has obvious valgus instability to the knee consistent with an MCL sprain. Her x-rays show no evidence of a fracture which would limit her weight bearing during treatment. She has no effusion or joint line tenderness that would indicate a possible meniscal tear or other intra-articular pathology. MRI is often ordered if associated injuries are suspected but based on her clinical exam she appears to have an isolated MCL injury. For this reason an MRI is not necessary. An MCL tear can be diagnosed on clinical exam alone. The patient should be treated with a hinged knee brace with weight bearing as tolerated. 1,2
    The MCL has an abundant blood supply that allows the torn tendon ends to heal together. Treatment with a hinged knee brace protects the knee from further valgus stress while allowing for knee motion. Early knee motion in a hinged knee brace helps enhance healing and improve biomechanical properties of the tendon. Weight bearing in a hinged knee brace doesn’t stress the MCL and should be encouraged early. A typical rehab protocol includes a hinged knee brace with physical therapy for continued strengthening of the knee. The brace is usually worn for 3-4 weeks for grade I injuries and 6 weeks for grade II and III injuries. A brace is used until the patient is able to perform activities without pain or instability. The MCL is also tested with valgus stress and any persistent laxity should be braced and followed closely before the brace is removed. 1,2

    References
    1. Wijdicks CA, Griffith CJ, Johansen S, Engebretsen L, Laprade RF. Injuries To The Medial Collateral Ligament And Associated Medial Structures Of The Knee. JBJS – Current Concepts Review: May 2010 – Volume 92 – Issue 5 – p. 1266-1280
    2. Miyamoto, Ryan G.; Bosco, Joseph A.; Sherman, Orrin H.Treatment of Medial Collateral Ligament Injuries. JAAOS. 2009. 17(3):152-161.

  44. Question 44 of 50
    44. Question


    A 26-year-old male presents to the office with recurrent instability of the right patella. He has had two dislocations in the past year during athletic activities. These were closed reduced in the emergency room on both occasions. He has failed conservative treatment with physical therapy and bracing. Superimposed axial CT images of the femoral trochlea and tibial tubercle are shown in figure 1. The vertical white lines in the image show the tibial tubercle- trochlea groove (TT-TG) distance. Which statement is true regarding the TT-TG distance?

    Correct

    Patella instability can be due to an incompetent medial patellofemoral ligament (MPFL), trochlea dysplasia, patella alta, vastus medialis insufficiency, and a lateralized tibial tubercle. A tibial tubercle- trochlea groove (TT-TG) measurement is an important tool to guide treatment of patellofemoral instability. A TT-TG distance is measured by superimposing axial CT or MRI images of the femoral trochlea and tibial tubercle. Lines are then drawn vertically through shallowest part of the trochlea and the most anterior part of the tibial tubercle. A lateralized tibial tubercle pulls the patella laterally in the trochlea resulting in an increased likelihood of a lateral patella dislocation. A TT-TG distance is significantly greater in patients with recurrent dislocations. A TT-TG distance of >20mm is widely used as an indication for surgical medialization of the tibial tubercle. A distance between 15mm and 20mm is considered a relative indication. The goal of an anterior medialization “AMZ” procedure is to move the tibial tubercle medially to reduce the TT-TG ratio to between 10-15mm. An AMZ procedure involves an osteotomy of the tibia tubercle which is shifted medially and fixed with two screws directed anterior to posterior. The TT-TG ratio doesn’t change with age and an AMZ is generally performed when patients reach skeletal maturity 1,2

    Answer C.

    References
    1. Dickens AJ, Morrell NT, Doeing A, Tandberg D, Treme G. Tibial Tubercle-Trochlear Groove Distance: Defining Normal in a Pediatric Population. The Journal of Bone & Joint Surgery – Scientific Articles: 19 February 2014 – Volume 96 – Issue 4 – p. 318-324
    2. Tensho K et al. Lateralization of the Tibial Tubercle in Recurrent Patellar Dislocation. The Journal of Bone and Joint Surgery – Scientific Articles: 02 May 2018 – Volume 100 – Issue 9 – p. e58

    Incorrect

    Patella instability can be due to an incompetent medial patellofemoral ligament (MPFL), trochlea dysplasia, patella alta, vastus medialis insufficiency, and a lateralized tibial tubercle. A tibial tubercle- trochlea groove (TT-TG) measurement is an important tool to guide treatment of patellofemoral instability. A TT-TG distance is measured by superimposing axial CT or MRI images of the femoral trochlea and tibial tubercle. Lines are then drawn vertically through shallowest part of the trochlea and the most anterior part of the tibial tubercle. A lateralized tibial tubercle pulls the patella laterally in the trochlea resulting in an increased likelihood of a lateral patella dislocation. A TT-TG distance is significantly greater in patients with recurrent dislocations. A TT-TG distance of >20mm is widely used as an indication for surgical medialization of the tibial tubercle. A distance between 15mm and 20mm is considered a relative indication. The goal of an anterior medialization “AMZ” procedure is to move the tibial tubercle medially to reduce the TT-TG ratio to between 10-15mm. An AMZ procedure involves an osteotomy of the tibia tubercle which is shifted medially and fixed with two screws directed anterior to posterior. The TT-TG ratio doesn’t change with age and an AMZ is generally performed when patients reach skeletal maturity 1,2

    Answer C.

    References
    1. Dickens AJ, Morrell NT, Doeing A, Tandberg D, Treme G. Tibial Tubercle-Trochlear Groove Distance: Defining Normal in a Pediatric Population. The Journal of Bone & Joint Surgery – Scientific Articles: 19 February 2014 – Volume 96 – Issue 4 – p. 318-324
    2. Tensho K et al. Lateralization of the Tibial Tubercle in Recurrent Patellar Dislocation. The Journal of Bone and Joint Surgery – Scientific Articles: 02 May 2018 – Volume 100 – Issue 9 – p. e58

  45. Question 45 of 50
    45. Question

    A 17-year-old female presents to the office with right great toe pain for 3 months. She is an avid basketball player and the pain seemed to start when she “jammed” her toe. The pain is now along the base of the great toe and she has sharp pain with any type of running. On physical exam she has tenderness over the medial sesamoid. AP x-ray shows what appears to be a bipartite tibial sesamoid. She has tried three months of treatment with a walking boot and sesamoid pads but the pain keeps returning. MRI of the foot reveals edema of the medial sesamoid indicating a possible fracture nonunion or avascular necrosis. She is frustrated and wants to return to sports. Which option would offer the patient an earlier return to sports?

    Correct

    The great toe has two sesamoid bones embedded in the flexor halluces brevis (FHB) tendon. The FHB tendon divides in half, with a tendon slip correlating with the medial (tibial) sesamoid and the lateral (fibular) sesamoid. The sesamoid/tendon complex helps absorb weight bearing stress, provides a fulcrum to increase metatarsophalangeal (MTP) flexion power, and reduces friction over the MTP joint. The medial sesamoid is the larger of the two sesamoids and more prone to injury due to increased weight bearing stresses. Diagnosing the exact etiology of sesamoid pain can be difficult. Pain can stem from a bipartite patella, sesamoid fracture, sesamoid avascular necrosis, and FHB tendinitis. MRI is the best test to differentiate these causes of sesamoid pain. If conservative treatments fail, the most reliable surgical treatment for an early return to activities and sports includes a sesamoidectomy. Athletes may notice loss of great toe push off strength after surgery, with a 16% loss with one sesamoid excised and a 30% loss when both are removed. Athletes may start sport activities around 7-8 weeks after surgery. 1,2
    Answer D.
    References
    1. Bartosiak K, McCormick JJ. Avascular Necrosis of the Sesamoids. Foot Ankle Clin. 2019 Mar;24(1):57-67.
    2. Cohen BE. Hallux sesamoid disorders. Foot Ankle Clin. 2009 Mar;14(1):91-104.

    Incorrect

    The great toe has two sesamoid bones embedded in the flexor halluces brevis (FHB) tendon. The FHB tendon divides in half, with a tendon slip correlating with the medial (tibial) sesamoid and the lateral (fibular) sesamoid. The sesamoid/tendon complex helps absorb weight bearing stress, provides a fulcrum to increase metatarsophalangeal (MTP) flexion power, and reduces friction over the MTP joint. The medial sesamoid is the larger of the two sesamoids and more prone to injury due to increased weight bearing stresses. Diagnosing the exact etiology of sesamoid pain can be difficult. Pain can stem from a bipartite patella, sesamoid fracture, sesamoid avascular necrosis, and FHB tendinitis. MRI is the best test to differentiate these causes of sesamoid pain. If conservative treatments fail, the most reliable surgical treatment for an early return to activities and sports includes a sesamoidectomy. Athletes may notice loss of great toe push off strength after surgery, with a 16% loss with one sesamoid excised and a 30% loss when both are removed. Athletes may start sport activities around 7-8 weeks after surgery. 1,2
    Answer D.
    References
    1. Bartosiak K, McCormick JJ. Avascular Necrosis of the Sesamoids. Foot Ankle Clin. 2019 Mar;24(1):57-67.
    2. Cohen BE. Hallux sesamoid disorders. Foot Ankle Clin. 2009 Mar;14(1):91-104.

  46. Question 46 of 50
    46. Question


    A 36-year-old female presents to the office with left knee pain and inability to bear weight for 8 weeks. She injured the knee after falling during a hike in Mexico. She returned home two weeks after the injury and was diagnosed with a knee sprain at a local urgent care. She was placed in a knee immobilizer and has been walking on the leg since the injury with only mild pain. She did not seek further treatment until 8 weeks after the injury as she hasn’t been able to walk without the immobilizer on. On physical exam she is unable to straight leg raise on the left side. AP and lateral x-ray of the left knee (figure 1) shows severe patella alta. What is the most likely treatment necessary for this patient?

    Correct

    The patient has severe patella alta or a high riding patella which indicates a patella tendon ruptures has occurred. Patella baja or a low riding patella indicates a quadriceps tendon rupture has occurred. Repair of patella tendon ruptures must be performed in a timely manner as the patella retracts proximally in as little as two weeks. Acute tears can be treated with a tendon to tendon repair or by suturing the torn tendon to the inferior pole of the patella. Neglected or chronic ruptures present after 6 weeks from injury due to neglect or a missed diagnosis. As time from injury passes the patella and surrounding tissue retracts proximally which makes reducing the patella tendon back down to the tibia increasingly difficult. During open surgical repair, if the gap between the tissue ends is less than 2 cm a distal release of the quadriceps tendon or proximal transposition of the tibial tubercle can be used. If the gap is larger a graft must be used to bridge the tissue ends. Ipsilateral hamstring gracilis and semitendinosus tendon grafts (the same tendons often used for an ACL autograft) are commonly used. Studies have shown that a patella tendon reconstruction with soft tissue autograft provides a good functional recovery and a return to pre-injury level of function. 1,2
    Answer C.
    References
    1. Nicola M, Angelo DB, Loppini M, Vincenzo D. Ipsilateral Hamstring Tendon Graft Reconstruction for Chronic Patellar Tendon Rupture. The Journal of Bone & Joint Surgery – Scientific Articles: 04 September 2013 – Volume 95 – Issue 17 – p. e123
    2. Sundararajan SR, Srikanth KP, Rajasekaran S. Neglected patellar tendon ruptures: a simple modified reconstruction using hamstrings tendon graft. Int Orthop. 2013;37(11):2159-2164.

    Incorrect

    The patient has severe patella alta or a high riding patella which indicates a patella tendon ruptures has occurred. Patella baja or a low riding patella indicates a quadriceps tendon rupture has occurred. Repair of patella tendon ruptures must be performed in a timely manner as the patella retracts proximally in as little as two weeks. Acute tears can be treated with a tendon to tendon repair or by suturing the torn tendon to the inferior pole of the patella. Neglected or chronic ruptures present after 6 weeks from injury due to neglect or a missed diagnosis. As time from injury passes the patella and surrounding tissue retracts proximally which makes reducing the patella tendon back down to the tibia increasingly difficult. During open surgical repair, if the gap between the tissue ends is less than 2 cm a distal release of the quadriceps tendon or proximal transposition of the tibial tubercle can be used. If the gap is larger a graft must be used to bridge the tissue ends. Ipsilateral hamstring gracilis and semitendinosus tendon grafts (the same tendons often used for an ACL autograft) are commonly used. Studies have shown that a patella tendon reconstruction with soft tissue autograft provides a good functional recovery and a return to pre-injury level of function. 1,2
    Answer C.
    References
    1. Nicola M, Angelo DB, Loppini M, Vincenzo D. Ipsilateral Hamstring Tendon Graft Reconstruction for Chronic Patellar Tendon Rupture. The Journal of Bone & Joint Surgery – Scientific Articles: 04 September 2013 – Volume 95 – Issue 17 – p. e123
    2. Sundararajan SR, Srikanth KP, Rajasekaran S. Neglected patellar tendon ruptures: a simple modified reconstruction using hamstrings tendon graft. Int Orthop. 2013;37(11):2159-2164.

  47. Question 47 of 50
    47. Question

    A 66-year-old male presents to the office with right shoulder pain for 6 months. He denies having an injury or known precipitating event. He has had weakness in the shoulder for several years which was not associated with pain until 6 months ago. He has failed conservative treatments with oral anti-inflammatories and shoulder injections. On physical exam he has significant weakness with forward flexion of the arm in the scapular plane. AP x-ray of the shoulder shows a decreased glenohumeral interval without evidence of shoulder arthritis. Coronal MRI shows a massive, irreparable rotator cuff tear with significant retraction involving the supraspinatus and infraspinatus. Which treatment option would be the best for this patient?

    Correct

    The rotator cuff functions to keep the humeral head centered on the glenoid during shoulder motion. As the deltoid contracts to lift the arm the rotator cuff resists superior migration of the humeral head. Massive chronic tears of the rotator cuff may result in the humeral head migrating superiorly as seen on AP x-ray as a decreased acromiohumeral interval. Rotator cuff arthropathy is the condition in which the rotator cuff is torn, retracted, and no longer functional. The treatment of rotator cuff arthropathy is very challenging, particularly in younger, active adults. The treatment of choice for most patients with severe rotator cuff arthropathy and glenohumeral arthritis who have failed conservative treatments is a reverse total shoulder replacement. A superior capsular reconstruction (SCR) is a procedure utilized for patients with mild rotator cuff arthropathy and no glenohumeral arthritis. An SCR technique first involves placing suture anchors in the superior glenoid to tie an acellular dermal graft down to the glenoid. Suture anchors are then placed in the rotator cuff footprint to tie the other side of the graft down on the proximal humerus. The purpose of the graft is to resist superior migration of the humeral head. Early studies have shown SCR to provide relief of pain and improved shoulder function in the short term (the first 2-3 years postoperatively). However, long term studies are lacking as the procedure is relatively new. 1,2
    Answer C.
    References
    1. Rachel F, Cvetanovich G, Savin D, Romeo AA Superior Capsular Reconstruction. JBJS Reviews – Review Article: 31 July 2018 – Volume 6 – Issue 7 – p. e10-e10
    2. Adams, Christopher R., et al. The arthroscopic superior capsular reconstruction. Am J Orthop 45.5 (2016): 320-4.

    Incorrect

    The rotator cuff functions to keep the humeral head centered on the glenoid during shoulder motion. As the deltoid contracts to lift the arm the rotator cuff resists superior migration of the humeral head. Massive chronic tears of the rotator cuff may result in the humeral head migrating superiorly as seen on AP x-ray as a decreased acromiohumeral interval. Rotator cuff arthropathy is the condition in which the rotator cuff is torn, retracted, and no longer functional. The treatment of rotator cuff arthropathy is very challenging, particularly in younger, active adults. The treatment of choice for most patients with severe rotator cuff arthropathy and glenohumeral arthritis who have failed conservative treatments is a reverse total shoulder replacement. A superior capsular reconstruction (SCR) is a procedure utilized for patients with mild rotator cuff arthropathy and no glenohumeral arthritis. An SCR technique first involves placing suture anchors in the superior glenoid to tie an acellular dermal graft down to the glenoid. Suture anchors are then placed in the rotator cuff footprint to tie the other side of the graft down on the proximal humerus. The purpose of the graft is to resist superior migration of the humeral head. Early studies have shown SCR to provide relief of pain and improved shoulder function in the short term (the first 2-3 years postoperatively). However, long term studies are lacking as the procedure is relatively new. 1,2
    Answer C.
    References
    1. Rachel F, Cvetanovich G, Savin D, Romeo AA Superior Capsular Reconstruction. JBJS Reviews – Review Article: 31 July 2018 – Volume 6 – Issue 7 – p. e10-e10
    2. Adams, Christopher R., et al. The arthroscopic superior capsular reconstruction. Am J Orthop 45.5 (2016): 320-4.

  48. Question 48 of 50
    48. Question


    A 16 year old girl presents to your office with her mother with complaints of worsening bilateral lower extremity pain over the last 2 months. She has been running 6-8 miles every day over the last few months for her cross country running team. Over the last 3 weeks she has “trained a little harder than usual” due to an upcoming state tournament. She admits to a normal diet and has had regular periods each month for the past year. Her medications include an oral contraceptive to help with acne but otherwise she has no past medical history. Her primary care physician ordered a bone density test which was normal. Coronal MRI image (figure 1) shows bilateral tibial stress fractures. Which choice is the most likely cause of her stress fractures?

    Correct

    Excessive endurance training in athletics can alter hormone balances and have negative consequences on the structural integrity of bone. Skeletal bone is constantly remodeled and repaired in response to stress placed during weight bearing activities. Bone is resorbed in response to stress and replaced with new bone. A period of rest, or removing weight bearing stresses, is critical to allow new bone formation. If repetitive stress is constantly placed on bone, microfractures may occur as bone formation lags behind resorbtion. Increasing endurance training can further damage bone and propagate stress fractures. Tibial shaft fractures pose a risk of fracture displacement if continued sport participation is allowed. Resolution of symptoms often takes 8-12 months of rest. Estrogen and testosterone are two hormones that influence bone remodeling. Estrogen slows bone resorbtion and helps maintain adequate bone density. Female athletes that participate in endurance sports like long distance running are at risk of low estrogen levels and amenorrhea. Menstrual cycle changes can be influenced by exercise as running 10 miles per week has a 6% incidence of amenorrhea while running 80 miles a week has a 43% incidence of amenorrhea. The patient has a normal diet, normal caloric intake, and a normal bone density which most likely rules out low caloric intake, estrogen levels, and metabolic bone disease as causes of the patient’s stress fracture. 1.2

    Answer B.

     

    References

    1. Tanaka MJ, Szymanski LM, Dale JL, Dixit S, Jones L. Team Approach: Treatment of Injuries in the Female Athlete. JBJS Reviews – Team Approach Review Articles: 22 January 2019 – Volume 7 – Issue 1 – p. e7-e
    2. Shindle, Michael K. MD; Endo, Yoshimi MD; Warren, Russell F. MD; Lane, Joseph M. MD; Helfet, David L. MD; Schwartz, Elliott N. MD; Ellis, Scott J. MD Stress Fractures About the Tibia, Foot, and Ankle, Journal of the American Academy of Orthopaedic Surgeons: March 2012 – Volume 20 – Issue 3 – p 167-176 doi: 10.5435/JAAOS-20-03-167.
    Incorrect

    Excessive endurance training in athletics can alter hormone balances and have negative consequences on the structural integrity of bone. Skeletal bone is constantly remodeled and repaired in response to stress placed during weight bearing activities. Bone is resorbed in response to stress and replaced with new bone. A period of rest, or removing weight bearing stresses, is critical to allow new bone formation. If repetitive stress is constantly placed on bone, microfractures may occur as bone formation lags behind resorbtion. Increasing endurance training can further damage bone and propagate stress fractures. Tibial shaft fractures pose a risk of fracture displacement if continued sport participation is allowed. Resolution of symptoms often takes 8-12 months of rest. Estrogen and testosterone are two hormones that influence bone remodeling. Estrogen slows bone resorbtion and helps maintain adequate bone density. Female athletes that participate in endurance sports like long distance running are at risk of low estrogen levels and amenorrhea. Menstrual cycle changes can be influenced by exercise as running 10 miles per week has a 6% incidence of amenorrhea while running 80 miles a week has a 43% incidence of amenorrhea. The patient has a normal diet, normal caloric intake, and a normal bone density which most likely rules out low caloric intake, estrogen levels, and metabolic bone disease as causes of the patient’s stress fracture. 1.2

    Answer B.

     

    References

    1. Tanaka MJ, Szymanski LM, Dale JL, Dixit S, Jones L. Team Approach: Treatment of Injuries in the Female Athlete. JBJS Reviews – Team Approach Review Articles: 22 January 2019 – Volume 7 – Issue 1 – p. e7-e
    2. Shindle, Michael K. MD; Endo, Yoshimi MD; Warren, Russell F. MD; Lane, Joseph M. MD; Helfet, David L. MD; Schwartz, Elliott N. MD; Ellis, Scott J. MD Stress Fractures About the Tibia, Foot, and Ankle, Journal of the American Academy of Orthopaedic Surgeons: March 2012 – Volume 20 – Issue 3 – p 167-176 doi: 10.5435/JAAOS-20-03-167.
  49. Question 49 of 50
    49. Question


    A 25 year old construction worker presents to your office with right knee pain after falling at work. He was walking down a hill when his feet slipped and his right knee hyperflexed. He felt a “pop” in the knee as he fell. He had immediate onset of pain and inability to bear weight. On physical exam the patient is unable to extend his right knee. Lateral radiograph of the right knee is shown in figure 1 and Sagittal MRI image is shown in figure 2. What is the most likely diagnosis?

    Correct

    An extensor mechanism injury should be suspected when patients are unable to perform knee extension or hold their leg straight in the air. The extensor mechanism of the knee consists of the quadriceps tendon, patella, and the patella tendon. Injury to any of these structures can cause impairment of knee extension or what is often referred to as an extension lag. Injuries to the quadricep and patella tendons typically occur with a forceful quadriceps contraction to a flexed knee. Patella fractures are the most common reason for an extensor mechanism injury and generally occur with a direct impact to the knee. In general quadriceps tendon ruptures occur in patients over 40 years of age while patella tendon ruptures typically occur in patients under 40. 1,2
    The patient has patella alta, or a high riding patella on his lateral x-ray. The length of the patella tendon (measured from the tibial tubercle to the inferior pole of the patella) should be roughly equal to the length of the patella. The patient’s MRI shows a complete rupture of the patella tendon with detachment off the inferior pole of the patella and attenuation of the tendon. Patella tendon ruptures require timely surgery with primary repair of the tendon back to the inferior pole of the patella. 1,2
    Answer B
    References
    1. Garner MR, Gausden E, Berkes MB, Nguyen JT, Lorich DG. Extensor Mechanism Injuries of the Knee. The Journal of Bone and Joint Surgery – Scientific Articles: 07 October 2015 – Volume 97 – Issue 19 – p. 1592-1596
    2. Patella tendon ruptures. http://www.orthobullets.com. Accessed on 5/14/20.

    Incorrect

    An extensor mechanism injury should be suspected when patients are unable to perform knee extension or hold their leg straight in the air. The extensor mechanism of the knee consists of the quadriceps tendon, patella, and the patella tendon. Injury to any of these structures can cause impairment of knee extension or what is often referred to as an extension lag. Injuries to the quadricep and patella tendons typically occur with a forceful quadriceps contraction to a flexed knee. Patella fractures are the most common reason for an extensor mechanism injury and generally occur with a direct impact to the knee. In general quadriceps tendon ruptures occur in patients over 40 years of age while patella tendon ruptures typically occur in patients under 40. 1,2
    The patient has patella alta, or a high riding patella on his lateral x-ray. The length of the patella tendon (measured from the tibial tubercle to the inferior pole of the patella) should be roughly equal to the length of the patella. The patient’s MRI shows a complete rupture of the patella tendon with detachment off the inferior pole of the patella and attenuation of the tendon. Patella tendon ruptures require timely surgery with primary repair of the tendon back to the inferior pole of the patella. 1,2
    Answer B
    References
    1. Garner MR, Gausden E, Berkes MB, Nguyen JT, Lorich DG. Extensor Mechanism Injuries of the Knee. The Journal of Bone and Joint Surgery – Scientific Articles: 07 October 2015 – Volume 97 – Issue 19 – p. 1592-1596
    2. Patella tendon ruptures. http://www.orthobullets.com. Accessed on 5/14/20.

  50. Question 50 of 50
    50. Question


    A 15 year-old male presents to your office 10 months out from a right midshaft clavicle fracture. At the time of injury he was playing basketball when he fell to the ground on his right shoulder. Figure 1 is an AP radiograph of the right shoulder taken one week after the injury. His motion has improved over the last 3 months but he continues to have mild discomfort with overhead activities. Figure 2 is an AP of the clavicle taken at the 10-month office visit. Which description below best describes the fracture?

    Correct

    The process of fracture healing usually takes place over 6 weeks and is typically complete in all skeletal sites by 3 to 4 months. The size and location of a fracture can determine healing time. Smaller bones in the hand may take 4-6 weeks to heal while a femoral shaft fracture may take 3 months. Fracture union is determined by the presence of bridging callus on radiographs and pain free full weight bearing of the involved extremity. A fracture with a persistent fracture line and pain with weight bearing for a period of 9 months, and without radiographic progression of healing for 3 months, is considered a non-union. Non-unions are divided into two types: hypervascular and avascular non-unions. Hypervascular non-unions have good blood supply to the ends of the fracture fragments and usually have some degree of bridging callus. Hypervascular non-unions will show vascularity of the fragments on bone scan. However hypervascular non-unions never fully heal as a result of motion through the fracture site. Hypervascular non-unions can have hypertrophic fracture ends rich with callus often called “elephant foot” non-unions. Oligotrophic non-unions also have vascular fracture ends but have minimal to no callus formation. For avascular non-unions the ends of the fracture fragments have no circulation and no healing potential. Avascular non-unions have persistent fractures lines without any bridging callus. A pseudarthrosis occurs when a hypervascular non-union forms a false joint as fracture motion “tricks” the body into thinking the fracture is a joint. A pseudarthrosis may be hypertrophic or oligotrophic in appearance but often has no pain with weight bearing activities. A pseudarthrosis may also present with new onset of pain and swelling 12 months or more after a clavicle fracture. A joint capsule may be seen during operative inspection of a pseudarthrosis non-union. 1,2
    Answer A.
    References
    1. Frölke, Jan Paul M., and Peter Patka. “Definition and classification of fracture non-unions.” Injury 38 (2007): S19-S22.
    2. Non-union. http://www.orthobullets.com. Accessed on 7/20/20.

    Incorrect

    The process of fracture healing usually takes place over 6 weeks and is typically complete in all skeletal sites by 3 to 4 months. The size and location of a fracture can determine healing time. Smaller bones in the hand may take 4-6 weeks to heal while a femoral shaft fracture may take 3 months. Fracture union is determined by the presence of bridging callus on radiographs and pain free full weight bearing of the involved extremity. A fracture with a persistent fracture line and pain with weight bearing for a period of 9 months, and without radiographic progression of healing for 3 months, is considered a non-union. Non-unions are divided into two types: hypervascular and avascular non-unions. Hypervascular non-unions have good blood supply to the ends of the fracture fragments and usually have some degree of bridging callus. Hypervascular non-unions will show vascularity of the fragments on bone scan. However hypervascular non-unions never fully heal as a result of motion through the fracture site. Hypervascular non-unions can have hypertrophic fracture ends rich with callus often called “elephant foot” non-unions. Oligotrophic non-unions also have vascular fracture ends but have minimal to no callus formation. For avascular non-unions the ends of the fracture fragments have no circulation and no healing potential. Avascular non-unions have persistent fractures lines without any bridging callus. A pseudarthrosis occurs when a hypervascular non-union forms a false joint as fracture motion “tricks” the body into thinking the fracture is a joint. A pseudarthrosis may be hypertrophic or oligotrophic in appearance but often has no pain with weight bearing activities. A pseudarthrosis may also present with new onset of pain and swelling 12 months or more after a clavicle fracture. A joint capsule may be seen during operative inspection of a pseudarthrosis non-union. 1,2
    Answer A.
    References
    1. Frölke, Jan Paul M., and Peter Patka. “Definition and classification of fracture non-unions.” Injury 38 (2007): S19-S22.
    2. Non-union. http://www.orthobullets.com. Accessed on 7/20/20.

Proudly powered by WordPress | Theme: Motif by WordPress.com.